Sie sind auf Seite 1von 529

1. Each Student of a class collected some money for the trip.

The total
money collected by each student was equal to the cube of the total
number of students. If the total amount collected was Rs. 29791. find
the total number of students.
1. 31
2. 30
3. 28

4. 37
5. 39

2. A dealer buys an article marked at Rs. 50,000 at 25% and 10%


successive discount. He spends Rs. 5000 on its repair and sells it for Rs.
50,000. What is his loss or gain percentage?
1. 33.63% loss
2. 26.43% gain
3. 1.25% gain
4. 29.03%
5. 27.13% loss

3. The average age of four boys, five years ago was 9 of all the five is
15 years. The present age of the new boy is
1. 22 years
2. 24 years
3. 21 years
4. 18 years
5. 19 years

4. P, Q and R can do a piece of work in 18,24 and 36 days on the second


day and P on the third day, again R on the fourth day and so on. Then in
how, many days will the work be completed?
1. 25
2. 24
3. 22
4. 20
5. 26

5. Harshika wants to purchase a mobile handset. The shopkeepers told


her to pay 30% tax if she asked for the actual sale price (without tax)
of the mobile and paid to the shopkeeper Rs. 16150. In the doing so,
she managed to avoid to pay the 30% tax. What is the amount of
discount that she received on the selling price (inclusive of tax)?

www.bankingpdf.com
1. Rs. 5495
2. Rs. 5850
3. Rs. 5950
4. Rs. 5685
5. Rs. 5975

6. The diluted wine contains only 10 litres of wine and the rest is water.
A new mixture is formed by replacing mixture with the pure wine
whose concentration is 40%. How many litres of mixtures was replaced
if initially there were 40 litres of water in the mixture?
1. 10 litre
2. 12 litre
3. 9 litre
4. 11.5 litre
5. 12.5 litre

7. A tank can be filled by a tap in 6 hours and emptied by an outlet pipe


in 8 hours. How long will it take to fill the cistern if both the taps are
opened together?
1. 24 hours
2. 22 hours
3. 21 hours
4. 18 hours
5. 20 hours

Answer :

1. 1;
Total number of students

2. 4;
CP of the article

∴ Profit percentage

www.bankingpdf.com
= 29.03% gain

3. 5;
Five years ago the average age of 4 boys = 9 years
The present average age of 4 boys = 9+5 = 14
So the present age of new boys = 15 + 4 x 1 = 19 years

4. 2;

Units of work done in 3 days = (2 + 3 + 4) = 9


∴ 72 units of work will be done in = 3 x 8 = 24 days

5. 3;
SP = 100, SP (with tax) = 130
New SP = 100 - 5 = 95
Effective discount = 130 - 95 = 35
So, when SP of 95, discount = 35
And when SP of Rs. 16,150, discount = 35/95x16150 = Rs. 5950

6. 5;
Initial ratio of water to wine = 40 : 10 = 4: 1
Final ratio of water to wine = (4 : 1) = 24:6
Required, final ratio of water to wine
= 60 : 40
= (6 : 4)
= 24 : 16x⁴
If total mixture is 40 litres, mixture replaced is = 10 litres
Here total mixture is 50 litre, mixture replaced is = 12.5 litres

7. 1;

www.bankingpdf.com
Total time required to fill the tank = 24/4-3 = 24 hours

Number Series & Simplification : 10 questions

Directions (Q. 1-5): What should come in place of question mark(?) in the following
number series?
1). 65 219 517 1007 1737 ?
a) 2213
b) 2000
c) 2755
d) 2765
e) 2855

2). 561 642 763 932 1157 ?


a) 1446
b) 1326
c) 1482
d) 1246
e) None of these

3). 1524 1443 1394 ? 1360 1359


a) 1303

www.bankingpdf.com
b) 1218
c) 1359
d) 1369
e) 1329

4). 5 13 58 357 2868 ?


a) 25823
b) 28695
c) 29548
d) 28545
e) 27695

5). ? 61 211 505 991 1717


a) 6
b) 5 .
c) 8
d) 7
e) 10

Directions (Q. 6-10): What approximate value should come in place of question mark (?)
in the following questions? (Note: You are not expected to calculate the exact value.)
6). 512.01 x 412.99 ÷ 119 =?
a) 1720
b) 1740
c) 1845
d) 1775
e) None of these

www.bankingpdf.com
7).1699.99 x 299.88 ÷ 59.9 -1498 + 3745 = ?
a) 10980
b) 11700
c) 11000
d) 10750
e) 9800

8). (13.96)2 + (16.23)2 + (17.26)2 – 32.95 = ?


a) 790
b) 720
c) 840
d) 780
e) 680

9). 1624.98 x 29.92 + 468.75 =?


a) 49290
b) 48220
c) 47220
d) 46365
e) None of these

10). 8499.99 ÷ 375.002 x 14.996 =?


a) 360
b) 290
c) 480
d) 380
e) 340

www.bankingpdf.com
Solutions:

1). The series is 43+1, 63+3, 83+5, 103+7, 123+9, 143=11,…


So, the answer will be 2755.
Answer: c)
2). The series is +92, +112, +132, +152, ++172
Answer: a)
3). The Series is -92, -72, -52, -32, -12
Answer: d)
4). The series is 5x2+3 = 13, 13x4+6, 58x6+9=357, 357x8+12,…
Answer: b)
5). The series is 23-1, 43-3, 63-5, 83-7,…
Answer: d)
6). ?= 512.01x (412.99/119) = (512x413)/(17x7)
= (510x413)/ (17x7) = 30x59 = 1770 ≈1775
Answer: e)
7). ?= (1700x300)/ 60 – 1498+3745
= 510000/60 – 1490+3745
=8500-1498+3745
= 12245 – 1498 = 10747 ≈ 10750
Answer: d)
8). ? = (14)2 + (16.2)2 + (17.25)2-33
=196+262.44+297.56-33
= 756 – 33- = 723 = 720 (Approx)
Answer: b)
9). ? = 1625 x 30 + 469
=48750 + 469 = 49219 ≈ 49220

www.bankingpdf.com
Answer: c)
10). ? = (8500/375)x 15 ≈ 340
Answer: e)

20-30 Questions :

Profit and Loss


1. A manufacture undertakes to supply 2000 pieces of a particular
component at Rs.25 per piece. According to his estimates, even
if 5% fail to pass the quality tests, then he will make a profit of
25%. However as it turned out, 50% of the components were
rejected. What is the loss to the manufacture? [TCS recruitment
exam question]
A. Rs 12,000
B. Rs 13,000
C. Rs 14,000
D. Rs 15,000
E. None f these

Solution

Answer – B. Rs 13,000
Explanation:
5% of 2000=100 so 2000-100=1900 so,if he sells 1900 he will get
25% profit
cost per piece rs 25……so 25x 1900 don’t solve here
if 125%…………………..25×1900
100 %………………….?
?= 25x1900x100/125=38000=CP
if 50% rejected,only 1000 pieces sold

www.bankingpdf.com
so 1000×25=25000=SP
loss=cp-sp= 38000-25000=13000

2. Shan bought 30 liters of milk at the rate of Rs.8 per liter. He got
it churned after spending Rs.10 and 5kg of cream and 30 liter of
toned milk were obtained. If he sold the cream at Rs.30 per kg
and toned milk at Rs.4 per liter, his profit in the transaction is:
A. 20%
B. 8%
C. 30%
D. 40%

Solution

Answer – B .8%
Explanation:
CP =Rs.(30 x 8 + 10) = Rs.250
SP = Rs. (30 x 5 + 30 x 4) = Rs.270
Gain% = (20/250 x 100)% = 8%

3. The sale price of an article including the sale tax is Rs. 616. The
rate of sale tax is 10% . If the shopkeeper has made a profit of
12%, the cost price of the article is [The Pearson Guide book]
A. Rs 500
B. Rs 515
C. Rs 550
D. Rs 600

Solution

Answer – A. Rs 500
Explanation – 110% of S.P. = 616
= S.P. = Rs.(616 x 100/110) = Rs.560.
C.P. = Rs (100/112 x 560)
= Rs 500

www.bankingpdf.com
4. When a article is sold for Rs.3400, there is a loss of 2%. What is
the cost price of the commodity?
A. Rs 3500.50
B. Rs 3200
C. Rs 3400.56
D. Rs 3469.34

Solution

Answer – D. Rs 3469.34
Explanation – loss=2% so,
98%………….3400
100%………..?
?=(3400×100)/98=3469.34

5. The marked price of a watch was Rs. 720.A man brought the
same for Rs.550.80 after getting two successive discounts the
first being 10% .What was the second discount rate?
A. 12%
B. 14%
C. 15%
D. 18%

Solution

Answer – C. 15%
Explanation – Let the second discount rate be x% Then,
(100 – x)% of 90% of 720 = 550.80
=> (100 – x)/100 x 90/100 x 720 = 550.80
= (100-x) = [55080/(9 x 72) = 85
=> x= 15
second discount rate =15%

6. Jagdeep bought a refrigerator with 20% discount on the


labeled price. Had he bought at it with 30% discount, he would

www.bankingpdf.com
have saved Rs. 500 more. At what price did he buy the
refrigerator?
A. Rs 5000
B. Rs 10,000
C. Rs 12,500
D. Rs 15,000

Solution

Answer – A. Rs 5,000
Explanation – Let the labelled price be Rs.x Then,
(80% of x)- (70% of x) = 500
10% of x =500
10%………..500
100%…………? ?= (500 x 100/10) = 5000=x

7. If the S.P of Rs.40 results in a 20% discount on list price. What


S.P would result in a 30% discount on list price?
A. Rs 18
B. Rs 20
C. Rs 35
D. Rs 27

Solution

Answer – C. Rs 35
Explanation – Let the list price be Rs. x
80/100x = 40
=> x = 40 x 100/80 = 50
∴ Required S.P. = 70% of Rs. 50=Rs.35

8. A discount of 25% on one article is same as a discount of 50%


on another article .The costs of two article can be:
A. Rs 30, Rs 20
B. Rs 90, Rs 40

www.bankingpdf.com
C. Rs 80, Rs 40
D. Rs 50, Rs 40

Solution

Answer – C. Rs 80, Rs 40
Explanation – Let the costs of the two articles be x and y. Then,
25% of x = 50% of y
=> x/y = 50/25 = 2/1
So,x and y must be in the ratio of 2:1

9. A article is listed at Rs. 2000 and a discount of 20% is offered


on the list price .What additional discount must be offered to
the customer bring the net price to Rs. 1400?
A. 12.5%
B. 10%
C. 12%
D. 15%

Solution

Answer – A. 12.5%
Explanation – S.P after 1st discount,
100%………2000
80%………?
?=2000×80/100=1600
Net S.P = Rs. 1400.
Discount on Rs. 1600 = Rs. 200.
∴Required discount = (200/1600 x 100)% =12.5%

10. A shopkeeper gives 12% additional discount on the


discounted price, after giving an initial discount of 20% on the
labeled price of a mobile. If the final sale price of the mobile is
704. then what is its labelled price?
A. Rs 844

www.bankingpdf.com
B. Rs 920
C. Rs 1000
D. Rs 1100

Solution

Answer – C. Rs 1000
Explanation – Let the labeled price be Rs. x
88% of 80% of x = 704
=> x = (704 x 100 x 100/88 x 80)= 1000.

11. The difference between the cost price and sale price of an
article is Rs. 500 if the profit is 20%. The selling price is:
A. Rs 4000
B. Rs 1500
C. Rs 3000
D. Rs 3300

Solution

Answer – C. Rs.3000
Explanation – 120%-100%=20%
20%…………………500
120%…………….?
?=3000

12. A dealer sold a Radio at a loss of 2.5%. Had he sold it for Rs.
100 more, he would have gained 7.5%. To gain 12.5% he
should sell it for :
A. Rs 2200
B. Rs 1000
C. Rs 1100
D. Rs 1125

Solution

www.bankingpdf.com
Answer – D. Rs.1125
Explanation – Let C.P be Rs.x then,(107.5 % of x) – (97.5 % of x) =
100
=> 10% of x = 100
=> x =1000
∴ desired S.P = 112.5 % of Rs. 100
= Rs.(225/2 x 1/100 x 1000) =Rs. 1125

30-40 Questions

Averages

1. 3 years ago, the average of a family of 5 members was 17


years. A baby having been born, the average age of the family
is the same today. The present age of the baby is:
A.5 years
B.2 years
C.1 year
D.4 years
E.None of these

Solution

Answer – B (2 years)
Explanation – Total age of 5 members, 3 years ago = (17 x 5) years
= 85 years
Total age of 5 members now = (85 + 3 x 5) years = 100 years
Total age of 6 members now = (17 x 6) years = 102 years
Age of the baby = (102 – 100) years = 2 years

2. Of the four numbers, the first is twice the second, the second is
one-third of the third and the third is 5 times the fourth. The

www.bankingpdf.com
average of the numbers is 24.75. The largest of these numbers
is:
A.45
B.25
C.30
D.45
E.None of these

Solution

Answer – D (45)
Explanation – Let the fourth number be a
then hen, third number = 5a, second number= 5a/3 and first=10a/3
a+ 5a+ 5a/3 + 10/3 = 24.75 x 4
so,a=9
So, the numbers are 9, 45, 15 and 30

3. The average age of students of a class is 15.8 years. The


average age of boys in the class is 16.4 years and that of the
girls is 15.4 years. The ratio of the number of boys to the
number of girls in the class is:
A. 1 : 4
B. 2 : 3
C. 3 : 4
D. 4 : 2
E. None of these

Solution

Answer – B (2 : 3)
Explanation – Let the ratio be k:1
Then, k x 16.4 + 1 x 15.4 = (k + 1) x 15.8
(16.4 – 15.8) k = (15.8 – 15.4)
k=2/3

www.bankingpdf.com
4. The average price of 10 books is Rs. 12 while the average price
of 8 of these books is Rs. 11.75. Of the remaining two books, if
the price of one book is 60% more than the price of the other,
what is the price of each of these two books?
A.Rs 10 and Rs 16
B.Rs 12 and Rs 24
C.Rs 24 and Rs 18
D.Rs 28 and Rs 12
E.None of these

Solution

Answer – A (Rs 10 and Rs 16)


Explanation – Total price of the two books = Rs. [(12 x 10) – (11.75
x 8)] = Rs. (120 – 94) = Rs. 26
Let the price of one book be Rs.x
Then, the price of other book = Rs. (x + 60% of x )= x + (3/5)x=
(8/5)x
so, x +(8/5)x=26 , x=10
The prices of the two books are Rs. 10 and Rs. 16

5. The average age of 30 boys of a class is equal to 14 years.


When the age of the class teacher is included the average
becomes 15 years. Find the age of the class teacher?
A.50
B.44
C.45
D.42
E.None of these

Solution

Answer – C (45)
Explanation – Total ages of 30 boys = 14 x 30 = 420 years

www.bankingpdf.com
Total age when class teacher is included = 15 x 31 = 465 years
Age of class teacher = 465 – 420 = 45 years

6. The Average of marks obtained by 120 candidates in a certain


examination is 35. If the average marks of passed candidates is
39 and that of failed candidates is 15, what is the number of
candidates who passed the examination?
A.90
B.100
C.108
D.115
E.None of these

Solution

Answer – B (100)
Explanation – Let the number of passed candidates be a
Then total marks =>120 x 35 = 39 a + (120 – a) x 15
4200 = 39 a + 1800 – 15 a
a = 100

7. The average of 11 results is 50. If the average of first 6 results


is 49 and that of last 6is 52, find the sixth result?
A.55
B.56
C.65
D.62
E.None of these

Solution

Answer – B (56)
Explanation – The total of 11 results = 11 x 50 = 550
The total of first 6 results = 6 x 49 = 294
The total of last 6 results = 6 x 52 = 312

www.bankingpdf.com
The sixth result is common to both:
Sixth result = 294 + 312 – 550 = 56

8. The average age of a family of 6 members is 22 years. If the


age of the youngest member be 7 years, then what was the
average age of the family at the birth of the youngest member?
A.15
B.18
C.28
D.24
E.None of these

Solution

Answer – B (18)
Explanation – Total age of all members = 6 x 22 = 132 years
7 years ago, total sum of ages = 132 – (6 x 7) = 90 years
But at that time there were 5 members in the family
Average at that time = 90 /5 = 18 years

9. A batsman in his 17th innings makes a score of 85, and thereby


increases his average by 3. What is his average after 17
innings?
A.30
B.37
C.40
D.45
E.None of these

Solution

Answer – B (37)
Explanation – Let the average after 16th innings be a, then total
score after 17th innings =>
16a+85 = 17 (a+3)

www.bankingpdf.com
a = 85-51 = 34
Average after 17 innings = a + 3 = 34 + 3 = 37

10. There were 35 students in a hostel. If the number of


students increase by 7, the expenses of the mess increase by Rs
42 per day while the average expenditure per head diminishes
by Rs 1. Find the original expenditure of the mess?
A.450
B.420
C.430
D.410
E.None of these

Solution

Answer – B (420)
Explanation – Suppose the average expenditure was Rs a.
Then total expenditure = 35a
When 7 more students join the mess, total expenditure = 35a + 42
Now, the average expenditure= (35a+42) / (35 + 7)
Now, we have (35a + 42)/ 42 =(a – 1)
or, 35a + 42 = 42a – 42
7a = 84
a = 12

40-50 Questions :

Ages

1. The sum of ages of family members (both children and parents)


is 360 years.The total ages of children and parents are in the
ratio 2:1 and the ages of wife and husband are in the ratio
5:7.What will be the age of husband?

www.bankingpdf.com
A.65
B.75
C.72
D.70

Answer

Answer : D (70)
Explanation-
Sum of ages is 360 years.The ratio of children and parents ages is 2:1.
Total age of parents = 360 x 1 / 3 = 120 years
Ratio of wife and husband age is 5:7. Therefore, the age of husband =
120 x 7 / 12 = 70
Hence the age of husband is 70 years.

2. The ratio between the present ages of A and B is 6:7. If B is 4


years old than A. What will be the ratio of the ages of A and B
after 4 years?
A.5 : 7
B.7 : 8
C.8 : 7
D.Data inadequate
E.None of these

Answer

Answer – B (7 : 8)
Explanation –
Let A’s age and B’s age be 6X years and 7X years respectively
Then, 7X – 6X = 4
X=4
Required ratio = (6X + 4) : (7X + 4) = (6*4+4) : (7*4 +4) =28 : 32 =
7:8

www.bankingpdf.com
3. The sum of ages of family members (both children and parents)
is 360 years.The total ages of children and parents are in the
ratio 2:1 and the ages of wife and husband are in the ratio
5:7.What will be the age of husband?
A.65
B.75
C.72
D.70

Answer

Answer : D (70)
Explanation-
Sum of ages is 360 years.The ratio of children and parents ages is 2:1.
Total age of parents = 360 x 1 / 3 = 120 years
Ratio of wife and husband age is 5:7. Therefore, the age of husband =
120 x 7 / 12 = 70
Hence the age of husband is 70 years.

4. Sush was thrice as old as Poonam 6 years back. Sush will be


5/3 times as old as Poonam 6 years hence. How old is Poonam
today?
A.20
B.14
C.12
D.15
E.None of these

Answer

Answer – C (12)
Explanation –
Let Poonam age 6 years back = x.
Then, Sush age 6 years back = 3x.
(5/3) * (X + 6 + 6) = (3X + 6 + 6)

www.bankingpdf.com
So 5(x+ 12) = 3(3x+ 12),
so x=6.
Poonam Age = (x+ 6) years = 12 years

5. Present ages of Simmi and Anu are in the ratio of 5 : 4


respectively. Three years hence, the ratio of their ages will
become 11 : 9 respectively, What is Anu’s present age in years?
A.20
B.24
C.28
D.35
E.None of these

Answer

Answer – B (24)
Explanation –
Let the present ages of Simmi and Anu be 5x years and 4x years
respectively
Then, [5X + 3] / [4X + 3] =11/9
x=6
Anu’s present age = 4X = 24 years

6. If 6 years are subtracted from the present age of Anuj and the
remainder is divided by 18, then the present age of his
grandson Gopal is obtained.If Gopal is 2 years younger to
Mohan whose age is 5 years, then what is the age of Anuj?
A.44
B.60
C.80
D.92

Answer

www.bankingpdf.com
Answer : B (60)
Explanation –
Let Anuj’s age be X
Gopal is 2 years younger than Mohan, so Gopal is 3 years (i.e 5 – 2 =
3)
If Arun had born 6 years before, his age would had been X – 6. As per
the question, X – 6 should be 18 times as that of Gokul’s age.
i.e. (X – 6) /18= 3
X-6 =3 x 18
x = 60

7. Ten years ago, Khush was thrice as old as Sam was but 10
years hence, he will be only twice as old. Find Khush’s present
age?
A.30
B.35
C.40
D.36
E.None of these

Answer

Answer – A (30)
Explanation –
Let Khush’s present age be x years and Sam’s present age be y years
Then, according to the first condition,
x – 10 = 3(y – 10)
or, x – 3y = -20 ……..(1)
Now. Khush’s age after 10 years = (x + 10) years
Sam’s age after 10 years = (y + 10)
(x+10) = 2(y+10)
or, x – 2y = 10 ……..(2)
Solving (1) and (2), we get

www.bankingpdf.com
x = 70 and y = 30
Khush’s age = 70 years and Sam’s age = 30 years

8. Father is aged three times more than his son Mohit. After 8
years, he would be two and a half times of Mohit’s age. After
further 8 years, how many times would he be of Mohit’s age?
A.4
B.2
C.2.5
D.3
E.None of these

Answer

Answer – B (2)
Explanation –
Let Monit’s present age be X years. Then, father’s present age = (X +
3X) years = 4X years.
(4X + 8) =5/2 x (X + 8)
8X + 16 = 5X + 40
3X = 24 so, X=8
Hence, required ratio = (4X + 16) / (X + 16)=48 / 24 = 2

9. A father said to his son, “I was as old as you are at present at


the time of your birth.” If the father’s age is 38 years now, the
son’s age five years back was:
A.14
B.19
C.38
D.40
E.None of these

Answer

www.bankingpdf.com
Answer – A (14)
Explanation – Let son’s present age be X years. Then, (38 – X) = X
2X = 38
X = 19
Son’s age 5 years back = (19 – 5) years = 14 years

10. One year ago the ratio between Sam and Ash’s age was 4:3.
One year hence the ratio of their ages will be 5:4. What is the
sum of their present ages in years?
A.14
B.16
C.18
D.12
E.None of these

Answer

Answer – B (16)
Explanation – Let one year ago
Sam’s age be 4X years
And, Ash’s age be 3X years
Present age of Sam = (4X + 1) years
Present age of Ash= (3X + 1) years
One year hence
Sam’s age = (4X + 2) years
Ash’s age = (3X + 2) years
According to question,
[4X + 2] divide by [3X + 2] = 5 / 4
16X + 8 = 15X + 10
or, X = 2
Sum of their present ages = 4X + 1 + 3X + 1
= 7X + 2
= 7 x 2 + 2 = 16 years.

www.bankingpdf.com
60- 70 Questions:

Simple Interest
 A portion of $6600 is invested at a 5% annual return, while the remainder is invested at
a 3% annual return. If the annual income from the portion earning a 5% return is twice
that of the other portion, what is the total income from the two investments after one year?
A.200
B.270
C.250
D.280
E.none of these
Answer
Answer- B (270)
Explanation – 5x + 3y = z (total)
x + y = 6600
5x= 2(3y) [ condition given] 5x – 6y = 0
x + y = 6600
5x -6y = 0
Subtract both equations and you get x = 3600 so y = 3000
3600*.05 = 180
3000*.03 = 90
z (total) = 270
 If the simple interest on a certain sum of money is 4/25th of the sum and the rate percent
equals the number of years, then the rate of interest per annum is:
A.2%
B.4%
C.8%
D.10%
E.None of these
Answer
Answer – B (4%)
Explanation – let principal =x then SI= 4/25 x
let rate be ” r” then time =r years
SI= PXRXT /100
put all here all will cut and we left with
r ^2 =400/25 = 4%
 An automobile financier claims to be lending money at the simple interest, but he
includes the interest every six months for calculating the principal. If he is charging an
interest of 10%, the effective rate of interest becomes:
A.10%
B.10.25%%
C.10.10%

www.bankingpdf.com
D.10.80%
E.None of these
Answer
Answer – B (10.25%)
Explanation –
Let the sum be Rs. 100. Then,
S.I. for first 6 months = Rs. [100 x 10 x 1] / [100×2]= Rs. 5.
S.I. for last 6 months = Rs.[105 x 10 x 1] / [100 x 2]= Rs. 5.25
So, amount at the end of 1 year = Rs. (100 + 5 + 5.25) = Rs. 110.25.
Effective rate = (110.25 – 100) = 10.25%.
 The difference between the simple interest received from two different sources on Rs.
1500 for 3 years is Rs. 13.50. The difference between their rate of interest is:
A.0.20%
B.0.30%
C.0.50%
D.0.80%
E.None of these
Answer
Answer-B (0.30%)
SI=PXRXT/100 so,SI( 1) – SI (2)
[1500 x R1 x 3] /100 – [1500 x R2 x 3] /100 =13.5
4500 (R1 – R2) = 1350
R1 – R2 =1350/4500=0.30%
 Nishu invested an amount of Rs. 13,900 divided in two different schemes A and B at the
simple interest rate of 14% p.a. and 11% p.a. respectively. If the total amount of simple
interest earned in 2 years be Rs. 3508, what was the amount invested in Scheme B?
A.6400
B.6500
C.7200
D.7500
E.None of these
Answer
Answer – A (6400)
Explanation –
Let the sum invested in Scheme A be Rs. x and that in Scheme B be Rs. (13900 – x).
Then, ( x X 14 X 2 ) /100 + [(13900 – x) X 11 X 2] /100= 3508
28x – 22x = 350800 – (13900 X 22)
6x = 45000
x = 7500.
So, sum invested in Scheme B = Rs. (13900 – 7500) = Rs. 6400.
 The difference between the Simple Interest and Compound Interest on a certain sum for
2 years at 15% Interest is Rs. 90. Find the Principal?
A.2500
B.3000

www.bankingpdf.com
C.3500
D.4000
E.None of these
Answer
Answer – D (4000)
Explanation –
Difference between the compound interest and simple interest for 2 years = D= p(r/100)^2
P= Dx(100 /R)^2 = 90x100x100 /15×15=4000
 If simple interest on a certain sum of money for 4 years at 5% per annum is same as the
simple interest on Rs. 560 for 10 years at the rate of 4% per annum then the sum of money
is:
A.1180
B.1120
C.1200
D.1250
E.None of these
Answer
Answer – B (1120)
Explanation – SI = PXRXT/100
make equation for both,and equate
 A sum of Rs. 800 amounts to Rs. 920 in 3 years at simple interest. If the interest rate is
increased by 3%, it would amount to how much?
A.780
B.992
C.848
D.700
E.None of these
Answer
Answer – B (992)
Explanation –
S.I = Rs. (920 – 800) = Rs. 120; P = Rs. 800, T = 3 yrs
use SI=Px R x T/100 so, R = Si x 100 /Px t = ( 100 X 120 ) / 800 X 3 = 5%
New rate = (5 + 3) % = 8%
New S.I. = Rs. (800 X 8 X 3)/100 == Rs. 192.
New amount = Rs. (800 + 192) = Rs. 992
 David invested certain amount in three different schemes A, B and C with the rate of
interest 10% p.a., 12% p.a. and 15% p.a. respectively. If the total interest accrued in one
year was Rs. 3200 and the amount invested in Scheme C was 150% of the amount invested
in Scheme A and 240% of the amount invested in Scheme B, what was the amount invested
in Scheme B?
A.Rs 5000
B.Rs 6500
C.Rs 8000

www.bankingpdf.com
D.Rs 10000
E.None of these
Answer
Answer – A (Rs 5000)
Explanation:-Let x, y and z be the amounts invested in schemes A, B and C respectively. Then,
add individual interest to get total using Si= pxrxt/100
[x x 10 x 1]/100 + [y x 12 x 1]/100 + [z x 15 x 1]/100 = 3200
10x + 12y + 15z = 320000…. (i)Now, z = 240% of y =(12/5)y……… (ii)And, z = 150% of x
=(3/2)x so,x=(2/3 )z = (2/3) x value of z from ii
x= (2/3) x (12/5) y = (8/5)y………..(iii)
From (i), (ii) and (iii), we have :
16y + 12y + 36y = 320000
64y = 320000
y = 5000
Sum invested in Scheme B = Rs. 5000
 A certain sum is invested for T years. It amounts to Rs. 400 at 10% per annum. But
when invested at 4% per annum, it amounts to Rs. 200. Find the time (T)?
A.39 years
B.41 years
C.45 years
D.50 years
E.None of these
Answer
Answer – D (50 years)
Explanation –
We have, A1 = Rs. 400, A2 = Rs. 200, R1 = 10%, R2 = 4%
Time (T) = [A1 – A2] x 100 divide by A2R1 – A1R2
= [400 – 200]x 100 divide by [200 x 10 – 400 x 4]= 20000/400 = 50 Years

70-80 Questions :

Data Interpretation

www.bankingpdf.com
1. What is the profit obtained by the two companies P and Q
together in 2013?
A. Rs.33 cr
B. Rs.37 cr
C. Rs.41 cr
D. Rs.45 cr
E. None of these

Answer

www.bankingpdf.com
B. Rs.37 cr

2. In 2010, what was the ratio of profit of the company P to the


profit of the company Q?
A. 10 : 7
B. 13 : 10
C. 16 : 13
D. 19 : 16
E.None of these

Answer

A.10 : 7

3. What is the approximate percentage increase in the total profit


of companies P and Q together from 2010 to 2012?
A. 8.45%
B. 13.35%
C. 17.65%
D. 20.25%
E. 24.15%

Answer

C. 17.65%

4. In which of the given years was the total profit of both the
companies together the highest?
A. 2009
B. 2010
C. 2011
D. 2012
E. 2013

Answer

www.bankingpdf.com
D.2012

5. In how many of the given years was the profit of the company P
more than the profit of the company Q by at least Rs.15 crore?
A. 1
B. 2
C. 3
D. 4
E. 5

Answer

C. 3

80-90 Questions :

Time and Distance

1. Nishu is standing on a railway bridge which is 180 m long. He


finds that a train crosses the bridge in 20 seconds but himself
in 8 seconds. Find the speed of the train?
A.35 kmph
B.54 kmph
C.62 kmph
D.70 kmph
E.None of these

Solution

Answer – B (54 kmph)


Explanation – Let the length of the train be x metres.
Then, the train covers x metres in 8 seconds (train is actually covering
itself because length of man is very less compartable to train)and (x +
180) metres in 20 seconds.

www.bankingpdf.com
equate speed in both case,s=d/t
∴ x/8 =( x + 180) / 20 ⇔ 20x = 8(x + 180) ⇔ x = 120.
∴ Length of the train = 120 m.
Speed of the train = [120/8]m/sec (convert to km/hr i.e x 18/5)
=[120/8] x 18/5kmph = 54 kmph.

2. Two trains are running at 40 km/hr and 20 km/hr respectively


in the same direction. Fast train completely passes a man
sitting in the slower train in 5 seconds. What is the length of
the fast train?
A.27 7/9 m
B.28 m
C.29 m
D.30 2/7 m
E.None of these

Solution

Answer – A (27 7/9 m)


Explanation – When SAME direction- MINUS
Relative speed = (40-20) km/hr = [20 x 5/18] m/sec = [50/9] m/sec.
Length of faster train = sxt=[50/9 x 5] m = 250/9 m = 27 7/9 m.

3. Two train travel in opposite directions at 36 kmph and 45 kmph


and a man sitting in slower train passes the faster train in 8
seconds. Then length of the faster train is:
A.120 m
B.140 m
C.160 m
D.180 m
E.None of these

Solution

www.bankingpdf.com
Answer – D (180 m)
Explanation – When OPP. direction-PLUS
Relative speed = (36 + 45) km/hr
= [81 x 5/18] m/sec = [45/2] m/sec.
Length of train = [45/2 x 8] m = 180 m.

4. Two trains of equal lengths take 10 seconds and 15 seconds


respectively to cross a telegraph post. If the length of each
train be 120 metres, in what time (in seconds) will they cross
each other travelling in opposite direction?
A.12 sec
B.18 sec
C.14 sec
D.25 sec
E.None of these

Solution

Answer – A (12 sec)


Explanation – Speed of the first train = [120 / 10] m/sec = 12 m/sec.
Speed of the second train = [120 / 15] m/sec = 8 m/sec.
Relative speed = (12 + 8) = m/sec = 20 m/sec.
∴ Required time = (120 + 120) / 20 secc = 12 sec

5. Two trains are running in opposite directions with the same


speed. If the length of each train is 120 metres and they cross
each other in 12 seconds, then the speed of each train (in
km/hr) is:
A.12 kmph
B.24 kmph
C.36 kmph
D.48 kmph
E.None of these

Solution

www.bankingpdf.com
Answer – C (36 kmph)
Explanation – Let the speed of each train be x m/sec.
Then, relative speed of the two trains = 2x m/sec.
So, 2x = (120 + 120)/12 ⇔ 2x = 20 ⇔ x = 10.
∴ Speed of each train = 10 m/sec = [10 x 18/5] km/hr = 36 km/hr

6. Excluding stoppages, the speed of a bus is 54 kmph and


including stoppages, it is 45 kmph. For how many minutes does
the bus stop per hour?
A.10 min
B.12 min
C.18 min
D.15 min
E.None of these

Solution

Answer – A (10 min)


Explanation – Due to stoppages, it covers 9 km less.Time taken to
cover 9 km = (9/54 x 60) min = 10 min

7. A motor car starts with the speed of 70 km/hr with its speed
increasing every two hours by 10 kmph. In how many hours
will it cover 345 kms?
A.4 hrs
B.4 hrs 5 mins
C.4 1/2 hrs
D.2 hrs
E.None of these

Solution

Answer – C (4 1/2 hrs)


Explanation – Distance covered in first 2 hours = (70 x 2) km = 140
km

www.bankingpdf.com
Distance covered in next 2 hours = (80 x 2) km = 160 km
Remaining distance = 345 – (140 + 160) = 45 km.
Speed in the fifth hour = 90 km/hr
Time taken to cover 45 km = as speed is 90km/hr means it covers 90
km in 1 hour
so,if 90km………..1 hr
45km……….?
?=45/90hr=1/2hr
Total time taken = 2 + 2 +1/2=4 (1/2)hrs

8. A person travels from P to Q at a speed of 40 kmph and returns


by increasing his speed by 50%. What is his average speed for
the both the trips?
A.35 kmph
B.4o kmph
C.48 kmph
D.55 kmph
E.None of these

Solution

Answer – C (48 kmph)


Explanation – Speed on return trip = 150% of 40 = 60 kmph
Average speed =
[2 x 40 x 60] / [40 + 6]km/hr =4800/100km/hr = 48 km/hr.

9. Three trains are running from a place A to another place B.


Their speeds are in the ratio of 4 : 3 : 5. The time ratio to reach
B by trains will be
A. 4 : 3 : 5
B. 5 : 3 : 4
C. 15 : 9 : 20
D. 15 : 20 : 12

Solution

www.bankingpdf.com
Answer: D(15 : 20 : 12)
Explanation: Ratio of speeds = 4 : 3 : 5
Therefore Ratio of times taken [t=d/s or t indirectly proportional to s
when distance is same]= (1/4) : (1/3) : (1/5) = 15 : 20 : 12

10. A man in a train notices that he can count 21 telephone


posts in one minute. If they are known to be 50 meters apart,
then at what speed is the train travelling?
A.50 kmph
B.54 kmph
C.60 kmph
D.62 kmph
E.None of these

Solution

Answer – C (60 kmph)


Explanation – Number of gaps between 21 telephone posts = 20
Distance traveled in 1 minute = (50 x 20) m = 1000 m = 1 km
Speed = 60 km/hr

11. Two trains starting at the same time from two stations 200
km apart and going in opposite directions cross each other at a
distance of 110 km from one of the stations. What is the ratio of
their speeds?
A.9 : 28
B.11 : 9
C.11 : 8
D.9 : 22
E.None of these

Solution

www.bankingpdf.com
Answer – B (11 : 9)
Explanation – In the same time, they cover 110 km and 90 km
respectively.Ratio of their speeds = 110 : 90 = 11 : 9

12. A and B walk around a circular track. They start at 8 a.m.


from the same point in the opposite directions. A and B walk at
a speed of 2 rounds per hour and 3 rounds per hour
respectively. How many times shall they cross each other
before 9.30 a.m.?
A.15
B.8
C.7
D.10
E.None of these

Solution

Answer – C (7)
Explanation – Relative speed = (2 + 3) = 5 rounds per hourSo, they
cross each other 5 times in an hour and 2 times in half an hourHence,
they cross each other 7 times before 9.30 a.m.

13. The distance between two cities A and B is 330 km. A train
starts from A at 8 a.m. and travels towards B at 60 km/hr.
Another train starts from B at 9 a.m. and travels towards A at
75 km/hr. At what time do they meet?
A.10:30 am
B.10:45 am
C.11 am
D.11:25 am
E.None of these

Solution

www.bankingpdf.com
Answer – C (11 am)
Explanation – Suppose they meet x hrs after 8 a.m.
Then, (Distance moved by first in x hrs) + [Distance moved by second
in (x-1) hrs] = 330
60x + 75(x – 1) = 330
x=3
So, they meet at (8 + 3), i.e. 11 a.m

14. The speed of a car increases by 2 kms after every one hour.
If the distance travelled in the first one hour was 35 kms, what
was the total distance traveled in 12 hours?
A.456 kms
B.482 kms
C.552 kms
D.556 kms
E.None of these

Solution

Answer – C (552 kms)


Explanation – Total distance travelled in 12 hours = (35 + 37 + 39 +
…… upto 12 terms) This is an A.P. with first term,
a = 35, number of terms, n = 12, common difference, d =2.
Required distance =12/2 (2 x 35 + (12 – 1) x 2) = 6(70 + 22) = 552
kms

15. A thief is noticed by a policeman from a distance of 200 m.


The thief starts running and the policeman chases him. The
thief and the policeman run at the rate of 10 km and 11 km per
hour respectively. What is the distance between them after 6
minutes?
A.100 m
B.150 m
C.190 m

www.bankingpdf.com
D.200 m
E.None of these

Solution

Answer- A(100m)
Explanation:
Relative speed of the thief and policeman = (11 – 10) km/hr = 1
km/hr
Distance covered in 6 minutes =[(1/60)*6] km = (1/10)km = 100 m.
Distance between the thief and policeman = (200 – 100) m = 100 m.

90-100 Questions :

Profit and Loss Set 2

1. The cash difference between the selling price of an article at a profit of


8% and 4% is Rs 3 the ratio of the two selling price is
A. 51 : 52
B. 27 : 26
C. 51 : 53
D. 52 : 55

Solution

Answer – B (27:26)
Explanation – Let the C.P of the article be Rs. x
Then required ratio = 108% of x / 104% of x
= 108/104= 27:26

2. A man sells two flats at the rate of Rs. 2 lakhs each. On one he
gains 3% and on the other, he loses 3%. His gain or loss % in
the whole transaction is
A. 9% loss

www.bankingpdf.com
B. 2% loss
C. 3% loss
D. 25% loss

Solution

Answer – A (0.09% loss)


Explanation – Loss% = X^2/100=(3)^2/100 % =0.09%

3. A man sells two flats for Rs. 8000 each. neither loosing nor
gaining in the deal. If he sold one at a gain of 25%, the other
commodity sold at a loss of:
A. 50/3 %
B. 18 /3%
C. 25%
D. 32%

Solution

Answer – A (50/3 %)
Explanation – Total S.P = 16000 Rs. and Total C.P = 16000 Rs.
S.P. of Ist = Rs. 8000. Gain on it = 25%
C.P of Ist = Rs. (100/125 x 8000) = Rs.6400
C.P of 2nd = Rs, (16000 – 6400) = Rs. 9600.
S.P of 2nd = 8000.
∴loss on 2nd commodity = (1600/9600 x 100)% = 50/3%

4. A house and shop were sold for Rs. 1 lakh each. In this
transaction the house sell resulted into 20% loss whereas the
shop sell resulted into 20% of profit. The entire transaction
resulted in:
A. no loss no gain
B. loss of Rs 1/12 lakh
C. loss of Rs 4lakh
D. gain of Rs 6lakh

www.bankingpdf.com
Solution

Answer – B (loss of Rs. 1/12 lakh)


Explanation – Total S.P = Rs. 2 lakh.
C.P of house = Rs. (100/80 x 1)lakh = Rs 5/4 lakh
C.P of shop = Rs. (100/120 x 1)lakh = Rs. 5/6 lakh
Total C.P = Rs. (5/4 + 5/6)lakh = Rs. 25/12 lakh
∴ loss = Rs.(25/12 – 2)lakh = Rs. 1/12 lakh.

5. Reshma purchased 120 chairs at price of Rs. 110 per chair. He


sold 30 chairsat a profit of Rs. 12 per chair and 75 chairs at
profit of Rs. 14 per chairs. The remaining chairs were sold at a
loss of Rs. 7 per chairs. What is the average profit per table?
A. Rs 10.56
B. Rs 10.87
C. Rs 12.123
D. Rs 12.67

Solution

Answer – B (Rs 10.875)


Explanation – Total C.P = Rs. (120 x 110) =Rs. 13200.
Total S.P = Rs.[(30 x 110 + 30 x 12) + (75 x 110 + 75 x 14) + (15 x
110 – 15 x 7)
=Rs..14505
Average profit = Rs. (14505 – 13200) /120
= Rs. 1305/120= 10.875

6. Hemant sold 10 sarees for a total profit of Rs. 460 and 12


sarees for a total profit of Rs. 144. At what profit per saree
should he sell the remaining 20 sarees so that he gets an
average profit of Rs. 18 per sarees?
A. Rs 7.10
B. Rs 7.60

www.bankingpdf.com
C. Rs 7.99
D. Rs 8

Solution

Answer – B (Rs 7.60)


Explanation – Total profit required = Rs.(42 x 18) = Rs.756
Profit on 22 sarees = Rs. (460 + 144) = Rs. 604
Profit on 20 sarees = Rs. (756 – 604) = Rs. 152
Average profit on these sarees = Rs.(152/20) = Rs. 7.60

7. Sanki purchased 20 dozen notebook at Rs. 48 per dozen. He


sold 8 dozen at 10% profit and remaining 12 dozen with 20%
profit. What is his profit percentage in the transaction?
A. 8%
B. 15%
C. 16%
D. 19%

Solution

Answer – C (16%)
Explanation – C.P of 20 dozen = Rs.(48 x 20) = Rs.960
C.P of 8 dozen = Rs. (48 x8) = Rs. 384.
C.P of 12 dozen =Rs. (960 – 384) = Rs. 576
Total S.P = Rs. (110/100 x 384 +120/100 x 576) = Rs.1113.60
∴ Profit % = (153.60/960 x 100)% =16%

8. A man purchased wheat worth Rs. 400. He sold 3/4 at a loss of


10% and the reminder at a gain of 10%. On the whole he gets:
A. loss of 5%
B. gain of 5
C. loss of 19%
D. loss of 6%

www.bankingpdf.com
Solution

Answer – A (loss of 5%)


Explanation – C.P of 3/4th = Rs. (3/4 x 400) = Rs. 300,
C.P of 1/4th = Rs. 100
∴ Total S.P = (90% of Rs. 300 + 110% of Rs. 100) = Rs. 380
Loss = (20/400 x 100)% = 5%

9. Reema buys dress for 10% less than its value and sells it for
10% more than its value. His gain or loss percent is:
A. no profit, no loss
B. 20% profit
C. less than 20% profit
D. more than 20% profit

Solution

Answer – D (more than 20% profit)


Explanation –
Let the article be worth Rs. x.
C.P. = 90% of Rs. x. = Rs.(9/10)x S.P. = 110% of Rs.x = Rs.(11/10)x
Gain = Rs.11/10x- 9/10x=(1/5)x
gain %=(2/5x) x (10/9x) x 100 =44.44% >20%

10. A retailer buys 40 pens at the marked price of 36 pens from


a wholesaler. If he sells these pens giving a discount of 1%,
what is the profit percent?
A. 10%
B. 22%
C. 25%
D. 26%

Solution

www.bankingpdf.com
Answer – A (10%)
Explanation – Let the marked price of each pen be Re. 1.
Then, C.P. of 40 pens = Rs. 36.
S.P. of 40 pens = 99% of Rs. 40 = Rs. 39.60.
∴ Profit% = [3.60/36 x 100]% = 10%.

11. Jimmy bought 25 apples for Rs.10 and sold them at the rate
of 24 apples for Rs.12. What is the percentage of profit made by
him?
A. 25%
B. 70%
C. 60%
D. 100%

Solution

Answer – A (25%)
Explanation – CP of 1 apple=10/25=2/5
SP of 1 apple=12/24=1/2
profit= 1/2-2/5=1/10
%= [(1/10) / (2/5) ] x 100 =25 %

12. 60% loss on cost price is what percent loss on selling price?
A. 28
B. 38 %
C. 40
D. 150%

Solution

Answer – D (150 %)
Explanation – Loss = 60% on CP i.e., 0.6CP
⇒ SP = 0.4CP Loss % on SP = loss/SP x 100
= 0.6CP/0.4CP x 100 = 150%.

www.bankingpdf.com
13. A person, by selling an article at half of the list price incurs a
loss of 20%. Find the profit percentage if he sells at the list
price?
A. 25%
B. 60%
C. 5%
D. 30 %

Solution

Answer – B (60 %)
Explanation – 0.50 MP = 0.8 CP (since 20% loss) So, MP = 160CP ⇒
60% gain.

100 -110 Questions :

Data Interpretation Set 5

Directions for questions : These questions are based on the bar graph given
below.

www.bankingpdf.com
1. For which of the following companies is the percentage
increase in the production of steel the highest from 1999 to
2000?
1:A
2:B
3:D
4:E
5:F

Answer

1:A

2. For which of the following companies is the average production


for all the three years the least?
1:A
2:B
3:C
4:E
5:F

www.bankingpdf.com
Answer

5:F

3. Approximately by what percentage was the total production of


all the six companies in 2000 more than the total production of
all the companies in 1999?
1 : 11%
2 : 12%
3 : 13%
4 : 14%
5 : 16%

Answer

5 : 16%

4. The total production of company A for the given years exceeds


that of the company F by approximately _______.
1 : 1.8%
2 : 2%
3 : 2.2%

www.bankingpdf.com
4 : 2.4%
5 : 3%

Answer

1 : 1.8%

5. The average annual growth in the production of steel of


company D is _______ (lakh tonnes).
1 : 2.5
2 : 3.3
3:5
4 : 6.6
5 : None of these

Answer

3:5

www.bankingpdf.com
110-120 Questions :

Mixtures And Alligations Set 1


1. Sum of Rs.118 was shared among 50 boys and girls,each
girl receive Rs.2.60 and boy receive Rs.1.80.Find the
number of girls.
A) 15
B)25
C)30
D)35

Answer

D)35

2. A Mixture contain 88% of sulphur,another mixture contain


70% of sulphur.In order to get 84% of sulphur,in what
ratio these two must be mixed.
A) 2:7
B)7:2
C)7:4
D)4:5

Answer

www.bankingpdf.com
B)7:2

3. In what ratio water be mixed with juice costing Rs.12 per


liter to obtain a mixture worth of Rs.8 per litre?
A)2:3
B)2:1
C)1:2
D)3:2

Answer

C)1:2

4. In what proportion must sugar at Price 4.10 per kg must


be mixed with weat at Price 4.60 per kg, so that the
mixture be worth Rs 4.30 a Kg ?
A)3:2
B)2:3
C)5:3
D)None of these

Answer

www.bankingpdf.com
A)3:2

5. In what proportion must water be mixed with milk to gain


40 % by selling it at cost price ?
A) 3:5
B)5:3
C)2:5
D)3:2

Answer

C)2:5
Solution:
40 : 100
2:5

6. 400 gm spirit solution has 30 % spirit in it , what is the


ratio of spirit should be added to make it 80 % in the
solution ?
A)2:5
B)4:3
C)5:2
D)2:7

Answer

www.bankingpdf.com
A)2:5

7. One alloy contains silver and copper in the ratio 5:1 and
the other contains in the ratio 7:2 respectively. What
weights of the 2 must be melted together, so as to make a
5 lb mass with 80% silver?
A)5:3
B)3:2
C)2:3
D)2:5

Answer

B)3:2

8. Box A contains wheat worth Rs.30 per kg and box B


contains wheat worth Rs.40 per kg.If both A and B are
mixed in the the ratio 4:7 then the price of mixture per kg
is
A)36.36
B)35.80
C)42.50
D)31.30

Answer

www.bankingpdf.com
A)36.36

9. In what ratio must a person mix three kinds of Oats


costing him Rs 1.20,Rs 1.44 and Rs 1.74 per Kg so that
the mixture may be worth Rs 1.41 per Kg?
A)11:77:7
B) 11:45:7
C)25:45:8
D) 27:23:6

Answer

A)11:77:7

10. Some amount out of Rs. 10000 was lent at 6 % p.a.


and the remaining at 4% p.a. If the total simple interest
from both the fractions in 5 years was Rs. 1500, then the
ratio of the 2 different amount is
A)1:3
B)3:1

www.bankingpdf.com
C)1:4
D)3:2

Answer

A)1:3

120-130 Questions :

Data Interpretation Set 4

The graph below shows the end of the month market values of 4 shares
for the period from January to June. Answer the following questions
based on this graph.

www.bankingpdf.com
1. Which share showed the greatest percentage increase in
market value in any month during the entire period?
(a) A
(b) B
(c) C
(d) D

Answer

Answer (d) D
From the table we clearly seen that highest % increase is for D in Feb.
i.e 25%

www.bankingpdf.com
2. In which month was the greatest absolute change in market
value for any share recorded?
(a) March
(b) April
(c) May
(d) June

Answer

Answer (a) March


The greatest absolute change in the market value is 20
i.e. In the month of March the share of A = 115 – 95 = 20

3. In which month was the greatest percentage increase in market


value for any share recorded?
(a) February
(b) March
(c) April
(d) May

Answer

Answer (a) February


The greatest percentage change in any share was recorded for share D
for the month of February viz. 25%

4. An individual wishes to sell 1 share of C and 1 share of D to buy


1 share of A at the end of a month. At which month-end would
the individual’s loss from this decision, due to share value
changes, be the most?
(a) February
(b) March
(c) April
(d) June

www.bankingpdf.com
Answer

Answer (d) June

Due to share value changes the maximum loss is 10 for the month of
June.
Hence the answer is (d).

5. An individual decides to sell 1 share of C and 1 share of D to


buy 1 share of A at the end of the month. What can be the
individual’s greatest gain from this decision, due to share value
changes?
(a) 5
(b) 10
(c) 15
(d) none

Answer

Answer (a) 5
From the above table again we can see that the individual’s highest
gain is Rs.5.

www.bankingpdf.com
Study the following table carefully and answer the questions given
below.

1. What is the total number of female students studying in the


college R in the given six departments together?
1 : 293
2 : 303
3 : 313
4 : 323
5 : None of these

Answer

2 : 303

2. What is the ratio of total number of male students of the college


Q in the departments CSE and EC to the total number of female
students in the departments EEE and CE of the college U?

www.bankingpdf.com
1 : 55 : 19
2 : 60 : 23
3 : 65 : 27
4 : 70 : 31
5 : None of these

Answer

3 : 65 : 27

3. What is the total number of male students of the colleges P, Q


and R in CSE department and the total number of male students
of the colleges T, U, and V in ME departments?
1 : 444
2 : 464
3 : 484
4 : 504
5 : None of these

Answer

1 : 444

www.bankingpdf.com
4. What approximate percentage of students in CSE of all the
colleges together are males?
1 : 50.49%
2 : 55.89%
3 : 60.69%
4 : 65.29%
5 : 70.09%

Answer

3 : 60.69%

5. In which one of the given colleges are there the highest number
of female students?
1:V
2:U
3:T
4:R
5 : None of these

Answer

www.bankingpdf.com
1:V

130-140 Questions :

MIX QUANTS QUIZ

1. The price of oil is increased by 20%,as a result a family


decrease consumption by 25%.The expenditure of the family on
oil will be
1).90%increase
2).10%incraese
3).90%decrease
4).10%increase.

Answer

www.bankingpdf.com
4).10%increase.
Exp: (120/100)×(75/100)×100=90.
100-90=10%(should be decrease)

2. A rectangular grassy plot 110 m by 65 m has a gravel path 2.5


m wide all round it on the inside. Find the cost of gravelling the
path at 80 paise per sq. Meter
1).Rs.680
2).Rs.860
3).Rs.608
4).None.

Answer

1).Rs.680
Exp: overall area of the grassy plot is 110 x 65 =7150 sq. meters.The
area of the grassy plot
after applying the gravel path is 105 x 60=6300 sq. meters.Area of
the gravel path is 7150 –
6300 = 850 sq.mtrs.Hence x 0.80=Rs.680 .

3. A game of the football with 11 players lasts for exactly


80minutes.There are 4 substitutes that alternate equally.If
each player plays for the same length of the time,What is the
duration?
1)120m
2)20m
3)84m
4)40m

Answer

2)20m
Exp: 80/4=20m.

www.bankingpdf.com
4. 20 workers working 5hours a day can finish the work in 20
days.If the workers went to work for 4hours a day.How many
more workers are required to complete the work in same
20days?
1)7
2)4
3)5
4)6

Answer

3)5
Exp: workers required for 4h=>(5/4)×20=25.
25-20=5.

5. Pencils are bought at 3 for Rs.2 and sold at 2 for Rs.3.Find the
gain percentage
1)100%
2)110%
3)125%
4)115%

Answer

3)125%
Exp: [3(3)-2(2)]/4=5/4
In percentage=> (5/4)×100=125%.

6. In how many ways letter of the world BANKING can be


arranged so that vowels always come together?
1)240
2)120
3)720
4)540

www.bankingpdf.com
Answer

3)720
Exp: [(6!)/(2!)]×(2!)=360×2=720 [(6!)/(2!)]-letters formed, 2!-
Vowels.

7. A bag contain 4 white & 5 red and 6blue color balls,3 balls are
drawn randomly.What is the probability of all the balls are red?
1)1/22
2)3/22
3)2/90
4)2/91

Answer

4)2/91
Exp: 15C3/5C2=(15×14×13)/(3×2×1)=10/455=2/91

8. A boat cover a certain distance downstream in 2hr while it


comeback 3/2hrs.If the speed of the stream be 4kmph.what is
the speed of the boat in still water?
1)21kmph
2)18kmph
3)22kmph
4)12kmph

Answer

1)21kmph
Exp:(x+3) ×2=(x-3)(3/2)=>21.

9. Two pipes A and B can fill a tank in 5 and 6hrs.Pipe C can empty
it in 12hrs.If all the 3 pipes are opened together,then the tank
will be filled in
1)1(13/17)hrs
2)2(8/11)hours.

www.bankingpdf.com
3)3(9/17)hrs
4)4(1/2)hrs

Answer

3)3(9/17)hrs
Exp:Net part filled in 1hr=>(1/5+1/6-1/12)=17/60
Tank filled in=60/17=3(9/17).

10. a/3=b/4=c/7 then find (a+b+c)/c.


1)7
2)2
3)1/2
4)1/7.

Answer

2)2
Exp: (3+4+7)/7=>14/2=2.

140-175 Questions :

Profit and Loss Set 3

1.A trader buys two articles at the same price. He sold one article at
20% profit and sold the other at 10% loss . Find his overall profit/loss
percentage

a)10% loss b)10% profit c)5% loss d)5% profit.

Answer
D

2.If a book is sold at 20% more than its usual price, an extra profit of
Rs.120 would be made on it. find its usual selling price.

www.bankingpdf.com
a)Rs.500 b)Rs.600 c)Rs.750 d)Rs.800

Answer
B

3.The loss made by selling 20 m of a cloth equals the cost price of 5 m


of that cloth. Find the loss percentage

a)33 1/3% b)25% c)20% d)40%

Answer
B

4.The profit made by selling 30 m of a cloth equals the cost price of 6 m


of that cloth. find the profit percentage

a)25% b)20% c)16 2/3% d)30%

Answer
B

5.The profit made by selling 25 m of a cloth equals the selling price of 5


m of that cloth. find the profit percentage?

a)25% b)20% c)16 2/3% d)30%

Answer
A

6.The loss made by selling 16 m of a cloth equals the selling price of 4


m of that cloth. Find the loss percentage

a)15% b)20% c)33 1/3% d)25%

Answer
B

www.bankingpdf.com
7. A shirt was sold at a profit of Rs.50. If a trouser was sold at half the
profit percentage and the cost price of the trouser is thrice that of shirt,
find the profit made on the trouser (in.Rs.)

a)25 b)50 c)75 d)100

Answer
C

8.A trader wants to sell a watch at 25% profit. If its cost price was Rs.x
more, an extra profit of Rs.50 would be made on it. find x(in Rs.)

a)100 b)125 c)150 d)200

Answer
D

9. Amar sold a camera at 20% profit to Bhavan. Bhavan sold it to


Chetan at 30% loss. Chetan bought the camera for Rs.840. find the cost
price of Chetan, had Amar sold the camera to Bhavan at 30% loss and
Bhavan sold it to Chetan at 20% profit (in Rs.)

a)1160 b)1080 c)840 d)720

Answer
C

10. A shopkeeper sold a toothpaste at a profit of 12.5%. His cost price


was Rs.5 less than his selling price. find his cost price (in Rs.)

a)30 b)36 c)40 d)50

Answer
C

11. Rohit sold his bicycle at 4 % profit. If he sold it for Rs.204 less, he
would have made a 30% loss. What was his cost price(in Rs.)?

www.bankingpdf.com
a)630 b)640 c)600 d)560

Answer
C

12. The Selling price of 12 articles equals the cost price of 15 articles.
find the profit/loss percentage?

a)25% profit b)20% profit c)25% loss d)20% loss

Answer
A

13.A trader sold a trouser at 44% profit after announcing two


successive discounts of 10% and 20%. At what percentage did he mark
the trouser above the cost price?

a)60% b)100% c)180% d)175%

Answer
B

14. A merchant claims to sell goods at his cost price. But by using false
weights, he give only 600 gm of goods for every 1 kg that he buys.What
is his profit percentage?

a)40% b)100% c)60% d)66 2/3%

Answer
D

15. Ajay sold a motorbike at a profit of 20% to Balu. Balu sold it to


Dinesh at a loss of 20%. If the profit made by Ajay was Rs.120 less
than the loss made by Balu, find the cost price of Ajay(in Rs.)

a)3000 b)3600 c)3900 d)4500

www.bankingpdf.com
Answer
A

16. A shopkeeper marked a suit at Rs.400. He sold it after allowing a


discount, and still made at a profit of 25%. At what percentage above
its cost price did he marked the suit, if he sold it for Rs.320?

a)50% b)66 2/3% c)62 1/2% d)56 1/4%

Answer
D

17. Arun sold two napkins at same price.He earned 10% profit on one
napkin and suffered 10% loss on the other, what was his overall
profit/loss percentage?

a)No profit no loss b)1% profit c)1% loss d)None of these

Answer
C

18. Ashwin purchased a second hand machine for Rs.1 lakh. Over the
first year after its purchase. He incurred an expense of Rs.20000 in
maintaining it. At the end of the first year he sold it for Rs.80000. Find
his profit/loss percentage. If 25% of the total cost price (i.e.,
purchased price + expenses) is to be deducted on account of
depreciation.

a)12.5% profit b)12.5% loss c)11 1/9% profit d)11 1/9% loss

Answer
D

19. Arjun bought 100 mangoes at the same price . 40 of them were of
lower quality. He sold the remaining 60 mangoes at 60% more than the
cost price. If in total, he earned a 28% profit, What is his profit/loss
percentage, on the 40 mangoes of lower quality?

www.bankingpdf.com
a)20% profit b)15% profit c)20% loss d)15% loss

Answer
C

20. Vijay marked his camera at 50% above his cost price. He sold it
after allowing discount and still made a profit of 20%. What is the
discount percentage he offered on it?

a)20% b)25% c)30% d)35%

Answer
A

Now the following questions will be based on the below data.

Merchants A,B and C sell the same article. They mark the article at Rs.x,
Rs.x and Rs.y respectively. A sells his articles after two successive
discounts of 30% and 10% respectively. B sells his article after two
successive discounts of 20% each.

21. If x=2000, which of the two merchants A and B has a higher selling
price. What is difference is in their selling prices.

a)A, Rs.10 b)B,Rs.10 c)A,Rs.20 d)B,Rs.20

Answer
D

22.If C sells his article after a single discount of 50% at Rs.80 less than
the selling price of B calculated using the value of x in the previous
question, find y (in Rs.)

a)1500 b)1350 c)1800 d)2400

Answer
D

www.bankingpdf.com
23. Ganesha, Harish and Raghu started a business with investments of
Rs.15000, Rs.18000 and Rs.21000. At the end of the one year, they
earned a total profit of Rs.5400. Find the respective shares of Ganesh,
Harish and Raghu in the profit (in Rs.)

a)1500,1800,2100 b)1360,1800,2240 c)1750,2100,2450 d)1890,2100,2310

Answer
A

24. Antony, Ben and Charles enter into a business with investments in
the ratio of 3:4:6. At the end of one year, they shared the profits in the
ratio of 6:4:3. find the ratio of the respective time periods for which
they invested.

a)1:2:4 b)4:2:1 c)4:1:2 d)1:4:2

Answer
B

25. Francis and George started a business with investments of Rs.7000


and Rs.10000. After x months, Francis left. After two more months
David joined the business with an investment of Rs.10000. If the
annual profit is shared among David, Francis and George in the ratio
10:7:24, find the respective time periods of David, Francis and George
for which they stayed that year.

a)5:5:12 b)1:1:3 c)1:1:4 d)1:1:2

Answer
A

26. Prasad and Satish started a business with investments of Rs.2000


and Rs.5300 respectively. At the end of every month, Prasad invests
Rs.300 while Satish withdraws Rs.300. What is the ratio in which they
should share the profits at the end of one year.

www.bankingpdf.com
a)13:14 b)1:1 c)14:13 d)Cannot be determined

Answer
B

27. Anwar marked his radio at 40% above his cost price. He, then sold
it after offering a discount of 40%. If he made a loss of Rs.640, find his
selling price (in Rs.)

a)4320 b)4640 c)3360 d)3720

Answer
C

28.Ram and Shyam started a business with investments of Rs.40000


and Rs.60000. At the end of one year, out of the total Ram got Rs.2000
less profit than Shyam. Find the total profit (in Rs.)

a)12000 b)10000 c)8000 d)16000

Answer
B

29. Anwar and Bhaskar started a business with investments of


Rs.15000 and Rs.24000. At the end of 4 months from the start of the
business, both withdrew one-third of their respective capital. Find the
ratio of their profits in the total profit at the end of one year.

a)1:2 b)3:4 c)4:5 d)5:8

Answer
D

30. Ravi marked his Walkman at 60% above his cost price. If he had
decreased his discount from 20% to 10%. he would have earned Rs.80
more. Find his cost price.

www.bankingpdf.com
a)640 b)600 c)500 d)800

Answer
C

31. A vendor bought 30 dozen bananas at Rs.16 per dozen. He sold 10


dozens at Rs.20 per dozen and 16 dozens at Rs.25 per dozen. The
remaining 4 dozens were rotten and he threw away. Find his profit
percentage.

a)20% b)25% c)30% d)33 1/3%

Answer
B

32.The profit made by David in selling his calculator for Rs.400 would
be the same as the loss he would have made in selling it at a 25% loss.
If he sold his calculator after offering a discount of Rs.100, find the
percentage at which he marked it above the cost price.

a)56.25% b)62.5% c)60% d)50%

Answer
A

33.Francis sold his pen at a profit of Rs.5. He calculated his profit


percentage on his selling price and found it to be 50%. Find his profit
percentage?

a)100% b)150% c)200% d)250%

Answer
A

34. Akbar and Birbal started a business with investments of Rs.30000


and Rs.40000 respectively. At the end of the one year, they gave 10%

www.bankingpdf.com
of the total profit of Rs.7000 to charity. Find the difference in the profit
of shares of both out of the remaining profit (in Rs.)

a)600 b)900 c)500 d)8001

Answer
B

35. Arun and Bharat started a business with investments of Rs.10000


and Rs.15000 respectively. Arun being a working partner gets Rs.100
every month as salary from the profit. At the end of one year the
business makes a profit of Rs.4800. Find the total share of Arun out o
this amount (in Rs.)

a)2880 b)2640 c)3240 d)2760

Answer
B

175-185 Questions :

Directions (Q. 1-10): What should come in place of question mark (?) in
the following number series?

1. 18, 96, 161, 213, 252, ?


1) 264
2) 278
3) 265
4) 291
5) none of these

Answer
(2) 278
Explanation: +78 ,+65 ,+52 ,+39 ,+26.
Table of 13

www.bankingpdf.com
2. 7, 13, 24, 40, 61, ?
1) 87
2) 92
3) 89
4) 93
5) none of these

Answer
(1) 87
Explanation:
7+6=13
13+11=24
24+16=40
40+21=61
61+26=87
[Gap of +5 between 6,11,16,21,26]

3. 8, 12, 39, 55, 180, ?


1) 216
2) 238
3) 240
4) 206
5) 228

Answer
(1) 216

4. 3, 7, 19, 39, 67, ?


1) 107
2) 113
3) 109
4) 103
5) none of these

Answer

www.bankingpdf.com
(4) 103
Explanation: +4 ,+12 ,+20 ,+28 ,+36.
Table of 4 with odd numbers

5. 759, 423, 255, 171, 129, ?


1) 118
2) 107
3) 98
4) 92
5) none of these

Answer
(5) NOT
Explanation: -336 ,-168 ,-84 ,-42 ,-21
Divide these differences by 2.

6. 0.8, 3.8, 12.6, 44.8, 188.2, ?


1) 758.6
2) 868.8
3) 952.00
4) 1012.2
5) 1112.2

Answer
(3) 952
Explanation:
0.8 × 1 + 3 = 3.8
3.8 × 2 + 5 = 12.6
12.6 × 3 + 7 = 44.8
44.8 × 4 + 9 = 188.2
188.2 × 5 + 11 = 952

7. 124, 215, 342, 511, ?, 999


1) 695
2) 625

www.bankingpdf.com
3) 728
4) 806
5) 573

Answer
(3) 728
Explanation: 5^3-1 ,6^3-1 ,7^3-1 ,8^3-1 ,9^3-1

8. 18, 82, 118, 134, 138, ?


1) 132
2) 198
3) 125
4) 138
5) 192

Answer
(4) 138
Explanation: +8^2 ,+6^2 ,+4^2 ,+2^2 ,+0^2.

9. 5, 22, 103, 406, ?, 2422


1) 1516
2) 1298
3) 1136
4) 1602
5) 1213

Answer
(5) 1213
Explanation: *6-8 ,*5-7 ,*4-6 ,*3-5 ,*2-4

10. 516, 256, 126, 61, 28.5, ?


1) 12.25
2) 16.75
3) 14.25

www.bankingpdf.com
4) 20.20
5) 18.75

Answer
(1) 12.25
Explanation: -4/2
516 – 4 = 512/2
256 – 4 = 252/2
.
.
28.5 – 4= 24.5/2 = 12.25
185-195 Questions :

Directions (Q. 1-10): In the following questions two equations


numbered I and II are given. You have to solve both the equations and
give answer if:
1. x > y
2. x > y
3. x < y
4. x < y
5. x = y or the relationship cannot be established.

1. I. x^2 – √(1296)^½ = 58
II. y^7/3 * y^2/3 – 262 = 250

Answer

1) 4

2. I. 20x^2 – 31x + 12 = 0
II. 20y^2 – y – 12 = 0

Answer

www.bankingpdf.com
2)5

3. I. 2x + 3y = 19
II. 7x – 4y = 23

Answer

3) 1

4. I. x^2 + 12 = 7x
II. y^2 + 30 = 11y

Answer

4) 3

5. I. 3x^2 – 47x + 184 = 0


II. 2y^2 – 23y + 66 = 0

Answer

5) 1

6. I. 30x – 49√x + 20 = 0
II. 42y – 5√y – 25 = 0

Answer

6) 2

7. I. x^2 – 14x + 48 = 0
II. y^2 – y – 30 = 0

Answer

8. I. x^2 + 15x + 36 = 0
II. 4y^2 – 13y – 17=0

Answer

www.bankingpdf.com
8) 3

9. I. 10x^2 – 17x -11 = 0


II. 6y^2 + 19y + 15 = 0

Answer

9) 1

10. I. x^2 + 5x -234 = 0


II. y^3 = 2197

Answer

10) 4

195-205 Questions :

11.A shopkeeper sells an item for 28% profit. If the cost price of the
item is Rs.400, what is the marked price of the item if he offered a
discount of 20%?
1 : Rs.462.70
2 : Rs.640
3 : Rs.680
4 : Rs.512
5 : None of these

Answer

2 : Rs.640

www.bankingpdf.com
12.Rama invested a certain amount in debt and equity funds in the ratio
of 3 : 4. At the end of the first year he earned a dividend of 35% on his
investment. After one year he spent the interest on debt but reinvested
the total amount including the dividend in the ratio of 2 : 5 in debt and
equity funds. If the amount reinvested in equity funds was Rs.30,000,
what was the original amount invested in debt funds?
1 : Rs.12000
2 : Rs.15000
3 : Rs.18000
4 : Rs.27000
5 : None of these
Answer

2 : Rs.15000
13.The ages of Gaurav and Goutham will be in the ratio 5 : 7 after ten
years from now and will be in the ratio 13 : 18 after twelve years from
now. Find the ratio of the sum of their ages 10 years hence to the sum
of their ages 12 years hence.
1 : 25 : 27
2 : 13 : 15
3 : 14 : 17
4 : 18 : 23
5 : 30 : 31
Answer

www.bankingpdf.com
5 : 30 : 31
14.The product of 60% of a number and 125% of another number is
what percent of the product of the original numbers?
1 : 90%
2 : 75%
3 : 80%
4 : 100%
5 : None of these
Answer

2 : 75%
15.There are two sets of five numbers each – set P and set Q. P is a set
of consecutive even numbers and Q is a set of consecutive numbers.
The sum of the numbers of P is 230. The second least number in Q is 33
less than twice the least number in P. Find the sum of the numbers in Q.
1 : 260
2 : 250
3 : 240
4 : 270
5 : None of these
Answer

www.bankingpdf.com
1 : 260
16.The shares of A, B and C are in the ratio 4 : 5 : 8. If each of the
shares is increased by 75%, 20% and 25% respectively, what will be
the ratio of their new shares?
1:5:3:5
2:7:6:9
3 : 7 : 6 : 10
4 : 5 : 6 : 10
5 : None of these
Answer

3 : 7 : 6 : 10
17.A man takes 10 hours to cover a distance while traveling upstream
on a boat, whereas while traveling downstream it takes 6 hours. If the
speed of the boat in still water is 6 kmph, what is the speed of the
stream?
1 : 2.5 kmph
2 : 2.2 kmph
3 : 2 kmph
4 : 1.5 kmph
5 : None of these
Answer

www.bankingpdf.com
4 : 1.5 kmph
18.A person sold one-third of the total quantity of goods at a profit of
15%. At what profit percent should the remaining quantity be sold so
that he makes an overall gain of 21%?
1 : 18%
2 : 27%
3 : 33%
4 : 30%
5 : None of these
Answer

5 : None of these
19.Two concentric circles have radii of 23 cm and 9 cm. A rectangle has
an area equal to the annular space between the above two circles. If
the breadth of the rectangle is 22 cm then find its length.
1 : 32 cm
2 : 48 cm
3 : 76 cm
4 : 64 cm
5 : None of these
Answer

www.bankingpdf.com
4 : 64 cm
20.Vijay purchased two kg of sugar and three kg of rice for Rs.145. Ravi
purchased three kg of sugar and two kg of rice for Rs.130. What is the
price of one kg of sugar and one kg of rice?
1 : Rs.45
2 : Rs.50
3 : Rs.55
4 : Rs.60
5 : None of these
Answer
3 : Rs.55

205-215 Questions :

Quadratic Equations Set 1

1.I. 20a^2 + 51a + 28 = 0


II. 15b^2 − 61b + 56 = 0
1 : if a > b
2 : if a < b
3 : if a ≥ b
4 : if a ≤ b
5 : if a = b or the relation between a and b cannot be established.

Answer

www.bankingpdf.com
2 : if a < b
2.I. 2a^2 − 11a + 12 = 0
II. 35b^2 − 18b − 8 = 0
1 : if a > b
2 : if a < b
3 : if a ≥ b
4 : if a ≤ b
5 : if a = b or the relation between a and b cannot be established.
Answer

www.bankingpdf.com
1 : if a > b
3.I. 28a^2 + 25a − 8 = 0
II. 5b^2 − 13b − 6 = 0
1 : if a > b
2 : if a < b
3 : if a ≥ b
4 : if a ≤ b
5 : if a = b or the relation between a and b cannot be established.
Answer

www.bankingpdf.com
5 : if a = b or the relation between a and b cannot be

established.
4.I. 12a^2 − a − 6 = 0
II. 8b^2 − 26b + 15 = 0
1 : if a > b
2 : if a < b
3 : if a ≥ b
4 : if a ≤ b
5 : if a = b or the relation between a and b cannot be established.
Answer

www.bankingpdf.com
4 : if a ≤ b
5.I. 32a^2 + 52a + 15 = 0
II. 8b^2 + 38b + 35 = 0
1 : if a > b
2 : if a < b
3 : if a ≥ b
4 : if a ≤ b
5 : if a = b or the relation between a and b cannot be established.
Answer

www.bankingpdf.com
3 : if a ≥ b
6.I. 6x^2 – 71x + 195 = 0
II. 12y^2 – 97y + 195 = 0
1 : if x ≥ y
2 : if x ≤ y
3 : if x > y
4 : if x < y
5 : if x = y or relationship cannot be established
Answer

www.bankingpdf.com
1 : if x ≥ y
7.I. 56x^2 – 127x + 72 = 0
II. 6y^2 – 17y + 12 = 0
1 : if x ≥ y
2 : if x ≤ y
3 : if x > y
4 : if x < y
5 : if x = y or relationship cannot be established
Answer

4 : if x < y

www.bankingpdf.com
8.I. 3x^2 – 47x + 184 = 0
II. 3y^2 – 38y + 119 = 0
1 : if x ≥ y
2 : if x ≤ y
3 : if x > y
4 : if x < y
5 : if x = y or relationship cannot be established
Answer

3 : if x > y
9.I. 20x^2 – 119x + 176 = 0
II. 18y^2 – 123y + 209 = 0
1 : if x ≥ y
2 : if x ≤ y
3 : if x > y
4 : if x < y
5 : if x = y or relationship cannot be established
Answer

www.bankingpdf.com
5 : if x = y or relationship cannot be

established
10.I. 63x^2 – 275x + 300 = 0
II. 18y^2 – 85y + 100 = 0
1 : if x ≥ y
2 : if x ≤ y
3 : if x > y
4 : if x < y
5 : if x = y or relationship cannot be established
Answer
2 : if x ≤ y

215-225 Questions :

Compound Interest Set 1

1.The Simple interest on a certain sum for 2 years at 10% per annum is
Rs. 90. The corresponding compound interest is:
A.97
B.90

www.bankingpdf.com
C.94.50
D.100
E.None of these

Answer & Explanation


Answer – C (94.50)
Explanation:

Sum = 100 x90 = Rs. 450


2 x 10
C.I. = Rs.450 x (1+10)^ 2 – 450 = Rs.94.50
100
2.Manoj borrowed a certain sum from Anuj at a certain rate of simple
interest for 2 years. He lent this sum to Rakesh at the same rate of
interest compounded annually for the same period. At the end of two
years, he received Rs. 4200 as compound interest but paid Rs. 4000
only as simple interest. Find the rate of interest?
A.10%
B.20%C.25%
D.30%
E.None of these
Answer & Explanation
Answer – A (10%)

Explanation –
Suppose the sum borrowed = Rs X
Rate of interest = R% Time = 2 years
4000=[X x R x 2 ]/100 [simple interesr formula] RX=200,000………….(1)
compound interest= P(1+r/n)^nt
X[1+ (R/100)]^2= X+4200
after solving and using value of RX
we get R=10%
3.If the difference between the simple interest and compound interests
on some principal amount at 20% for 3 years is Rs. 48, then the
principal amount is
A.636
B.650

www.bankingpdf.com
C.375
D.400
E.None of these
Spoiler title
Answer – C (375)
Explanation – For three years
Sum = Difference x (100)^3
r^2(300 + r)
= 48 x (100)^3
20^2 (300 + 20)
= Rs. 375

4.What sum invested for 2 years at 12% compounded annually will


grow to Rs. 4390.40…..
A.4000
B.3875
C.3800
D.3500
E.None of these

Answer & Explanation


Answer – D (3500)
Explanation –
P(1+ R)^T = Amount
100
P (1+12)^2= 4390.40
100

P (112)^2 = 4390.40
100
P = 43904000= 3500.
112 x 112

5.A sum of money is borrowed and paid back in two annual installments
of Rs. 882 each allowing 5% compound interest. The sum borrowed
was:
A.1620
B.1640

www.bankingpdf.com
C.1680
D.1700
E.None of these

Answer & Explanation


Answer – B (1640)
Explanation –
=
882 + 882
(1+ 5 ) ( 1+ 5 )^2
100 100
= 882 x 20 + 882 x 400
21 441
= Rs. 1640

6.Divide Rs. 3903 between A and B, so that A’s Share at the end of 7
years may equal to B’s share at the end of 9 years, compound interest
being at 4 percent.
A.2018 and 1885
B.2028 and 1875
C.2008 and 1895
D.2038 and 1865
E.None of these

Answer & Explanation


Answer – B (2028 and 1875)
Explanation –
We have (A’s present share) (1+ 4 )^ 7= (B’s present share) (1+ 4 )^9
100 100
A’s present share =(1+ 4)^2
B’s present share 100
=( 26 )^2=676
25 625
Dividing Rs. 3903 in the ratio of 676:625
A’s present share = 676 of Rs. 3903
(676+625)
= Rs. 2028
B’s present share = Rs. 3903 – Rs. 2028
=Rs. 1875

www.bankingpdf.com
7.If a sum on compound interest becomes three times in 4 years, then
with the same interest rate, the sum will become 27 times in:
A.8 years
B.12 years
C.24 years
D.36 years
E.None of these

Answer & Explanation


Answer – B (12 years)
Explanation –
P(1 + R )^4 = 3P
100
(1 + R)^4 = 3 …(i)
100
Let P
(1 + R ) ^n = 27P
100
(1 + R )n = 27 = (3)^3 = ( 1 + R )^(4×3)
100 100 [using (i)]

(1 + R )^n ( 1 +R )^12
100 100

n = 12
Required time = 12 years.

225-235 Questions :

Data Interpretation Set 2

Directions for questions: Study the following bar graph carefully and answer
the questions that follow.

www.bankingpdf.com
Number of units of three different products P1, P2 and P3, produced (units in
hundreds) at four different locations (L1, L=, L3 and L4) of a factory the graph.

1. What is the total number of units of P2 produced (in hundreds)


in the four locations together?
1 : 540
2 : 550
3 : 510
4 : 530
5 : None of these

Answer

2 : 550

2. What is the average number of units (in hundreds) of P1


produced per location?
1 : 135
2 : 155
3 : 165

www.bankingpdf.com
4 : 175
5 : None of these

Answer

1 : 135

3. In which location is the maximum number of units produced?


1 : L2
2 : L1
3 : L4 and L2
4 : L3 and L1
5 : None of these

Answer

3 : L4 and L2

4. By what approximate percent is the number of units of P1


produced more than the number of units of P3 produced in the
four locations together?
1 : 10.9%
2 : 12.4%
3 : 16.5%
4 : 18.7%
5 : 5.9%

Answer

5 : 5.9%

5. If the production cost of each of product P2 is Rs.430, then


what is the total production cost of P2?
1 : Rs.2,26,200
2 : Rs.2,36,500
3 : Rs.2,46,800

www.bankingpdf.com
4 : Rs.2,56,400
5 : None of these

Answer

5 :None of these

235 – 245 Questions :

Time and Distance Set 3 – Boats

1. If Nishu can swim downstream at 6kmph and upstream at


2kmph.What is his
speed in still water ?
A.5 km/hr
B.4 km/hr
C.8km/hr
D.7km/hr

Answer

Answer- B
Basic Formula:
If the speed downloadstream is a km/ hr and the speed upstream is b
km/ hr
then Speed in still water is = ½ (a+b) km / hr [memory tool last 2 L
cross and make +] Explanation:
Given : speed downstream a = 6 km ph
Speed upstream b = 2kmph
Speed in still water = ½ (a+b) kmph
= ½ (6+2)
= 8/2 = 4kmph
speed in still water = 4kmph

www.bankingpdf.com
2. Ashok can row upstream at 8kmph and downstream at
12kmph.What is the
speed of the stream ?
A.6km/hr
B.3km/h
C.2 km/hr
D.4km/hr

Answer

Answer -C
Basic Formula:
If the speed downstream is a kmph and the speed upstream is b kmph
then
Speed of the stream = ½ (a-b) kmph
Explanation:
Speed downstream a = 12kmph
Speed upstream b = 8 kmph
Speed of the stream = ½ (a-b) = ½ (12-8)
= 4/2 = 2 kmph
speed of the stream = 2kmph

3. A man rows 750m in 775 seconds against the stream and


returns in 7
1/2 minutes. What is rowing speed in still water ?
A.4.7km/hr
B. 4km/hr
C.3.5km/hr
D.6km/hr

Answer

Answer-A
Basic Formula:
i) Speed in still water = ½ (a+b) kmph where ‘a’ is speed

www.bankingpdf.com
downstream and ‘b’ is speed upstream
ii) a km / hr = a x 5/18 m /s
iii) a m/sec = a x 18/5 km/hr
Explanation:
Speed upstream ‘b’ = 750m / 775 sec = 30/31 m/sec
Speed downstream ‘a’ = 750 m/ (15/2)minutes [ 1min=60 sec] a =
750m/450 sec =5/3 m/sec
speed in still water = ½ (a+b)
= ½ (750/450 + 750/675 ) m /sec
= ½ (750/450 + 750/675 ) x 18/5 km/hr
= ½ (5/3 + 30/31) x 18/5 km/hr
= 4.7 km/hr

4. A man can row 9 (1/3) kmph in still water and finds that it
takes him
thrice as much time to row up than as to row down the same
distance in the
river. What is speed of the current ?
A. 5km/hr
B.3(1/2) km/hr
C.4 (2/3) km/hr
D.8 (3/2)km/hr

Answer

Answer- C
Basic Formula:
Speed of current = ½ (a-b) km/hr
Explanation:
Let man’s rate upstream be x km/hr. Then his rate downstream is 3 x
km/hr
Given:
Speed in still water = 9 (1/3) = 28/3 km/hr
i.e, ½ (a+b) = 28/3 km/hr

www.bankingpdf.com
½ (x+3x) = 28/3
2x = 28/3 x = 28/ 2 x 3 = 14/3 km/hr
rate upstream b = 14/3 km/hr and
rate downstream a = 14/3 x 3 = 14 km/hr
speed of the current = ½ (a-b) = ½ (14 – 14/3)
= ½ (42-14/3) = 28/6 = 4 (2/3) km/hr

5. Sham can row a boat at 10kmph in still water. IF the speed of


the
stream is 6kmph, the time taken to row a distance of 80km
down the stream
is
A.4 hours
B.5hours
C.3 hours
D.2 hours

Answer

Answer- B
Basic Formula:
Speed of stream = ½ (a-b) km/hr
Speed in still water = ½ (a+b) km/hr
Explanation:
Given:
Speed in still water, ½ (a+b) = 10 km/hr
a+b = 20 km/hr…………….(1)
speed of the stream, ½ (a-b) = 6km/hr
a-b = 12 km/hr …………….(2)
(1)+(2 ) we get 2a = 32
a = 16 km/hr
speed downstream =distance traveled / time taken
time taken = 80/16 = 5 hours

www.bankingpdf.com
6. A boat takes 4hours for traveling downstream from point P to
point
Q and coming back to point P upstream. If the velocity of the
stream is 2km
ph and the speed of the boat in still water is 4kmph, what is the
distance
between P and Q?
A.9 km
B.7 km
C.5 km
D.6km

Answer

Answer- D
Basic Formula:
Speed of stream = ½ (a-b) km/hr
Speed of still water = ½ (a+b) km/hr
Explanation:
Time taken by boat to travel upstream and downstream = 4 hours
Velocity of the stream, ½ (a-b) = 2km/hr
a-b = 4km/hr ……………….( 1)
velocity of the boat in still water = ½ (a+b) = 4km/hr
a+b = 8 km/hr ………………(2)
1 +2 we get a = 6 km/hr ,b = 2km/hr
let the distance between A and B be x km
x/2+x/6=4
3x + x / 6 = 4 4x = 24 so,x = 6
distance between P and Q = 6km

7. Speed of a boat in standing water is 9kmph and the speed of


the
stream is 1.5kmph. A man rows to a place at a distance of 10.5
km and

www.bankingpdf.com
comes back to the starting point. Find the total time taken by
him.
A.24 hours
B.16 hours
C.20 hours
D.15 hours

Answer

Answer- A
Basic Formula:
i. speed = distance traveled / time taken
ii. speed of the stream = ½ (a-b) km/hr
iii. speed in still water = ½ (a+b) km/hr
Explanation:
Speed in still water= ½ (a+b) = 9km ph
= a+b = 18 …………….1
speed of the stream = ½ (a-b) = 1.5 kmph
= a-b = 3 kmph…………2
solving 1 and 2 gives a = 10.5km/hr ; b=7.5 kmphr
Total time taken by him = 105/10.5 + 105/7.5 = 24 hours

8. A man rows to a place 48km distant and back in 14 hours. He


finds
that he can row 4km with the stream in the same time as 3km
against the
stream. Find the rate of the stream.
A.2 km/hr
B.1 km/hr
C.3 km/hr
D.3.5km/hr

Answer

www.bankingpdf.com
Answer- B
Basic Formula:
Speed of the stream = ½ (a-b) km / hr
Speed = distance traveled / time taken
Explanation:
Suppose he moves 4km downstream in x hours
Then, downstream a= 4 / x km/hr
Speed upstream b = 3/ x km/hr
48 / (4 /x) + 48 / (3/x) = 14
12x + 16x = 14
x = 1/2
a=8 km/hr ,b = 6 km/hr
rate of stream = ½ (8 – 6 )
= 1 km/hr

9. There is road besides a river. Two friends started from a place


P, moved to a shopping mall
situated at another place Q and then returned to P again. One
of them moves on a cycle at
a speed of 12 km/hr, while the other sails on a boat at a speed
of 10 km/hr. If the river
flows at the speed of 4 km/hr, which of the two friends will
return to place P?
A. Both
B. Boater
C. Cyclist
D. None of these

Answer

Answer-C
Explanation:
The cyclist moves both ways at a speed of 12khr so average speed fo
the

www.bankingpdf.com
cyclist – 12 km/hr
boat sailor moves downstream at 10+4 = 14km/hr and upstream 10-
4 = 6km/hr
Average speed of the boat sailor = 2 x 14 x 6 / 14 +6 = 42/ 5 =
8.4km/hr
The average speed of cyclist is greater .so,cyclist comes first and
return to
place P.

10. A this usual rowing rate, Mohit can travel 12 miles


downstream in a certain river in 6
hours less than it takes him to travel the same distance
upstream. But if he could double
his usual rowing rate for his 24 miles round trip, the
downstream 12 miles would then
take only one hour less than the upstream 12 miles. What is the
speed of the current in
miles per hour?
A.2.5m/hr
B.4 m/hr
C.8/3 m/hr
D. 5/3m/hr

Answer

Answer-C
Basic Formula:
Speed of the stream = ½ (a-b) km/hr
Explanation:
Let the speed in still water be x m/hr
Speed of stream be y m/hr
Then, speed upstream = x-y m/hr and
Speed downstream = x+y m/hr
12/x-y – 12 / x+y = 6 so,6 (x^2 – y^2) = 24 y

www.bankingpdf.com
x^2 – y^2 = 4y
x^2 = y^2 + 4y…………..1
also
12/ 2x-y – 12/2x +y = 1 4x^2 – y^2 = 24y
x^2 = [24y + y^2] / 4 ……………….2
16y + 4y^2 = 24y + y2 [put X^2 value from 1] 3y^2 = 8 y so, y =
8/3
speed of the current = 8/3 m/hr = 2 (2/3) m/hr

245-255 Questions :

Time and Distance Set 2 – Trains

1. Two trains are moving in opposite directions at 60 km/hr and


90 km/hr. Their lengths are 1.10 km and 0.9 km respectively.
The time taken by the slower train to cross the faster train in
seconds is:
A.58 sec
B.50 sec
C.48 sec
D.56 sec
E.None of these

Answer

Answer – C (48 sec)


Explanation – Relative speed = all lengths/time
[60+90] = [1.10 +0.9 ]/time [ PLUS when opposite direction]
time=2/150 = 1/75 h
1 hour______3600 sec
1/75 hr______?
?= 3600/75=48 sec

www.bankingpdf.com
2. A 270 m long train running at the speed of 120 kmph crosses
another train running in opposite direction at the speed of 80
kmph in 9 seconds. What is the length of the other train?
A.180
B.230
C.245
D.235
E.None of these

Answer

Answer – B (230)
Explanation – Relative speed = total lenghths /time
[120 +80] x5/18 ={ 270 + other train length (say L) } / 9 sec (x5/18
to convert km/hr to m/sec )
200 x (5/18) x9 = 270 + L
500 = 270 +L
L=230 m
Note: always do cutting ,avoid solving exact

3. Two trains of equal length are running on parallel lines in the


same direction at 46 km/hr and 36 km/hr. The faster train
passes the slower train in 36 seconds. The length of each train
is:
A.40 m
B.55 m
C.65 m
D.50 m
E.None of these

Answer

Answer- D (50)
Explanation – Relative speed= total lengths/time
(46-36) x 5/18 = [L + L ] / 36

www.bankingpdf.com
10 x (5/18) x36 x (1/2)= L
L=50 m

4. A train 125 m long passes a man, running at 5 kmph in the


same direction in which the train is going, in 10 seconds. The
speed of the train is:
A.35 km/hr
B.50 km/hr
C.48 km/hr
D.55 km/hr
E.None of these

Answer

Answer – B (50 km/hr)


Explanation – Let speed be S
Relative speed = total lengths/time [ in this case man length is neglible
compare to train so neglected] (S – 5) x 5/18 = 125 /10
S- 5 = (125/10)x(18/5)
S-5 = 45
S=50 km/hr

5. A train travelling at a speed of 75 mph enters a tunnel 7/2


miles long. The train is 1/4 mile long. How long does it take for
the train to pass through the tunnel from the moment the front
enters to the moment the rear emerges?
A.1 min
B.3 min
C.5 min
D.6 min
E.None of these

Answer

www.bankingpdf.com
Answer – B (3 min)
Explanation – Actually train is covering length of tunnel + its own
length here
so total distance = 7/2 + 1/4= 15/4 miles
time= distance /speed
time= (15/4) / 75 = 1/20 hour
(1/20)x60 = 3 min

6. A 300 metre long train crosses a platform in 39 seconds while it


crosses a signal pole in 18 seconds. What is the length of the
platform?
A.310 m
B.350 m
C.600 m
D.490 m
E.None of these

Answer

Answer – B (350 m)
Explanation – when it cross a pole actually it is crossing itself
so, S = 300/18
let length of platform be “p”
relative speed = total lengths/time
speed of train = [300 + p ]/39
(300/18)x39=300 +p
650 -300=p
p=350 m
Note : in this type of question one thing is pole,man and other is of
considerable length like platform ,bridge.
when train crosses pole,man it crosses itself

7. Two trains running in opposite directions cross a man standing


on the platform in 27 seconds and 17 seconds respectively and

www.bankingpdf.com
they cross each other in 23 seconds. The ratio of their speeds
is:
A.1 : 3
B.3 : 4
C.3 : 2
D.Data inadequate
E.None of these

Answer

Answer – C (3 : 2)
Explanation – Let the speeds of the two trains be x m/sec and y
m/sec respectively.
Then, length of the first train = 27x metres,
and length of the second train = 17y metres.
relative speed = total lengths/time
x + y = [27x + 17 y ]/23
23x + 23y = 27x + 17 y
27x-23x= 23 y-17y
4x=6y
x/y=6/4=3/2= 3:2

8. The distance between two cities A and B is 330 km. A train


starts from A at 8 a.m. and travels towards B at 60 km/hr.
Another train starts from B at 9 a.m. and travels towards A at
75 km/hr. At what time do they meet?
A.10:30 am
B.10:45 am
C.11 am
D.11:25 am
E.None of these

Answer

www.bankingpdf.com
Answer – C (11 am)
Explanation – Suppose they meet x hrs after 8 a.m.
Then, (Distance moved by first in x hrs) + [Distance moved by second
in (x-1) hrs] = 330
60x + 75(x – 1) = 330
x=3
So, they meet at (8 + 3), i.e. 11 a.m

9. Two, trains, one from Howrah to Patna and the other from
Patna to Howrah, start simultaneously. After they meet, the
trains reach their destinations after 9 hours and 16 hours
respectively. The ratio of their speeds is:
A. 1: 2
B. 4 : 3
C. 7 : 8
D. 3 : 4
E. none of these

Answer

Answer- B (4:3)
Explanation:
Let us name the trains as A and B. Then,
trick formula
(A’s speed) : (B’s speed) = square root of b : square-root of a
=square-root of 16 : square-root of 9 = 4 : 3.

10. A train overtakes two persons walking along a railway track.


The first one walks at 4.5 km/hr. The other one walks at 5.4
km/hr. The train needs 8.4 and 8.5 seconds respectively to
overtake them. What is the speed of the train if both the
persons are walking in the same direction as the train?
A.59 km/hr
B.65 km/hr

www.bankingpdf.com
C.70 km/hr
D.81 km/hr
E. None of these

Answer

Answer-D (81 km/hr)


Explanation-
4.5 km/hr =(4.5 x 5/18)m/sec =5m/4sec = 1.25 m/sec, and
5.4 km/hr =(5.4 x 5/18)m/sec =3m/2sec = 1.5 m/sec.
Let the speed of the train be x m/sec.
Then, (x – 1.25) x 8.4 = (x – 1.5) x 8.5
=> 8.4x – 10.5 = 8.5x – 12.75
=> 0.1x = 2.25
=> x = 22.5
Therefore Speed of the train =(22.5 x18 /5)km/hr = 81 km/hr.

255 – 265 Questions :

Number Series Set 1

1. 3,14,58,234,938,?
1 : 4152
2 : 4110
3 : 3754
4 : 4154
5 : 4624

Answer

Answer- 3 (3754)
Explanation- x4+2

www.bankingpdf.com
2. 3,25,255,2545,25455, ?
1 : 254550
2 : 254555
3 : 254545
4 : 254555
5 : 254556

Answer

Answer-3 (254545)
Explanation-
25=(3×10)-5
255=(25×10)+5
2545=(255×10)-5
25455=(2545×10)+5
?=(2545×10)-5 = 254545

3. 4725,1050,300,120,80,?
1 : 40
2 : 160
3 : 120
4 : 110
5 : 60

Answer

Answer-2 (160)
Explanation- 4725 / 4.5=1050
1050/3.5=300
300/2.5=120
120/1.5=80
80/0.5=160

4. 24,48,144,576,2880,?
1 : 17280

www.bankingpdf.com
2 : 16640
3 : 14400
4 : 20160
5 : 14240

Answer

Answer- 1 (17280)
Explanation- 24 x 2=48
48×3=144
144×4=576
576×5=2880

5. 363,795,1227,1659,2091, ?
1 : 2423
2 : 2613
3 : 2813
4 : 2513
5 : 2523

Answer

Answer- 5 (2523)
Explanation- +432 in every step

6. 5, 5, 15, 75, 525, 4725, _____.


1 : 51795
2 : 50135
3 : 51025
4 : 50175
5 : 51975

Answer

Answer- 5(51975)
Explanation- multiply by consecutive odd numbers

www.bankingpdf.com
5×1=5
5×3=15
15×5=75
75×9=4725
4725×11=51975

7. 5, 13, 29, 61, 125, 253, _____


1 : 512
2 : 521
3 : 509
4 : 552
5 : None of these

Answer

Answer-3(509)
Explanation-
5 x2 +3 =13
13×2 +3=29
29×2+3=61 and so on..

8. 16, 32, 35, 140, 145, 870, 877, _____


1 : 7016
2 : 885
3 : 5020
4 : 6018
5 : None of these

Answer

Answer- 1(7016)
Explanation-
16×2 =32
32+3=35
35×4=140

www.bankingpdf.com
140+5=145
145×6=870
870+7=877
877×8=7016

9. 5, 10, 35, 160, 785, _____.


1 : 2010
2 : 3910
3 : 2525
4 : 2860
5 : None of these

Answer

Answer- 2 (3910)
Explanation-
add 5^1,5^2,5^3,5^4,5^5 and so on in each step.

10. 196, 200, 209, 234, 283, 404, 573, _____


1 : 872
2 : 840
3 : 884
4 : 878
5 : 862

Answer

Answer-5 (862)
Explanation- add square of prime no. in each step
4,9,25,49,121,so on

www.bankingpdf.com
265-275 Questions ;

LCM & HCF Set 1

1. Six bells commence tolling together and toll at intervals of 2, 4,


6, 8, 10 and 12 seconds respectively. In 30 minutes, how many
times do they toll together?
A.8
B.11
C.13
D.16
E.None of these

Answer

Answer – D (16)
Explanation – L.C.M. of 2, 4, 6, 8, 10, 12 is 120.So, the bells will toll
together after every 120 seconds, i.e, 2 minutes.In 30 minutes, they
will toll together 30/2 + 1 = 16

2. The least multiple of 7, which leaves a remainder of 4, when


divided by 6, 9, 15 and 18 is:
A.68
B.98
C.180
D.364
E.None of these

Answer

Answer – D (364)
Explanation – L.C.M. of 6, 9, 15 and 18 is 90.
Let required number be 90k + 4, which is multiple of 7.

www.bankingpdf.com
Least value of k for which (90k + 4) is divisible by 7 is k = 4.
Required number = (90 x 4) + 4 = 364.

3. The least number, which when divided by 48, 60, 72, 108 and
140 leaves 38, 50, 62, 98 and 130 as remainders respectively,
is:
A.11115
B.15110
C.15130
D.15310
E.None of these

Answer

Answer – B (15110)
Explanation – Here (48 – 38) = 10, (60 – 50) = 10, (72 – 62) = 10,
(108 – 98) = 10 & (140 – 130) = 10.
Required number = (L.C.M. of 48, 60, 72, 108, 140) – 10
= 15120 – 10 = 15110

4. The H.C.F. of two numbers is 11 and their L.C.M. is 7700. If one


of the numbers is 275, then the other is:
A.269
B.275
C.308
D.310
E.None of these

Answer

Answer – C (308)
Explanation –
Other number =[11 x 7700]/275 = 308

www.bankingpdf.com
5. A, B and C start at the same time in the same direction to run
around a circular stadium. A completes a round in 252 seconds,
B in 308 seconds and C in 198 seconds, all starting at the same
point. After what time will they meet again at the starting
point?
A.15 minutes 15 seconds
B.42 minutes 30 seconds
C.42 minutes
D.46 minutes 12 seconds
E.None of these

Answer

Answer – D (46 minutes 12 seconds)


Explanation – L.C.M. of 252, 308 and 198 = 2772.So, A, B and C will
again meet at the starting point in 2772 see i.e., 46 min. 12 sec

6. The L.C.M. of two numbers is 48. The numbers are in the ratio 2
: 3. Then sum of the number is:
A.30
B.22
C.40
D.60
E.None of these

Answer

Answer – C (40)
Explanation – Let the numbers be 2x and 3x.
Then, their L.C.M. = 6x.
So, 6x = 48 or x = 8.
The numbers are 16 and 24.
Hence, required sum = (16 + 24) = 40.

www.bankingpdf.com
7. The least number, which when divided by 12, 15, 20 and 54
leaves in each case a remainder of 8 is:
A.534
B.486
C.544
D.548
E.None of these

Answer

Answer – D (548)
Explanation – Required number = (L.C.M. of 12, 15, 20, 54) + 8
= 540 + 8
= 548.

8. The product of two numbers is 4107. If the H.C.F. of these


numbers is 37, then the greater number is:
A.124
B.100
C.111
D.175
E.None of these

Answer

Answer – C (111)
Explanation – Let the numbers be 37a and 37b.
Then, 37a x 37b = 4107
ab = 3.
Now, co-primes with product 3 are (1, 3).
So, the required numbers are (37 x 1, 37 x 3) i.e., (37, 111).
Greater number = 111.

9. The product of two numbers is 2028 and their H.C.F. is 13. The
number of such pairs is:

www.bankingpdf.com
A.1
B.2
C.3
D.5
E.None of these

Answer

Answer – B (2)
Explanation – Let the numbers 13a and 13b.
Then, 13a x 13b = 2028
ab = 12.
Now, the co-primes with product 12 are (1, 12) and (3, 4).

10. The least number which should be added to 2497 so that the
sum is exactly divisible by 5, 6, 4 and 3 is:
A.10
B.14
C.23
D.30
E.None of these

Answer

Answer – C (23)
Explanation – L.C.M. of 5, 6, 4 and 3 = 60. On dividing 2497 by 60,
the remainder is 37. Number to be added = (60 – 37) = 23

275-285 Questions :

Number System Set 1


1. The digits of a two-digit number are in the ratio of 2 : 3 and the
number obtained by interchanging the digits is bigger than the
original number by 27.What is the original number?

www.bankingpdf.com
1. 63
2. 48
3. 96
4. 69
5. 66

Answer

Answer- 4 (69)
Explanation:- lets digit at tens and ones place be 2x and 3x
So,number is 2x*10 + 3x
After interchanging number will be 3x*10 +2x
so, [3x*10 +2x] – [2x*10 + 3x]27
we get,x=3
so number =69

2. What least number would be subtracted from 427398 so that


the remaining number is divisible by 15?
1. 13
2. 3
3. 16
4. 11
5. 14

Answer

Answer- 2 ( 3)
Explanation:- On dividing 427398 by 15 we get the remainder 3, so 3
should be subtracted

3. In a division sum, the divisor is 10 times the quotient and 5


times the remainder. If the remainder is 46, the dividend is:
1.4236
2.4306

www.bankingpdf.com
3.4336
4.5336

Answer

Answer – 4 (5336)
Explanation – Divisor = (5 x 46) = 230.
Also, 10 x Q = 230
Q = 23. And, R = 46.
Dividend = (230 x 23 + 46) = 5336

4. A number gets reduced to its one-third when 48 is subtracted


from it. What is two-third of that number?
1. 22
2. 76
3. 36
4. 48
5. None of these

Answer

Answer:- 4 (48)
Expalanation – let number be X
X-48 = X/3
X-X/3 =48
2/3X= 48
so 2/3th of number=48

5. One-fifth of a number is equal to 5/8th of another number. If


35 is added to the first number, it becomes four times of the
second number. Find the second number.
1. 39
2. 70
3. 40

www.bankingpdf.com
4. 25
5. None of these

Answer

Answer- 3(40)
Explanation:- (1/5)x =(5/8)y………..1
if 35 is added to first no. , x + 35= 4y
so,x= 4y -35
put in eq 1… (1/5)( 4y-35) = (5/8)y
y=40 =second number

6. One number is 3 more than another and the sum of their square
is 269.Find the number.
1.10
2.12
3.15
4.20

Answer

Answer-1 (10)
Explanation:-
x is one number
x+3 the other
x^2 +x^2 +6x+9 ; (that is (x+3)(x+3) =269
2x^2+6x -260=0
x^2+3x-130=0 , dividing by 2.
(x+13)(x-10)=0
one number is 10, the other is 13
squared 10=100
square 13=169
sum is 269

www.bankingpdf.com
7. Find the number which when multiplied by 13 is increased by
180:
1. 20
2. 15
3. 12
4. 5
5. 14

Answer

Answer- 2 (15)
Explanation:- let no. be X
X *13= X+180
!2x=180
x=15

8. The sum of twice the square of a number and 7 times the


number equals 15. What is the number?
1.1/2
2.3/4
3.3/2
4.5/3

Answer

Answer-3 (3/2)
Explanation:-2x^2 + 7x = 15
2x^2 + 7x – 15 = 0
2x^2 + 10x-3x – 15 = 0
2x(x+5)-3(x+5) = 0
(x+5)(2x-3) = 0
x = -5 or x = 3/2

9. A two digit number is such that the sum of the digits is


11.When the number with the same digits is reversed is

www.bankingpdf.com
subtracted from this number, the difference is 9.What is the
number?
1. 23
2. 24
3. 65
4. 14
5. 32

Answer

Answer- 3(65)
Explanation:-
let the number be written as xy
x-ten’s place
y units place
x+y=11…………………1
(10x+y)-(10y+x)=9
10x+y-10y-x=9
9x-9y=9
x-y=1………………….2
Add (1) & (2)
2x=12
x=6
y=5
the number is 65

10. Three consecutive numbers such that twice the first, 3 times
the second and 4 times the third together make 182. The
numbers in question are
1. 18, 22 and 23
2. 18, 19 and 20
3. 19, 20 and 21
4. 20, 21 and 22
5. 21, 22 and 23

www.bankingpdf.com
Answer

Answer- 3( 19,20,21)
Explanation:- let number be x,x+1,x+2
2x + 3(x+1)+4(x+2)=180
x=19
so numbers are 19,20,21

285-295 Questions :

Partnership Set 1

1. A, B and C enter into a partnership and their shares are in ratio


1/2 : 1/3 : 1/4, after 2 months, A withdraws half of his capital
and after 12 months, a profit of Rs 1050 is divided among them.
What is B’s share ?
A.Rs.420
B.Rs. 300
C.Rs.400
D.Rs.350

Answer

Answer- C(Rs.400)
Explanation : Ratio of investments =1/2 : 1/3 : 1/4 , LCM of 2, 3, 4
= 12 ,multiply ratios with 12 we get 6 : 4 : 3 , so,initial investment be
6x, 2x and 3x so, we can write: A : B :C
=(6x X 2 + 3x X 10 ) : (4x X 12) :(3x X 12) = 7:8:6
B’s share= (8/21) x 1050 = 400

2. A started a business with Rs. 21,000 and is joined afterwars by


B with Rs. 36,000. After how many months did B join if the
profits at the end of the year are divided equally?
A.8 months

www.bankingpdf.com
B.2 months
C.5 months
D.7 months

Answer

Answer – C (5 months)
Explanation – let B joined after x months
Then, 21000 x 12 = 36000 x (12 – x)36 = 180x = 5Hence, B joined
after 5 months

3. Anju and Bimal are partners in a business. Anju contributes 1 /


4 of the capital for 15 months and Bimal received 2 / 3 of the
profit, for how long Bimal’s money was used ?
A.5 months
B.3months
C.8 months
D.10 months

Answer

Answer- D (10 months)


Explanation- Let total profit is x
Then Bimal’s share in profit= (2/3)x
so anju’s share in profit= x- (2/3)x =x/3
so,we get ratios of profit of Anju :Bimal :: 1:2
Let total capital invested be Rs P and Anju’s money was used for 15
months while Bimal’s money was used for b months .
then, (1/4)P X 15 divide by (3/4)P x b = 1/2 [ Capital x time =profit]
so,we get b=10
Bimal’s money was used for 10 months

4. In a business P and R invested amounts in the ratio 2 : 1,


whereas the ratio between amounts invested by P and Q was 3
: 2 . If Rs. 2,236 was their profit, how much amount did Q

www.bankingpdf.com
receive ?
A.Rs.650
B.Rs.688
c.Rs.588
D.Rs.490

Answer

Answer- B(Rs.688 )
Solution : P : Q = 3 : 2 , P : R = 2 : 1 [given] Q : P = 2 : 3 [reverse],
Q : P = 4 : 6 [multiply by 2] Now, P : R = 2 : 1
P : R = 6 : 3 [multiply by 3] P : Q =6 : 4 [after x3] , So Q : P : R = 4 :
6:3
or, P : Q : R = 6 : 4 : 3
Q’s share= 4/13 x 2236=Rs.688

5. A, B and C rent a land. A puts 10 tractors for 7 months, B puts


12 tractors for 5 months and C puts 15 tractors for 3 months . If
the rent of the land is Rs. 175000, how much must C pay as his
share of rent?
A.45000
B.50000
C.57000
D.64000

Answer

Answer – A (45000)
Explanation – A : B : C = 10 x 7 : 12 x 5 : 15 x 3
= 70 : 60 : 45
= 14 : 12 : 9.
∴ C’s rent = Rs. [175000 x 9/35] = Rs. 45000.

6. P and Q started a partnership business investing some amount


in the ratio of 3:5. R joined them after six months with an

www.bankingpdf.com
amount equal to that of Q. In what proportion should the profit
at the end of one year be distributed among P, Qand R?
A.5 : 8 : 10
B.6 : 10 : 5
C.6 : 4 : 10
D.10 : 6 : 3

Answer

Answer – B (6 : 10 : 5)
Explanation – Let the initial investments of P and Q be 3a amd 5a.
P : Q : R = (3a x 12) : (5a x 12) : (5a x 6) = 36 : 60 : 30 = 6 : 10 : 5.

7. A and B established a firm together. A’s investment was thrice


that of B’s. A also kept the investment for twice as much time
as B. If B got a profit of 4000, what was the total profit?
A.30,000
B.28,000
C.40,000
D.45,000

Answer

Answer- B(28,000)
Explanation:- let B’s investment= X then A’s investment= 3X
lets tim for B = t then,A’s time= 2t
A:B
3X x 3t : X x t
6:1
B’s share= 1/7 x total = 4000
total=28,000

8. A and B entered into a partnership investing Rs. 16000 and Rs.


12000 respectively. After 3 months, A withdrew Rs. 5000 while
B invested Rs. 5000 more. After 3 more months C joins the

www.bankingpdf.com
business with a capital of Rs. 21000. The share of B exceeds
that of C, out of a total profit of Rs. 26400 after one year by
A.Rs. 2400
B.Rs. 3600
C.Rs. 3000
D.Rs. 4800

Answer

Answer: B (Rs. 3600)


Explanation: A:B: C = 16000 x3 + 11000×9:12000 x 3 + 17000 x
9:21000 x 6 = 147:189:126 = 7:9:6
Difference of B and C’s shares = Rs. [26400 x (9/22) — 26400 x
(6/22)) = Rs. 3600.

9. Ashok being the sleeping partner receives 1/10th of profit and


the remaining is divided between pramod and prakash in the
ratio of 1:2..If the difference between the profit shares of
Ashok and Prakash is Rs.2000.What is pramod’s share in Rs.?
A.Rs.1800
B.Rs.2200
C.Rs.1200
D.Rs.1500

Answer

Answer:- C (1200)
Explanation:-
let total profit =x
ashok’s share in profit is (1/10)x
remaining profit = x= (1/10)x= (9/10)x
pramod’s share= 1/3 x (9/10)x = (3/10 )x
Prakash’s share= 2/3 x (9/10)x = (6/10)x
ashok – prakash= (6/10)x -(1/10)x = (5/10)x

www.bankingpdf.com
5/10 x = 2,000 so, x= 4000
pramod’s share= (3/10) x 4000=1200

10. P and Q invest in a business in the ratio 3 : 2. If 5% of the


total profit goes to charity and P’s share is Rs. 855, the total
profit is:
A.Rs. 1435
B.Rs. 1500
C.Rs. 1538
D.Rs. 1580

Answer

Answer- B (Rs.1500)
Explanation:
Let the total profit be Rs. 100.
After paying to charity, A’s share = Rs. 95 x 3/5 = Rs. 57.
If A’s share is Rs. 57, total profit = Rs. 100.
If A’s share Rs. 855, total profit =(100/57) x 855 = 1500.

300-310 Questions :

Number Series&Miscellaneous

Directions (Q. 1-5): What will come in place of the question mark (?) in the following
questions?
1). 2 3 10 ? 172
a) 45
b) 39
c) 36
d) 42
e) None of these

www.bankingpdf.com
2). 37 84 260 ? 5234
a) 306
b) 512
c) 1146
d) 1046
e) None of these

3). 5 6 ? 45 184
a) 15
b) 12
c) 16
d) 9
e) None of these

4). 7 15 ? 63 127
a) 32
b) 29
c) 33
d) 31
e) None of these

5). 8 4 6 ? 52.5
a) 9
b) 12.5
c) 15
d) 16
e) None of these

www.bankingpdf.com
6). A two digit number exceeds forty percent of another two digit number by 16. If the
sum of these two digit numbers is 72, which of the following is the difference between
these two digit numbers ?
a) 10
b) 6
c) 8
d) 14
e) None of these

7). A task was assigned to P, Q and R to be completed by not more than seven days. P
alone can complete the task in ten days. Q alone can complete the task in twelve days
and R alone can complete the task in fifteen days. All three of them worked for two days
and P and Q together worked for another two days. What portion of the task will remain
after four days?
a) 3/10
b) 1/15
c) 1/9
d) 5/16
e) None of these

8). The price of two dozens of oranges is Rs.120/-more than the price of one kg. of
mango. The price of one dozen of apple is Rs.80/- more than the price of one kg of
mango. Which of the following represents the ratios of the price of one orange and one
apple respectively?
a) 4 : 5
b) 3 : 2
c) 7 : 9
d) Cannot be determined

www.bankingpdf.com
e) None of these

9). Three-fourth of two-third of one-sixth of a number is 15.What is 30 percent of that


number?
a) 54
b) 48
c) 72
d) 64
e) None of these

10). The average age of the students in a class is 14. The average age of half of the
students in that class and the class teacher is 16. The average age of the remaining half
of the students is 15. What is the number of students in that class?
a) 20
b) 70
c) 80
d) Cannot be determined
e) None of these

Answer:
1)b 2)d 3)e 4)d 5)c 6)c 7)e 8)d 9)a 10)d

Solution:
1). The series is 2 × 1+ 1^2, 3 ×2 + 2^2, 10 ×3 + 3^2, 39×4+4^2, 172×5+5^2
Answer: b)

2). The series is 37×2+10, 84×3+8, 260×4+6, 1046×5+4, 5234×6+2

www.bankingpdf.com
Answer: d)

3). The series is 5×1+1, 6×2+2, 14×3+3, 45×4+4, 184×5+5


Answer: e)

4). The series is 7×2+1, 15×2+1, 31×2+1, 63×2+1, 127×2+1


Answer: d)

5). The series is 8×0.5, 4×1.5, 6×2.5, 15×3.5, 52.5× 4.5


Answer: c)

6). Let first two digit no. and second two digit no. be x and y respectively.
x = y × 40/100 +16
x - 2y/5 = 16....(i)
x + y = 72...(ii)
From equation (i) and (ii)
x = 32, y = 40
Difference = 8
Answer: c)

7). 1day's work of (P + Q+ R)


= 1/10 + 1/12 +1/15 = 15/60
2 day's work of (P + Q+ R)
= 15/60 × 2 = 1/2
1 day's work of (P+Q) = 1/10 +1/12 = 11/60
2 day's work of (P+Q) = 11/60 × 2 = 11/30
Rem. work of 4 days = 1- 1/2 + 11/30 = 2/15

www.bankingpdf.com
Answer: e)

8). Because no. of mangoes is unknown. So ratio cannot be calculated.


Answer: d)

9). Suppose no. is x


3x/4 ×2/3 ×1/6
x/12 = 15/1
x = 180, 30% of 180 = 54
Answer: a)

10). Because total no. of students is not known we cannot find the answer.
Answer: d)

310-320 Questions :

Number Series & Simplification

Directions (Q. 1-5): What should come in place of question mark(?) in the following
number series?
1). 65 219 517 1007 1737 ?
a) 2213
b) 2000
c) 2755
d) 2765
e) 2855

2). 561 642 763 932 1157 ?

www.bankingpdf.com
a) 1446
b) 1326
c) 1482
d) 1246
e) None of these

3). 1524 1443 1394 ? 1360 1359


a) 1303
b) 1218
c) 1359
d) 1369
e) 1329

4). 5 13 58 357 2868 ?


a) 25823
b) 28695
c) 29548
d) 28545
e) 27695

5). ? 61 211 505 991 1717


a) 6
b) 5 .
c) 8
d) 7
e) 10

www.bankingpdf.com
Directions (Q. 6-10): What approximate value should come in place of question mark (?)
in the following questions? (Note: You are not expected to calculate the exact value.)
6). 512.01 x 412.99 ÷ 119 =?
a) 1720
b) 1740
c) 1845
d) 1775
e) None of these

7).1699.99 x 299.88 ÷ 59.9 -1498 + 3745 = ?


a) 10980
b) 11700
c) 11000
d) 10750
e) 9800

8). (13.96)2 + (16.23)2 + (17.26)2 – 32.95 = ?


a) 790
b) 720
c) 840
d) 780
e) 680

9). 1624.98 x 29.92 + 468.75 =?


a) 49290
b) 48220
c) 47220
d) 46365
e) None of these

www.bankingpdf.com
10). 8499.99 ÷ 375.002 x 14.996 =?
a) 360
b) 290
c) 480
d) 380
e) 340

Solutions:

1). The series is 43+1, 63+3, 83+5, 103+7, 123+9, 143=11,…


So, the answer will be 2755.
Answer: c)
2). The series is +92, +112, +132, +152, ++172
Answer: a)
3). The Series is -92, -72, -52, -32, -12
Answer: d)
4). The series is 5x2+3 = 13, 13x4+6, 58x6+9=357, 357x8+12,…
Answer: b)
5). The series is 23-1, 43-3, 63-5, 83-7,…
Answer: d)
6). ?= 512.01x (412.99/119) = (512x413)/(17x7)
= (510x413)/ (17x7) = 30x59 = 1770 ≈1775
Answer: e)
7). ?= (1700x300)/ 60 – 1498+3745
= 510000/60 – 1490+3745
=8500-1498+3745

www.bankingpdf.com
= 12245 – 1498 = 10747 ≈ 10750
Answer: d)
8). ? = (14)2 + (16.2)2 + (17.25)2-33
=196+262.44+297.56-33
= 756 – 33- = 723 = 720 (Approx)
Answer: b)
9). ? = 1625 x 30 + 469
=48750 + 469 = 49219 ≈ 49220
Answer: c)
10). ? = (8500/375)x 15 ≈ 340
Answer: e)

320-330 Questions :

1). Vignesh purchased a TV set for Rs.11, 250 after getting discount of 10% on the
labelled price. He spent Rs.150 on transport and Rs.800 on installation. At what price
should it be sold so that the profit earned would have been 15% if no discount was
offered?
a) Rs.11877.50
b) Rs.16367.50
c) Rs.14247.50
d) Rs.15467.50
e) Rs.17567.50

2). A shopkeeper declares that he sells rice at the cost price. However he uses a weight
of 425 grams. Instead of 500 grams, what is his percentage of profit?
a) a) 16 5/6
b) 15
c) c) 20 7/23
d) 25

www.bankingpdf.com
e) None of these

3). Cost of fencing a circular plot at the rate of Rs.15 per metre is Rs.3, 300. What will
be the cost of flooring the plot at the rate of Rs.100 per square metre?
a) Rs.3, 85,000
b) Rs.4, 70,000
c) Rs.2, 25,000
d) Rs.3, 50,000
e) Rs.2, 95,000

4). Ashwin borrowed a sum of Rs.6, 300 from Mishra at the rate of 14% for 3 years. He
then added some more money to the borrowed sum and lent it to Ramesh at the rate of
16% of simple interest for the same time. If Ashwin gained Rs.618/- in the whole
transaction, then what sum did he lend to Ramesh?
a) Rs.6600
b) Rs.6800
c) Rs.4800
d) Rs.9800
e) Rs.8500

5). Mr. Sathyan invested Rs.20, 000/- with rate of interest@20 p.c.p.a. The interest was
compounded half yearly for first one year and in the next year it was compounded
yearly. What will be the total interest earned at the end of two years?
a) Rs.7600
b) Rs. 9040
c) Rs. 8750
d) Rs. 6900

www.bankingpdf.com
e) Rs. 4750

6). If the positions of the digits of a two digit number are interchanged, the number
obtained is smaller than the original number by 27. If the digits of the number are in the
ratio of 1 : 2, what is the original number ?
a) 25
b) 81
c) 36
d) Cannot be determined
e) None of these

7). The average daily wages of 450 workers in a factory is Rs.45/- and the average daily
wages of 340 workers in another factory is Rs.60/-.What is the approximate average
daily wages of workers from both the factories together?
a) Rs.61
b) Rs.59
c) Rs.70
d) Rs.51
e) Rs.60

8). In how many different ways can the letters of the word DESIGN be arranged so that
the vowels are at the two ends?
a) 48
b) 66
c) 12
d) 72
e) 96

www.bankingpdf.com
9). 8 men and 4 women-together can complete a piece of work in 6 days. Work done by
a man in one day is double the work done by a woman in one day. If 8 men and 4
women started working and after 2 days, 4 men left and 4 new women joined, in how
many more days will the work be completed?
a) 7 days
b) 6 days
c) 5 days
d) 8 days
e) 4 days

10). If the numerator of a fraction is increased by 300% and the denominator of the
fraction is increased by 150%, the resultant fraction is 3/5. What is the original fraction?
a) 7/5
b) 3/7
c) 7/15
d) 1/6
e) None of these

Answer:
1)d 2)b 3)a 4)b 5)b 6)d 7)d 8)a 9)c 10)e

Solution:
1). Marked price = 11250 × 10/9 = Rs. 12500
Total cost=12500 + 150 + 800=Rs. 13450
S.P = 13450 × 115/100 = Rs. 15467.50
Answer: d)

www.bankingpdf.com
2). % profit = Loss/Actual Weight x 100
= 500 – 425/500 × 100
= 75/500 x 100 = 15%
Answer: b)

3). Perimeter of plot = 3300/15 = 220 meter


2πr = 220
2 × 22/7 × r = 220
r = 35 m.
Area of plot = πr^2
= 22/7 × 35 × 35 = 3850 mt^2
Cost of flooring = 3850 × 100=Rs. 385000
Answer: a)

4). Suppose Ashwin lend Rs. x to Ramesh then,


Xx16x3/100 - (X – 6300) × 14 × 3/100 = 618......(1)
From equation I ,
x = Rs. 6800
Answer: b)

5). Total interest


= 20000 × 110/100 × 110/100 × 120/100 - 20000
= 29040 - 20000= Rs. 9040
Answer: b)

www.bankingpdf.com
6). Cannot be determined
Answer: d)

7). Total wages of a factory


=450 x 45= Rs. 20250/
Total wages of another factory
= 340 x 60
= Rs. 20400
Total average = 20250 + 20400/450 + 340
= 40650/790
= 51.46 = Rs. 51
Answer: d)

8). Total ways = 4 ! x 2 !


= 4 × 3 × 2 x 2= 48
Answer: a)

9). 1 man = 2 women


8 men + 4 women = 20 women
2 day's work of 20 women = 20 × 6/1
= 20 × 2/x
x =1/3
Remaining work = 1 – 1/3 = 2/3
Total number of females = 16
20 × 6/1 = 16 × x ÷2/ 3

www.bankingpdf.com
x = 5 days
Short Cut
M1D1=M2D2+M3D3
10x6 =10x2 + 8 x
60-20= 8x
x= 5 days
Answer: c)

10). x + 300% of x / y +150% of y = 3/5


400x/250y = 3/5
x/y = 3 × 250 /5 × 400
x/y = 3/8

Answer: e)

330-340 Questions :

Data Interpretation

Directions (Q. 01-05): Study the following graph carefully and answer the given
questions.
The following graph shows the ratio of import to export of three different countries
over the years.

www.bankingpdf.com
1). If the import of India was Rs. 4 million in 2013 and the export of Brazil was Rs. 6
million in 2010, by what per cent was the import of Brazil in 2010 more than the
export of India in 2013?
a) 50%
b) 8%
c) 68.75%
d) 4)80%
e) 5) None of these
2). If the amount of export of Pakistan in 2011 was Rs.3 million, then find the amount
of import of India in the same year.
a) Rs. 0.6 million
b) Rs. 1.2 million
c) Rs. 0.75 million
d) Can't be determined
e) None of these
3). In which of the following years was the amount of import the maximum for
Pakistan? (Assume that the amount of import is the same for each year.
a) 2015
b) 2011
c) 2013
d) None of these

www.bankingpdf.com
e) 2014
4). If the amount of import for India in the year 2014 and the amount of export for
Brazil in 2015 respectively be the same, then find the ratio of the amount of export in
2014 for India to the amount of import in 2015 for Brazil.
a) 5:2
b) 2 :5
c) 16:5
d) 5 : 16
e) None of these
5). If the amount of import for Brazil in 2012 was 214 million, then find the amount of
export for Brazil in the same year.
a) Rs. 30 million
b) Rs. 35 million
c) Rs. 40 million
d) Can't be determined
e) None of these

Directions (06-10): Study the following table carefully and answer the given
questions.
The following table shows the sales of different types of cars (in hundred) from
January to September in Mumbai.

www.bankingpdf.com
6). The sale of which car shows continuous increase with respect to the previous
month?
a) Audi
b) I20
c) Hummer
d) Ciaz
e) BMW
7). What was the percentage increase in the sale of the total no. of cars of last three
types from Jan to September?
a) 21%
b) 30%
c) None of these
d) 16.66%
e) 33.33%
8). Find the average difference between the Hummer cars and Lancer cars sold over
the given months?
a) 2700
b) 2650
c) 3700
d) 2400
e) None of these

www.bankingpdf.com
9). In which of the following months was the total number of cars sold the maximum?
a) January
b) June
c) May
d) September
e) July
10). The number of Volvo cars sold from Jan to September is approximately what
per cent more than that of I20 sold during the same period?
a) 45%
b) 35%
c) 50%
d) 55%
e) 40%

Answer:
1)b 2)d 3)c 4)a 5)b 6)d 7)b 8)a 9)e 10)a

Solution:
1). Import of Brazil in 2010 = 6×0.9 = Rs. 5.4 million
:. Export of India in 2013 = (4/0.8) = Rs. 5 million
:. Reqd more% = [(5.4 - 5)/5] × 100 = (4/5) × 10 = 8%
Answer: b)
2). We can’t calculate.
Answer: d)
3). The ratio is maximum for Pakistan in the year 2015. Thus, the amount will be
maximum in that year.
Answer: c)
4). Import of India in 2014 = Export of Brazil in 2015 = x(say)

www.bankingpdf.com
:. Reqd ratio = (x/0.4) × (1/x) = 5 : 2
Answer: a)
5). Amount of export for Brazil in 2012 = 14× (1/0.4) = Rs. 35 million
Answer: b)
6). Ciaz only shows continuous increase with respect to the previous months.
Answer: d)
7). Total number of reqd cars sold in January = 22 + 30 + 18 = 70
Total number of reqd cars sold in September = 30 + 40 + 21 = 91
:. Reqd % increase = [(91-70)/70] × 100 = 30%
Answer: b)
8). Reqd Difference = [(552-363) / 7] × 100 = 2700
Answer: a)
9). Total no.of cars sold in different months Jan – 21400
June – 34800
July – 37000
May – 32300
September – 35100
Hence the total number of cars sold was the maximum in the month of July.
Answer: e)
10). :. Reqd % more = [(463 - 319) / 319] × 100 = 45%.

Answer: a)

340-350 Questions :

Data Interpretation

Directions (Q 1-5): The following pie chart shows the percentage distribution of the expenditure
incurred in publishing a book. Study the pie chart and answer the following questions.

www.bankingpdf.com
1). What is the central angle of the sector corresponding to the expenditure incurred on
Promotion ?

a) 84 degree

b) 88 degree

c) 95 degree

d) 90 degree

e) 80 degree

2). Which two expenditures together have a central angle of 126 degree in pie chart ?
a) Binding cost + Royalty cost

b) Printing cost + Paper cost

c) Binding cost + Transportation cost

d) Printing cost + Transportation cost

e) Royalty cost + Transportation cost

www.bankingpdf.com
3). If for a edition of a book, the cost of paper is Rs. 48500, then find the printing cost for this
edition ?
a) Rs 87500

b) Rs 87300

c) Rs 86500

d) Rs 86700

e) Rs 86300

4). If 7500 copies are published and the transportation cost on them amounts to Rs. 45500,
then what should be the selling price of the book so that the publisher can earn a profit of 20 %
?
a) Rs 48.53

b) Rs 45 .53

c) Rs 43.50

d) Rs 47.50

e) Rs. 49.73

5). Printing on the book is less than the Promotion cost by ?


a) 20%

b) 25%

c) 28%

d) 29%

e) 26%

www.bankingpdf.com
Directions (Q 6-10) : Study the following line graph which gives the number of students who
joined and left the school in the beginning of the year for six years from 2006 to 2011.

Initial strength of the school in 2005 was 2500 Answer questions based on the line graph given
below.

6). The strength of the school increased from 2007 to 2009 by what percent ?
a) 6.5 %

b) 6.25%

c) 7.25%

d) 6%

e) 6.75 %

7). The number of students studying in the school during 2010 was :

a) 2750

www.bankingpdf.com
b) 2400

c) 2600

d) 2250

e) 2550

8). The number of students studying in the school in 2010 was what percent of the number of
students studying in the school in 2007?
a) 108.75%

b) 106.75%

c) 105.25%

d) 106.25%

e) 108.50%

9). During which of the following pairs of years, the strength of the school was same ?
a) 2009 & 2010

b) 2006 & 2010

c) 2009 & 2007

d) 2008 & 2007

e) 2011 & 2010

10). For which year, the percentage rise/fall in the number of students who joined the school
compared to previous years is maximum ?
a) 2011

b) 2010

www.bankingpdf.com
c) 2007

d) 2008

e) 2009

Answers:

1).d) 2).e) 3).b) 4).a) 5).c) 6).b) 7).e) 8).d) 9).a) 10).c)

Solution:

1). Central angle corresponding to the promotion will be,

25% of 360 degree

= (25/100)*360

= 5*18 = 90 degree

Answer: d)

2). This type of question can be solved, first by getting 130 degree is what percent of 360, Then
choosing the correct option. So let solve,

126 is what percent of 360 ?

= (126 / 360)∗100%=35% of expenditure

So from the options given, it is clear that

Royalty cost (20%)+ Transportation cost (15%) = 35%

Answer: e)

www.bankingpdf.com
3). We are having the ratios of the percentage of two (from pie chart) also having cost of one.

So we can solve it in an easy way as,

10:18=48500: P

=> P=(48500∗18/10)=Rs.87300

Answer: b)

4). For the publisher to earn a profit of 20%,

S.P. = 120% of C.P.

Also transportation Cost = 15% of C.P.

Let the S.P. of 7500 books be Rs x.

15:120=45500: x =>x=120∗45500 / 15 => x=364000

S.P. of one book =364000 / 7500 =48.53

Answer: a)

5). Printing cost of book = 18% of C.P.

Promotion on book = 25% of C.P.

Difference = (25%-18%) = 7% of C.P.

% Differene = ((Difference / promotion cost)∗100)%

=((7% of C.P./ 25% of C.P.) ∗100)%=28%

Answer: c)

6). Let analyse the graph before answering question.

www.bankingpdf.com
Number of students in 2005 = 2500 [given]

Number of students in 2006 = 2500-500+450 = 2450

Number of students in 2007 = 2450-650+600 = 2400

Number of students in 2008 = 2400-450+750 = 2700

Number of students in 2009 = 2700-700+550 = 2550

Number of students in 2010 = 2550-700+700 = 2550

Number of students in 2011 = 2550-750+800 = 2600

Percentage increase in strength of the school from 2007 to 2009 will be,

((2550−2400)/2400)∗100%=6.25%

Answer: b)

7). Number of students in 2005 = 2500 [given]

Number of students in 2006 = 2500-500+450 = 2450

Number of students in 2007 = 2450-650+600 = 2400

Number of students in 2008 = 2400-450+750 = 2700

Number of students in 2009 = 2700-700+550 = 2550

Number of students in 2010 = 2550-700+700 = 2550

Answer: e)

8). In 2010 students = 2550

In 2007 students = 2400

Required Percentage =(2550/2400)∗100%=106.25%

www.bankingpdf.com
Answer: d)

9). we find in 2009 and 2010 number of students was same i.e. 2550

Answer: a)

10). Please note we are calculating the percentage rise/fall of number of students left.

for 2007=(600−450) /450∗100%=33.33%(rise) for2008=(750−600)/600∗100%=25%(rise)


for2009=(550−750)/750∗100%=26.67%(fall) for2010=(700−550)/550∗100%=27.27%(rise)
for2011=(800−700)/700∗100%=14.28%(rise)

So it was maximum in 2007.

Answer: c)

350-360 Questions :

Arithmetic

1). Madhan was 3 times as old as his daughter 10 years ago. After 10 years,
Madhan will be twice as old as his daughter. Find out the present age of Madhan.
a) 68 years
b) 70 years
c) 40 years
d) 55 years
e) 80 years

2). P walks around a circular field at the rate of two rounds per hour while Q runs
around it at the rate of eight rounds per hour. They start in the opposite direction
from the same point at 6.45 a.m. They shall first cross each other at?

www.bankingpdf.com
a) 6.45 a.m
b) 6.50 a.m
c) 7.51 a.m
d) 6.51 a.m
e) 7.45 am

3). Three pipes P, Q and R can fill a tank from empty to full in 40 minutes, 15
minutes, and 30 minutes respectively. When the tank is empty, all the three pipes
are opened. P, Q and R discharge chemical solutions X, Y and Z respectively. What
is the proportion of the solution Y in the liquid in the tank after 5 minutes?
a) 8 / 15
b) 7 / 15
c) 8 / 17
d) 6 / 13
e) 8 / 13

4). A boat running downstream covers a distance of 20 km in 5 hours while for


covering the same distance upstream, it takes 10 hours. What is the speed of the
stream?
a) 2km/hr
b) 4km/hr
c) 1km/hr
d) 1.5 km/hr
e) None of these

5). Two stations X and Y are 170 km apart on a straight line. One train starts from X
at 6 a.m. and travels towards Y at 25 kmph. Another train starts from Y at 8 a.m. and
travels towards X at a speed of 35 kmph. At what time will they meet?
a) 11. 30 a.m
b) 10.30 a.m

www.bankingpdf.com
c) 11 a.m
d) 9 a.m
e) 10 am

6). Simple interest on a certain sum of money for 4 years at 5% per annum is half
the compound interest on Rs. 8000 for 2 years at 10% per annum. The sum placed
on simple interest is:
a) Rs.4200
b) b)Rs.4250
c) Rs.5250
d) Rs.5200
e) Rs.4000

7). On selling 25 balls at Rs. 1015, there is a profit equal to the cost price of 10 balls.
What is the cost price of a ball?
a) Rs. 31
b) Rs. 29
c) Rs. 35
d) Rs. 27
e) Rs. 39

8). A letter lock consists of four rings each marked with five different letters. The
number of distinct unsuccessful attempts to open the lock is at the most -.
a) 625
b) 676
c) 576
d) 624
e) 575

www.bankingpdf.com
9). Arun started a business with Rs. 40,000 and is joined afterwards by Arul with
Rs.60, 000. After how many months did Arul join if the profits at the end of the year
are divided equally?
a) 3
b) 4
c) 5
d) 6
e) 2

10). Kathir and Anand can do a work in 20 days and 30 days respectively. Kathir
started the work and left after 4 days. Anand took over and completed the work. In
how many days was the total work completed?
a) 28 days
b) 20 days
c) 23 days
d) 25 days
e) 26 days

ANSWERS:
1. b
Let age of the daughter before 10 years = x
Then, age of Madhan before 10 years = 3x
After 10 years, Madhan will be twice as old as his daughter.
⇒ 3x + 20 = 2( x+20)
⇒ x = 20
⇒ Present age of Madhan = 3x + 10
= 3× 20+ 10
= 70 years.

2. d

www.bankingpdf.com
Since P and Q move in the opposite direction along the circular field, so they will first
meet each other when there is a difference of one round between the two.
Relative speed of P and Q = 8+2 = 10 rounds per hour.
Time taken to complete one round at this speed = 1/10 hr = 6 min.
6 Hrs 45 min + 6 min = 6 hrs 51 min i.e, 6.51 a.m

3. a
Part of the tank filled by pipe P in 1 minute = 1 / 40
Part of the tank filled by pipe Q in 1 minute = 1 / 15
Part of the tank filled by pipe R in 1 minute = 1/ 30
Here we have to find the proportion of the solution Y.
Pipe Q discharges chemical solution Y.
Part of the tank filled by pipe Q in 5 minutes
= 5 × 1 / 15 = 1 / 3
Part of the tank filled by pipe P, Q, R together in 1 minute
= 1/40 + 1/15 + 1/30
= (3+8+4)/120 = 15 / 120 = 1 / 8.
Part of the tank filled by pipe P, Q, R together in 5 minute
= 5 × 1/8 = 5/ 8
Required proportion = (1/3) / ( 5/8) = 8 / 15

4. c
Rate of downstream=(20 / 5 ) kmph= 4kmph
Rate of upstream =( 20/10) kmph= 2kmph
Therefore Speed of the stream= (1/2)(4 - 2) kmph= 1 kmph

5. e
Suppose they meet z hours after 6 a.m.
Distance covered by X in z hours = 25× z km.
Distance covered by Y in (z - 2) hours = 35(z - 2) km.

www.bankingpdf.com
Therefore 25z + 35(z- 2) = 170
60z = 240
x = 4.
So, they meet at 10 a.m.

6. a
C.I = 8000 * (1+(10/100))2 –8000
= 8000 *110/100 * 110/100 – 8000 = 9680 – 8000 = Rs.1680
Therefore, S.I = C.I / 2 = 1680 / 2=Rs. 840
840 = sum * 4 * 5 / 100
:. Sum = Rs.(840 * 100)/ (4 *5) = Rs.4200

7. b
Profit = (S.P of 25 balls) - (C.P of 25 balls) = 1015 - (C.P of 25 balls)
Given that Profit = (C.P of 10 balls)
=> 1015 - (C.P of 25 balls) = (C.P of 10 balls)
=> (C.P of 25 balls) + (C.P of 10 balls) = 1015
=> C.P of 35 balls = 1015
=> C.P of 1 ball=1015 / 35 = Rs. 29

8. d
Since each ring consists of five different letters, the total number of attempts
possible with the four rings is =5 * 5 * 5 * 5 = 625. Of these attempts, one of them is
a successful attempt.
Maximum number of unsuccessful attempts = 625 - 1 = 624.

9. b
Suppose, Arul joined after x months.
Then, 40000 * 12 = 60000 * (12 – x) => 48 = 72 - 6x => 6x = 24 → x = 4

10. a

www.bankingpdf.com
Kathir’s one day’s work= 1/20
Kathir’s 4 day’s work =4* (1/20) = 1/5
Work left= 1-1/5 = 4 /5
Anand’s one day’s work= 1/30
Anand can do work in = (4/5) *(30/ 1) = 24 days
So total days = 24+4 = 28 days

360-370 Questions :

Data Interpretation

Directions Q 1- 5: The bar graph provided below gives the data of the production of
paper (in lakh tonnes) by three different companies X, Y, Z over the years. Study the
bar chart and answer the questions.

1). What is the difference between the production of Company A in 2004 and
Company B in 2003?
a) 10,00,000 tonnes

www.bankingpdf.com
b) 15,00,000 tonnes
c) 20,00,000 tonnes
d) 25,00,000 tonnes
e) 5,00,000 tonnes

2). What is the ratio of the average production of Company C in the period 2002 -
2005 to the average production of company A in the same period?
a) 47:46
b) 49:47
c) 46:47
d) 49:46
e) 46:49

3). What is the percentage increase in the production of Company C from 2003 to
2005?
a) 27%
b) 26%
c) 28%
d) 25%
e) 30%

4). The average production for five years was maximum for which Company?
a) C
b) A
c) B
d) A and C
e) B and C

www.bankingpdf.com
5). In which year the percentage of production of Company B to the production of
company A is maximum?
a) 2004
b) 2002
c) 2001
d) 2005
e) 2003

Directions Q 6- 10: Study the following table and answer the questions based on it.

6). What is the average amount of interest per year which the company had to pay
during this period?
a) Rs. 22.43 lakhs
b) Rs. 31.72 lakhs
c) Rs. 28.22 lakhs
d) Rs. 26.66 lakhs
e) Rs. 24.22 lakhs

www.bankingpdf.com
7). Total expenditure on all these items in 1996 was approximately what percent of
the total expenditure in 1999?
a) 62%
b) 66%
c) 69%
d) 71%
e) 76%

8). The total expenditure of the company over these items during the year 1997 is?
a) Rs. 720.05 lakhs
b) Rs. 650.53 lakhs
c) Rs. 770.24 lakhs
d) Rs. 690.51 lakhs
e) None of these

9). The ratio between the total expenditure on Taxes for all the years and the total
expenditure on Fuel and Transport for all the years respectively is approximately?
a) 569:445
b) 443:560
c) 565:448
d) 445:569
e) None of these

10). The total amount of bonus paid by the company during the given period is
approximately what percent of the total amount of salary paid during this period?
a) 0.1%
b) 1%

www.bankingpdf.com
c) 1.5%
d) 1.25%
e) 0.25%

ANSWERS:
1. b
As per bar chart, Difference will be
[80-65]*100000 = 15,00,000 tonnes
Please note: we multiplied it by 100000, because graph is given in "lakh" tones
2. d
Average production of Company A in the period 2002 – 2005 = ( 55+ 60+ 65+ 50)/4=
57.5
Average production of Company C in the period 2002 – 2005 = (45+ 55+ 75+ 70)/4
=61.25
Required ratio = 61.25 : 57.5 = 2.45 : 2.30 = 49: 46
3. a
Percentage increase in the production of Company C from 2003 to 2005 = ((70-55) /
55) * 100
=(15/55) *100 = 27.27 % ≈ 27%
4. c
Average production of Company A in the period 2001 – 2005 = (70+ 55+ 60+ 65+
50)/5= 60
Average production of Company B in the period 2001 – 2005 = (55+ 65+ 80+45+
65)/5= 62
Average production of Company C in the period 2001 – 2005 = (60+45+ 55+ 75+
70)/5 =61
So company B has the maximum average.
5. e
2001 → % = (55 /70)*100= 78.5 %

www.bankingpdf.com
2002 → % = (65 / 55)*100= 118.2 %
2003→ % = (80/60)*100= 133.3%
2004→ % = (45/65)*100 = 69.2%
2005→ % = (65/50)*100= 130%
So for 2003 the percentage is highest.
6. c
Average amount of interest paid by the Company during the given period
= Rs.[21.5+34.2+24.3+29.6+31.5]/5 lakhs
= Rs.[141.1/5] lakhs

= Rs. 28.22 lakhs.

370-380 Questions :

Data Interpretation & Quadratic Equation

Directions (Q. 1-5): Study the following table carefully to answer these questions.
Number of students appeared and passed in an examination from five different exams
over the years

www.bankingpdf.com
1). What is the ratio between average number of students passed from Exams RRB PO
and RRB CLERK respectively for all the given years?
a) 70:51
b) 70:53
c) 53:70
d) 51:70
e) None of these

2). For Exam IBPS CLERK, which year had the lowest percentage of students passed
over appeared?
a) 2011
b) 2012
c) 2013
d) 2014
e) 2015

3). What is the ratio between the total numbers of students appeared from all the exams
together in the year 2011 and 2012 respectively?
a) 286 :295

www.bankingpdf.com
b) 277: 286
c) 286 : 277
d) 295 : 286
e) None of these

4). During year 2013, which Exam had the highest percentage of students passed over
appeared?
a) RRB CLERK
b) RRB PO
c) IBPS PO
d) IBPS CLERK
e) IBPS SO

5). What was the overall percentage of students passed over the number of students
appeared from all the exams together in the year 2014? (Rounded off to next integer)
a) 74
b) 73
c) 76
d) 72
e) None of these

Directions (Q. 6-10): In each of these questions, two equations (I) and (II) are given.
Solve both the equations and give answer
a) if x > y
b) if x < y
c) if x ≥ y
d) if x ≤ y
e) if x = y or no relation can be established between ‘x’ and ‘y’.

www.bankingpdf.com
6).
I. x - 7 √3x + 36 = 0
II. y -12 √2y + 70 = 0

7).
I. 10x + 6y = 13
II. 45x + 24y = 56

8).
I. 63x -194 √x +143 = 0
II. 99y - 255 √y +150 = 0

9).
I. 16x^2 – 40x – 39 = 0
II. 12y^2 – 113y + 255 = 0

10).
I. x^2 - 7 √7x + 84 = 0
II. y^2 - 5 √5y + 30 = 0

Answers:
1)d 2)d 3)b 4)e 5)a 6)b 7)b 8)e 9)b 10)a

1). Total Number of students passed from exam RRB PO / Total Number of students
passed from exam RRB CLERK
Average = 250 + 300 + 280 + 320 + 380/5 ÷ 350 + 420 + 400 + 450 + 480/5
= 1530/5 ÷ 2100/5
= 306 : 420
= 102 : 140

www.bankingpdf.com
= 51 : 70
Answer: d)

2). Percentage of passed students of exams IBPS CLERK in the year 2011
= 460/580 × 100 = 79.31%
Percentage of passed students of exams IBPS CLERK in the year 2012
= 480/600 × 100 = 80%
Percent of passed student of exam IBPS CLERK in the year 2013
= 420/560 × 100 = 75%
Percentage of passed students of exam IBPS CLERK in the year 2014
= 450/620 × 100 = 72.58%
Percentage of passed students of exam IBPS CLERK in the year 2015
=520/640 × 100 = 81.25
From the above calculation, it is clear that in the year 2014, exam IBPS CLERK have
lowest percentage of students passed over appeared.
Answer: d)

3). Total no. of students appeared in 2011 from all the exams together.
= 600+450+520+580+620
= 2770
Total no of students appeared in 2012 from all the exams together
= 580+480+550+600+650
= 2860
Ratio = 2770 : 2860
= 277 : 286
Answer: b)

4). Percentage of passed students in 2013 of exam IBPS PO


= 300/640 × 100 = 46.88%

www.bankingpdf.com
Percentage of passed students in 2013 of exams RRB PO
= 280/420 × 100 = 66.67%
Percentage of passed students in 2013 of exams RRB CLERK
= 400/500 × 100 = 80%
Percentage of passed students in 2013 of exam IBPS CLERK
= 420/560 × 100 = 75%
Percentage of passed students in 2013 of exam IBPS SO
= 500/580 × 100 = 86.21%
From the above calculation of all exams during the year 2013 exam E have highest
percentage of students passed over appeared.
Answer: e)

5). Total no. of students appeared in 2014


= 650+460+560+620+660
= 2950
Total no. of students passed in an examination in the year 2014
= 400+320+450+450+550= 2170
= 2170/2950 × 100 = 74%
Answer: a)

6). I. x - 7 √3x + 36 = 0
or x - 7 √3. √x + 36 = 0
or x - 3 √3. √x - 4 √3. √x + 36 = 0
or ( √x - 3 √3)(√x - 4√3) = 0
x = 27, 48
II. y - 5 √2y - 7 √2y + 70 = 0
or y - 5 √2. √y - 7 √2. √y + 70 = 0
or ( y - 5 2)( y - 7 2) = 0
y = 50, 98

www.bankingpdf.com
x<y
Answer: b)

7). I. 10x + 6y = 13
II. 45x + 24y = 56
On solving both equations,
x = 4/5, y = 5/6
Answer: b)

8). I. 63x -194 √x +143 = 0


or63x -117 √x - 77√x +143 = 0
or (7 √x -13)(9 √x -11) = 0
x = 169/49, 121/81
II. 99y - 225 √y +150 = 0
or 99y - 90 √y -165 √y +150 = 0
or (11 √y -10)(9√y -15) = 0
y = 100/121, 225/81
Therefore relation cannot be established between x and y.
Answer: e)

9). I. 16x^2 - 40x - 39 = 0


or 16x^2 - 52x + 12x - 39 = 0
or (4x- 13) (4x + 3)
x = 13/4, -3/4
II. 12y^2 - 113y + 255 = 0
or 12y^2 - 45y - 68y + 255 = 0
or (4y - 15) (3y - 17) = 0
y = 15/4, 17/3
Therefore y > x or, x < y

www.bankingpdf.com
Answer: b)

10). I. x^2 - 7 √7x + 84 = 0


or (x - 4 √7)(x - 3√7) = 0
x = 4 √7, 3 √7
II. y^2 - 5 √5y + 30 = 0
or (y - 2 √5)(y - 3 √5) = 0
y = 2 √5, 3 √5
x>y

Answer: a)

380-390 Questions :

Simplification & Number Series

Directions (Q. 1–5): What approximate value should come in place of the question mark
(?) in the following questions?

1). (4.36)^2 x 66.5 - 371 = ?


a) 850
b) 893
c) 910
d) 875
e) 885

2). 3√7469 x √668 = ?


a) 450
b) 300
c) 505

www.bankingpdf.com
d) 465
e) 555

3). 2.5 x 0.08 ÷ (1.9)^2= ?


a) 2.5
b) 1.2
c) 0.008
d) 0.06
e) 0.03

4). 1/8 × 3/5 × 4/7 x 7897 = ?


a) 310
b) 325
c) 375
d) 275
e) 338

5). 9654 ÷ 21 +7638 ÷ 44 = ?


a) 633
b) 600
c) 643
d) 621
e) 598

Directions (Q. 6-10): What value should come in place of the question mark (?) in the
following questions?

www.bankingpdf.com
6). 3 9 15 ? 35 49
a) 25
b) 30
c) 24
d) 20
e) None of these

7). 259 256 ? 244 235 224


a) 250
b) 251
c) 253
d) 249
e) None of these

8). 729 656 595 ? 494


a) 540
b) 530
c) 500
d) 529
e) 541

9). 4 32 ? 2048 16348


a) 250
b) 286
c) 256

www.bankingpdf.com
d) 225
e) None of these

10). 4 8 24 ? 124 224


a) 60
b) 65
c) 59
d) 62
e) None of these

Answers:
1)b 2)c 3)d 4)e 5)a 6)a 7)b 8)a 9)c 10)a

Solution:
1). (4.36)^2 × 66.5 - 371 = ?
= 19.0096 × 66.5 - 371 = ?
= 1264.1384 - 371
= 893.1384
= 893 (approx.)
Answer: b)

2). 3√7469 × √668 = ?


= 19.5 × 25.9
= 505.05 = 505 (approx)
Answer: c)

www.bankingpdf.com
3). 2.5 × 0.08 ÷ (1.9)^2 = ?
= 2.5 × 0.08/3.61 = ?
= 0.0554
= 0.06 (approx)
Answer: d)

4). ? = 7897 × 4 × 3 / 8 × 5 × 7
= 338.31
= 338 (approx.)
Answer: e)

5). 9654 ÷ 21 + 7338 ÷ 44 = ?


= 459.714 + 173.59
= 633.304
= 633 (approx)
Answer: a)

6). 22 – 1, 32, 42 – 1, 52, 62-1.


Answer: a)

7). Difference of odd numbers 3, 5, 7, 9, 11.


Answer: b)

www.bankingpdf.com
8). Difference of Difference 9.
Answer: a)

9). Multiply by 8.
Answer: c)

10). Add with 22, 42, 62, 82, 102.


Answer: a)

390-400 Questions :

1). Saina spent Rs. 44,668/- on her air tickets, Rs.56,732 on buying gifts to the family
members and the remaining 22% of the total amount she had as cash with her. What
was the total amount ?
a) Rs. 1,25,800/-
b) Rs. 1,30,000/-
c) Rs. 1,10,500/-
d) Rs. 1,68,300/-
e) Rs. 1,50,000/-

2). The profit earned after selling a Laptop for Rs.1,754/- is the same as loss incurred
after selling the article for Rs.1,492/-. What is the cost price of the article?
a) Rs. 1,239/-
b) Rs. 1,456/-
c) Rs. 1,786/-
d) Rs. 1,623/-
e) Rs. 1,345/-

www.bankingpdf.com
3). The compound interest accrued on an amount of Rs. 25,500 at the end of three
years is Rs. 8,440.5.What would be the simple interest accrued on the same amount at
the same rate in the same period?
a) Rs. 5,450/-
b) Rs. 8,750/-
c) Rs. 4,350/-
d) Rs. 3,650/-
e) Rs. 7,650/-

4). A sum of money is divided among Suresh, Ganesh, Vignesh and Mahesh in the ratio
of 3 : 4 : 9 : 10 respectively. If the share of Vignesh is Rs.2,580/-more than the share of
Ganesh, then what is the total amount of money of Suresh and Mahesh together ?
a) Rs. 6,985/-
b) Rs. 6,487/-
c) Rs. 6,708/-
d) Rs. 7,156/-
e) Rs. 8,457/-

5). Deepa decided to donate 16% of her monthly salary to an NGO. On the day of
donation she changed her mind and donated Rs. 6,567/- which was 75%of what she
had decided earlier. How much is Deepa’s monthly salary?
a) Rs. 54,725/-
b) Rs. 58,756/-
c) Rs. 56,700/-
d) Rs. 55,696/-
e) Rs. 52,696/-

Directions (Q. 6-10): What value should come in place of the question mark (?) in the
following questions?

www.bankingpdf.com
6). 1 3/4 + 5 1/3 + 3 2/5 = ?
a) 10 29/60
b) 8 29/60
c) 6 29/60
d) 4 29/60
e) None of these

7). √(4.9 − 0.49) = ?


a) 10.42
b) 5.84
c) 12.84
d) 2.1
e) None of these

8). Value of 1/ {2 + [1 / (2 +1/[ 2 – 1/2 ])]}


a) 6/19
b) 7/19
c) 8/19
d) 9/19
e) None of these

9). 720% of 40 + 520 = x – 28


a) 800
b) 700
c) 820
d) 840
e) 500

www.bankingpdf.com
10). (1024 − 263 − 233) ÷ (986 − 764 − 156) = ?
a) 9
b) 6
c) 7
d) 8
e) None of these

Answers:
1)b 2)d 3)e 4)c 5)a 6)a 7)d 8)c 9)a 10)d

Solution:
1). Saving = 22%
Spent = a00 - 22) = 78%
Spending money = 44668 + 56732 = 101400
78%- 101400
100% = 101400 100/78 = 130000
Answer: b)

2). Let the C.P. of a Laptop = Rs. x


1754 - x = x - 1492
2x = 3246
x = 3246 / 2 = 1623
Answer: d)

3). A = P + Interest
= 25500 + 8440.5 = 33940.5
A = P + 1 + (R/100)^3
33940.5 = 25500 a + R/100)^3

www.bankingpdf.com
a + R/100)^3 = 33940.5 / 25500
a + R/100)^3 = 1.331
a + R/100)^3 = a.1)^3
1 + R/100 = 1.1
R/100 = 1.1 - 1
R/100 = 0.1
R = 0.1×100
R = 10%
Simple Interest = Principle × Rate × Time / 100
= 25500 × 10 × 3/100 = 7650
Answer: e)

4). According to the question, difference of ratio of Ganesh and Vignesh is 5


so, 1 ratio = 2580/5 = 516
Now, total ratio of Suresh + Mahesh = 13
So, total money = 13 × 516 = Rs. 6708
Answer: c)

5). Let the Deepa Monthly Salary = x


Deepa decided to donate 16% of her monthly salary.
On the day of donation she changed.
So, 75%of she had decided earlier of 16%
16 × 75/100 = 12%
Therefore = 12%= Rs. 6567
100% = Rs .6567 × 100 / 12
x = Rs. 54725
Answer: a)

6).

www.bankingpdf.com
Answer: a)

7).
Answer: d)

8).
Answer: c)

9).
Answer: a)

10).
Answer: d)

400-410 Questions :

Simplification & Data Interpretation

Directions (Q. 1-5): Study the following graph carefully to answer these questions.
Number of students (Males & Females) passed out from various Colleges in a year.
(Number in thousands)

www.bankingpdf.com
1). The number of Males passing out from colleges A and B together is what percent of
the number of females passing out from colleges C and D together?
a) 45
b) 40
c) 35
d) 50
e) None of these

2). What is the average number of students (Males & Females) passed out from all the
colleges together?
a) 38000
b) 48000
c) 42000
d) 51000
e) None of these

www.bankingpdf.com
3). What is the respective ratio of the total number of Males to the total number of
Females passed out from all the colleges together?
a) 19:23
b) 18:25
c) 23:19
d) 25:18
e) None of these

4). The number of Females passed out from college C is approximately what percent
the total number of Females passed out from all the colleges together?
a) 28
b) 30
c) 36
d) 25
e) 40

5). What is the difference between the total number of students passing out from college
A and the total number of students passing out from college E?
a) 20,500
b) 21,000
c) 10,500
d) 10,000
e) None of these

Directions (Q. 6-10): What will come in place of the question mark (?) in the following
questions ?
6). 12.28× 1.5 -36÷ 2.4 = ?
a) 3.24
b) 7.325

www.bankingpdf.com
c) 6.42
d) 4.32
e) None of these

7). 185% of 400 + 35% of 240 = ?% of 1648


a) 85
b) 75
c) 125
d) 50
e) None of these

8). 66^2-34^2 =?
a) 3600
b) 3200
c) 2146
d) 2466
e) None of these

9). √24^4 + 224 = ? × 20^2


a) 20
b) 4
c) 2
d) 16
e) None of these

10). 3/8th of 4/9th of 1092 = ?


a) 182
b) 728

www.bankingpdf.com
c) 364
d) 218
e) None of these

Answers:
1)d 2)c 3)a 4)b 5)e 6)e 7)d 8)b 9)c 10)a

Solution:
1). Passed Boys from college A and B
= 15000 + 17500
Passed Girls from College C and D
= 35000 + 30000
% = 32500/65000 × 100 = 50%
Answer: d)

2). Average number of passed students in all Colleges


Boys + Girls = Students
A =15000 + 22500= 37500
B = 17500 + 20000= 37500
C = 27500 + 35000= 62500
D = 25000 + 30000= 55000
E= 10000 + 7500 = 17500
Total = 210000
Average = 210000/5 = 42000
Answer: c)

3). Ratio between passed all boys and all girls in all colleges
Boys : Girls
95000 : 115000

www.bankingpdf.com
95 : 115
19 : 23
Answer: a)

4). Passed Girls from college C = 35000


Passed Girls from All college = 115000
Approximate % = 35000 / 115000 × 100 = 29.78 = 30%
Answer: b)

5). Passed students from college A= 37500


Passed students from college E = 17500
Difference = 37500 - 17500
= 20000
Answer: e)

6). 12.28 × 1.5 - 15 = ?


18.42 - 15 = ?
? = 3.42
Answer: e)

7). 740 + 84 = 16.48 × ?


824 = 16.48 × ?
? = 50
Answer: d)

8). (66 + 34) (66 - 34) = ?


100 × 32 = ?

www.bankingpdf.com
? = 3200
Answer: b)

9). 576 + 224 = ? × 400


800 = ? × 400
?=2
Answer: c)

10). 1092 × 4/9 × 3/8 = ?


1092 × 1/3 × 1/2 = ?
? = 182
Answer: a)

410-420 Questions ::

1). Ages of DEEPA and Subha are presently in the ratio of 5 : 6 respectively. Six years
hence this ratio will become 6 : 7 respectively. What was Subha’s age 5 years ago?
a) 33 years
b) 32 years
c) 35 years
d) 31 years
e) None of these

2). Ishath bought 15 pieces of Mobiles@RS.4,500/- each and sold all of them at the
total price of Rs.81, 000/-. What is the percent profit earned in the deal?
a) 18.66
b) 20
c) 25
d) 36

www.bankingpdf.com
e) None of these

3). If the compound interest accrued on an amount of Rs.15,000/- in two years is


Rs.2,496/-.What is the rate of interest p.c.p.a.?
a) 8
b) 12
c) 7.5
d) Cannot be determined
e) None of these

4). 75%of a number is equal to four-fifth of another number. What is the ratio between
first number and the second number?
a) 4:3
b) 30:32
c) 7:9
d) 16:15
e) None of these

5). If the numerator of a fraction is increased by 25% and the denominator is doubled,
the fraction thus obtained is 5/9. What is the original fraction?
a) 3/4
b) 7/8
c) 8/9
d) Cannot be determined
e) None of these

www.bankingpdf.com
6). If the digits of a two digit number are interchanged, the number formed is greater
than the original number by 45. If the difference between the digits is 5.What is the
original number?
a) 18
b) 25
c) 36
d) Cannot be determined
e) None of these

7). Area of a rectangle is equal to the area of circle whose radius is 14 cms. If the
breadth of the rectangle is 22 cms. What is its length?
a) 27 cms
b) 28 cms
c) 25 cms.
d) 29 cms
e) None of these

8). In how many different ways can the letters of the word DISPLAY be arranged?
a) 2601
b) 676
c) 1724
d) 2401
e) 5040

9). Avinash, Bala and Chandran invested Rs.45,000/-, Rs. 70, 000/- and Rs. 90,000/-
respectively to start a business. At the end of two years, they earned a profit of
Rs.1,64,000/-. What will be Bala’s share in the profit?

www.bankingpdf.com
a) Rs.56, 000/-
b) Rs.46, 000/-
c) Rs. 50, 000/-
d) Rs.75, 000/-
e) None of these

10). Mr. Sachin spent 20% of his monthly income on food and 15% on children’s
education. 40% of the remaining he spent on entertainment and transport together and
30% on the medical. He is left with an amount of Rs.8, 775/- after all these
expenditures. What is Mr. Sachin’s monthly income ?
a) Rs.50,000/-
b) Rs.46,000/-
c) Rs.36, 000/-
d) Rs.29,000/-
e) None of these

Answers:
1)d 2)b 3)a 4)d 5)c 6)d 7)b 8)e 9)a 10)e

Solution:
1). Present ratio of Deepa&Subha = 5 : 6
Six year hence ratio of Deepa&Subha = 6 : 7
Difference between ratio = 1
Difference in ages between present & future = 6 year.
1 ratio=6 yrs & SUBHA's age = 6×6 =36 year
5 year ago SUBHA's age was =36 – 5 = 31 year
Alternatively,

www.bankingpdf.com
SUBHA's future age = 7×6 = 42 year
B's age 5 year ago = 42–6–5
= 42–11 = 31 year
Answer: d)

2). Total cost of 15 pieces of Mobiles= 15×4500 = Rs. 67500


Total Sales prices of 15 pieces of Mobiles= Rs. 81000
Profit% = 81000 – 67500 / 67500 × 100
= 13500/67500 × 100 = 20%
Answer: b)

3). Let the rate of interest p.c.p.a. be x.


2496 =15000 * x /100
x = 16.64
16.64 is the rate of interest compounded for two year & for one year = 8%
8% C.I. table for two year.
I year = 8%
II year = 8.64
Total =16.64%
Answer: a)

4). Let the two number be x & y.


According to question,
75/100 x : 4/5 y
3/4 x : 4/5 y
15x = 16y
x /y = 16/15

www.bankingpdf.com
Answer: d)

5). Let the numerator & denominator of a fraction be n & d.


According to questions,
= n × 5 / d × 8 = 5/9
= n/d = 8 × 5 / 5 × 9
= n/d = 8/9
Answer: c)

6). According to question both statements gives some equation which have two variable
and it is not possible to find out two variable by single eg.
Answer: d)

7). According to question, l×b = ¶r2


l ×22 = 22/7 ×14 × 14
l = 22/7 ×14 × 14/22 cms
= 28 cms.
Answer: b)

8). 7!
= 7 × 6 × 5 × 4 × 3 × 2 × 1 = 5040
Answer: e)

9). Profit sharing ratio of Avinash,Bala & Chandran = 45000 : 70000 : 90,000
= 45 : 70 : 90

www.bankingpdf.com
= 9 : 14 : 18
Bala's share of profit after two year
=Rs.164000 × 14 × 41
= Rs. 4000 ×14
= Rs. 56000
Answer: a)

10).Let the total income of Mr.Sachin be 100


According to question, 100 – 20 –15
= 65 × 60/100 × 70/100
= 27.3.
27.3 = 100
8775 = 100/27.3 × 8775 = Rs. 32142.
Answer: e)

430-440 Questions :

1). A sum of Rs.13000 amounts to Rs.16307.20 in two years when compounded


annually. What is the rate of interest?
a) 10% pa
b) 11% pa
c) 12% pa
d) 13% pa
e) 14% pa

2). In how many different ways can the letters of the word MULTIPLE be arranged
so that the vowels always come together?
a) 4320

www.bankingpdf.com
b) 2160
c) 1080
d) 40320
e) 20160

3). Pipe A can fill a water tank in 8 hours, Pipe B can fill the same tank in 12 hours
and Pipe C can empty the tank in 24 hours. If all three pipes are opened together, in
how many hours will the tank be completely filled or empty?
a) 4.8 hours
b) 5 hours
c) 5.4 hours
d) 6 hours
e) 7.2 hours

4). A person sold an item for Rs. 14280 after giving 15% discount on the labelled
price and earned 19% profit on the cost price. What would have been the
percentage profit, had he not given the discount?
a) 30%
b) 34%
c) 40%
d) 42.5%
e) None of these

5). What principal will amount to Rs.5140.80 in 3 years and 3 months at 8% simple
rate of interest?
a) Rs.4050
b) Rs.4080

www.bankingpdf.com
c) Rs.4220
d) Rs.4260
e) Rs.4350

6). A man sold a cycle at a loss of 15%. Had he sold it for Rs.241.87 more he would
have gained 24%. For what value should he sell it in order to earn a profit of 30%?
a) Rs. 782
b) Rs. 792
c) Rs. 796
d) Rs. 800
e) Rs. 806

7). The average age of the whole class is 12.05 years. The average age of all girls is
12.5 years and the average age of boys is 11.75 years. If the total number of boys is
45 then what is the total number of girls in the class?
a) 20
b) 25
c) 30
d) 35
e) 40

8). A man can do a piece of work in 10 days. With the help of a boy he can do the
same piece of work in 8 days. If they get Rs. 11115 for the work then what will be
the share of boys?
a) Rs.2223
b) Rs.2784
c) Rs.3163

www.bankingpdf.com
d) Rs.3312
e) None of these

9). A person covered a certain distance by bus at the rate of 40 kmph and walked
back to the initial point at the rate of 6 kmph. The whole journey took 13 hours and
48 minutes. What distance did he walk?
a) 60km
b) 64 km
c) 70km
d) 72km
e) 80 km

10. A box contains 4 green, 5 red and 6 white balls. Three balls are drawn randomly.
What is the probability that the balls drawn are of different colours?
a) 24/91
b) 67/91
c) 21/91
d) 70/91
e) 3/13

Answers:
1).c) 2).b) 3).d) 4).c) 5).b) 6).e) 7).c) 8).a) 9).d) 10).a)

Solution:
1). A= P[1+(r/100)]n
or, 13000 [1+(r/100)]2 = 16307.20
or, [1+(r/100)]2 = 16307.20/13000 = (112/100)2
or, 1+r/100 = 112/100

www.bankingpdf.com
or, r/100 = 12/100
:. R= 12% per annum
:. Now, cost price = (14280 × 100)/119 = Rs. 12000
= % profit = [(16800 - 12000)/12000] × 100
= [(4800×100)/12000] = 40%
Answer is: c)

2). We consider all the three vowels (U, I, E) as one letter, so total number of letters = 6,
and three vowels can be arranged in 3! Ways among themselves. However, the letter ‘L’
comes twice.
:. Total number of ways = (6! × 3!)/2! = 720 × 3 = 2160
Answer is: b)

3). If all the three pipes are opened together, in one hour they will fill 1/8 + 1/12 – 1/24 =
(3+2-1)/24 = 1/6 part of tank.
So, to fill it completely it will take 6 hours.
Answer is: d)

4). Labelled price = [(14280×100)/85] = Rs. 16800


Or, 23X/120 = 69/5
Or, X= [(69×120)/(23×5)] = 72km.
Answer is: c)

5). Let the principal be Rs.X


Now, 5140.8-X=(X×3.25×8)/100
or, 514080 – 100x = 26x
Or, x = (514080/126) = Rs. 4080
Answer is: b)

www.bankingpdf.com
6). Let the cost price be Rs. X
:. Selling price = 85X/100
If profit is 24% then selling price = 124X/100
Now, (124X/100) – (85X/100) = 241.8
Or. X= (24180/39) = Rs.620
For 30% gain SP = (130/100)×620 = Rs. 806
Answer is: e)

7). Let the number of girls be x.


The number of boys is 45.
:. Total age of boys and girls in the class = (x+45) × 12.05
or, (x+45) × 12.05 = 45 × 11.75 + x × 12.5
or, 12.05x + 542.25 = 528.75 + 12.5x
or, 13.5 = 0.45x
or, x = 13.5/0.45 = 30
Answer is: c)

8). In one day boys can do = (1/8) – (1/10) = (5-4)/10 = 1/40 part
so, boys can finish the work in 40 days.
Now, Men’s share : Boy’s share = 40 : 10 = 4 : 1
Boys share = (11115 / 5) × 1 = Rs. 2223
Answer is: a)

9). Let the distance be x km.


then (x/40) + (x/6) = 13 + (48/60) = 69/5
Answer is: d)

www.bankingpdf.com
10). Total number of balls = 15
:. n(S) = 15C3 = 455
Now, we have to pick one ball of each colour.
n(E) = 4C1 × 5C1 × 6C1 = 4 ×5 ×6 = 120
P(E) = (120/455) = 24/91
Answer is: a)

440-450 Questions :

1). 117.5 x (90 x 9.5 x 7.4) ÷ 9 =?


a) 82602.5
b) 86706.5
c) 83503.5
d) 84432.3
e) 87687.2

2). (138)2 + (511)2 – (132)2 = (85)2 + ?


a) 410416
b) 255516
c) 265918
d) 345622
e) 546056

3). 119 + 784 ÷ 32 x 116 = ?


a) 2961
b) 3121
c) 2564
d) 2861

www.bankingpdf.com
e) 2941

4). (37÷44) + (78÷34) x (480÷85) = ?


a) 11.769
b) 12.756
c) 13.796
d) 12.796
e) none

5). 5005 x 87 – 22115 = ? x 320


a) 1291.625
b) 1276.635
c) 1256.735
d) 1326.685
e) 1593.625

6). [( 5 √11 + √11 ) x ( 4 √11 + 8 √11)] – (17)2 =?


a) 403
b) 655
c) 503
d) 483
e) e ) 568

7). (3333÷30) + (665 ÷ 25) + (3771 ÷ 27) = ?


a) 357.637
b) 458.558
c) 229.357
d) 277.367
e) 498.627

www.bankingpdf.com
8). 65% of 70% of 4/5 of 3750 = ?
a) 1389
b) 1465
c) 1365
d) 1265
e) 2750

9). 18.4 x 17.5 + 27.6 x 13.5 = ? ÷ 4


a) 2778.4
b) 2786.4
c) 2866.4
d) 2986.3
e) 3256.8

10). 34.8% x 950 – 33.2% of 850 = ?


a) 58.4
b) 48.4
c) 56.4
d) 68.9
e) 76.3

Answers:
1).a) 2).b) 3).a) 4).c) 5).a) 6).c) 7).d) 8).c) 9).a) 10).b)

Solution:
1). 117.5 x (90 x 9.5 x 7.4) ÷ 9 =?
Sol: 117.5 x 6327 ÷ 9 = ?
117.5 x 703 = ?
? = 82602.5

www.bankingpdf.com
Answer: a)

2). (138)2 + (511)2 – (132)2 = (85)2 + ?


Sol: 19044 + 261121 – 17424 = 7225 + ?
19044 + 261121 – 17424 – 7225 = ?
? = 255516
Answer: b)

3). 119 + 784 ÷ 32 x 116 = ?


Sol: 119 + 24.5 x 116 =?
119 + 2842 = ?
? = 2961
Answer: a)

4). (37÷44) + (78÷34) x (480÷85) = ?


Sol: 0.841 + 2.29412 x 5.64706 =?
0.841 + 12.955 =?
? = 13.796
Answer: c)

5). 5005 x 87 – 22115 = ? x 320


Sol: 435435 – 22115 = ? x 320
413320/ 320 = ?
? = 1291.625
Answer: a)

6). [( 5 √11 + √11 ) x ( 4 √11 + 8 √11)] – (17)2 =?


Sol: 792 – 289 = ?
? = 503
Answer: c)

www.bankingpdf.com
7). (3333÷30) + (665 ÷ 25) + (3771 ÷ 27) = ?
Sol: 111.1 + 26.6 + 139.667 = ?
? = 277.367
Answer: d)

8). 65% of 70% of 4/5 of 3750 = ?


Sol: 0.65 x 0.7 x 0.8 x 3750 = ?
? = 1365
Answer: c)

9). 18.4 x 17.5 + 27.6 x 13.5 = ? ÷ 4


Sol: 322 + 372.6 = ? ÷4
694.6 x 4 = ?
? = 2778.4
Answer: a)

10). 34.8% x 950 – 33.2% of 850 = ?


Sol: 0.348 x 950 – 0.332 x 850=?
330.6 – 282.2=?
?= 48.4

Answer: b)

450-460 Questions :

(Directions Q. 1-5): Answer the following questions based on the given Chart.

www.bankingpdf.com
1). What is the average per ton selling price of all these products together ?
a) 20,525
b) 18,500
c) 22,500
d) 20,500
e) 18,525

www.bankingpdf.com
2). What is the selling price of product A per ton?
a) 17,580
b) 18,576
c) 17,850
d) 18,750
e) None

3). If the expenditure incurred in production of product C per ton was 16000, what was
the percent profit earned?
a) 12.5
b) 11.11
c) 12.25
d) 11.28
e) None

4). What was the average selling price per ton of produces D and E together ?
a) 21,800
b) 26,800
c) 22,000
d) 20,500
e) 23,680

5). Which product has the highest selling price per ton ?
a) A
b) B
c) C
d) D
e) E

www.bankingpdf.com
6). Two numbers are less than third number by 30% and 37% respectively. How much
percent is the second number less than by the first
a) 20%
b) 30 %
c) 10%
d) 40%
e) 50%

7). In an examination, 34% of the students failed in mathematics and 42% failed in
English. If 20% of the students failed in both the subjects, then find the percentage of
students who passed in both the subjects.
a) 40%
b) 41%
c) 43%
d) 44%
e) 45%

8). In the new budget , the price of kerosene oil rose by 25%. By how much percent
must a person reduce his consumption so that his expenditure on it does not increase ?
a) 20%
b) 30%
c) 22%
d) 24%
e) 27%

9). The value of a machine depreciates at the rate of 10% per annum. If its present is
Rs.1,62,000.? What was the value of the machine 2 years ago ?

www.bankingpdf.com
a) 200000
b) 20000
c) 2000
d) 25000
e) None of these

10). A student multiplied a number by 3/5 instead of 5/3. What is the percentage error in
the calculation?
a) 60%
b) 62%
c) 64%
d) 63%
e) 65%

Answers:
1).d) 2).d) 3).a) 4).e) 5).b) 6).c) 7).d) 8).a) 9).a) 10).c)

Solutions:
1). Average per ton selling price = 45000000/2000 = 22,500
Answer: d)

2). Selling price of product A per ton =(45000000*0.15)/ (0.18*2000) = 18750


Answer: d)

3). Expenditure incurred in production of product C per ton was 16000


Sale of product c per ton = (45000000/0.12)/(0.15*2000) = 18,000
Profit = [(18000-16000)/16000]*100 = 12.5%

www.bankingpdf.com
Answer: a)

4). Average selling price per ton of produces D and E together =


(45000000*0.4)/(0.38*2000) = 23684.21 = 23680 (approx)
Answer: e)

5). A=(45000000*0.15)/(0.18*2000)=18,750
B=(45000000*0.17)/(0.13*2000)=29,423
C=(45000000*0.12)/(0.15*2000)=18,000
D=(45000000*0.18)/(0.17*2000)=23,823
E=(45000000*0.22)/(0.21*2000)=23,571
Answer: b)

6). Let the third number is x.


then first number = (100-30)% of x
= 70% of x = 7x/10
Second number is (63x/100)
Difference = 7x/10 - 63x/100 = 7x/10
So required percentage is, difference is what percent of first number
=> (7x/100 * 10/7x * 100 )% = 10%
Answer: c)

7). Failed in mathematics, n(A) = 34


Failed in English, n(B) = 42
n(AUB) = n(A) + n (B) – n (A∩ B )
= 34 + 42- 20 = 56
Failed in either or both subjects are 56
Percentage passed = (100−56)% = 44%

www.bankingpdf.com
Answer: d)

8). Reduction in consumption = [((R/(100+R))*100]%


? [(25/125)*100]%=20%.
Answer: a)

9). Value of the machine 2 years ago


= Rs.[162000/(1-(10/100)2)]=Rs.[162000*(10/9)*(10/9)]= Rs.200000
Answer: a)

10). Let the number be x


Then, error = (5/3)x – (3/5)x =(16/15)x
Error% = [(16x/15) /(5x/3)] * 100% = 64%
Answer: c)

460-470 Questions :

Directions (Q.Nos.1-5): In the following questions, two equations I and II are given. You
have to solve both equations.
Give Answer
a) if x > y
b) if x ≥ y
c) if x < y
d) if x ≤ y
e) if x=y or the relationship cannot be established
1). I. x2 – 11x + 24 = 0
II.2y2-9y + 9 = 0
2).I. x3 × 12 = x2 × 247

www.bankingpdf.com
II.y1/3 × 14 = 294 ÷ y2/3
3).I.12 × 4 / x4/7 – 3 × 4 / x4/7 = x10/7
II.y3 + 783 = 999
4).I.√500 x + √402 = 0
II.√360 y + (200)1/2
5).I.(18)2 + 144 ÷ 18 = x
II.(25)2 -18 × 21 = y

Directions (Q. 6-10): What approximate value should come in place of the question
mark (?) in the following questions? (you are not expected to calculate the exact value)
6).249/15 × 299/19 ÷ 14/99 = ?
a) 1850
b) 1700
c) 1750
d) 1900
e) 2000
7).175 × 28 + 275 × 28 =?
a) 11800
b) 12600
c) 12800
d) 11600
e) 12200
8).63251 × 82 = ? × 42105
a) 101
b) 123
c) 147
d) 165
e) 189
9).(7171+3854+1195)÷(892 + 214 +543)=?

www.bankingpdf.com
a) 13
b) 18
c) 3
d) 26
e) 7
10).1595 ÷ 25 × 36.5
a) 2459
b) 2329
c) 2359
d) 2429
e) 2349

Answers:
1).b) 2).c) 3).e) 4).c) 5).b) 6).c) 7).b) 8).b) 9).e) 10).b)

Solutions:
1). I.x2 – 11x+24=0
=> x2 – 8x – 3x + 24 =0
=>x(x-8)-3(x-8) = 0
=>(x-8)(x-3) = 0
X=8 or 3
II.2y2 – 9y + 9 = 0
=>2y2 – 6y – 3y + 9 =0
=>2y(y-3)-3(y-3)=0
=> (y-3)(2y-3)=0
Y= 3 or 3/2
Hence, x ≥y
Answer: b)

www.bankingpdf.com
2).I.x3 × 13 = x2 × 247
=>x3/x2 = 247/13
=> x=19
II.y1/3 × 14 =294 ÷ y2/3
=>14y1/3 = 294/y2/3
=>14y1/3 ×y2/3 = 294
=>y=294/14 = 21
Hence, x < y
Answer: c)

3).I.12 × 4/x4/7 – 3 × 4 / x4/7 = x10/7


=>48/x4/7 -12 / x4/7 = x10/7
=>48-12/x4/7 = x10/7
=>36 = x10/7 × x4/7
=>36= x2 => x = √36 = 6
II.y3 + 783 = 999
=>Y3 =999-783
=> y3=216
y=3√216 = 6
Hence,x = y or the relationship cannot be established.
Answer: e)

4).I.√500 x + √402 = 0
=> √500 x = -√402
=> x = √402/500
X=-0.9

www.bankingpdf.com
II. √360y +(200)1/2=0
=>√360y=-√200
=>y=-√200/360 =>y=-0.74
Hence, x < y
Answer: c)

5).I.(18)2 + 144 + 18 = x
=> 324+8 = x
X=332
II.(25)2 – 18 × 21=y
625 – 378 = y
Y=247
Hence, x > y
Answer: b)

6).
Answer: c)
7).
Answer: b)
8).
Answer: b)
9).
Answer: e)
10).
Answer: b)

www.bankingpdf.com
470-480 Questions :

Directions (Q. 1-5): The following pie-charts show the distribution of students of
graduate and post-graduate levels in seven different institutes in a town.
Distribution of students at graduate and post-graduate levels in seven institutes:

1). What is the total number of graduate and post-graduate level students is institute R?
a) 8320
b) 7916
c) 9116
d) 8099
e) None of these

www.bankingpdf.com
2). What is the ratio between the number of students studying at post-graduate and
graduate levels respectively from institute S?
a) 14 : 19
b) 19 : 21
c) 17 : 21
d) 19 : 14
e) None of these

3). How many students of institutes of M and S are studying at graduate level?
a) 7516
b) 8463
c) 9127
d) 9404
e) None of these

4). What is the ratio between the number of students studying at post-graduate level
from institutes S and the number of students studying at graduate level from institute Q?
a) 13 : 19
b) 21 : 13
c) 13 : 8
d) 19 : 13
e) None of these

5). Total number of students studying at post-graduate level from institutes N and P is
a) 5601
b) 5944
c) 6669

www.bankingpdf.com
d) 8372
e) None of these

Directions (Q. 6-10): Two different finance companies declare fixed annual rate of
interest on the amounts invested with them by investors. The rate of interest offered by
these companies may differ from year to year depending on the variation in the
economy of the country and the banks rate of interest. The annual rate of interest
offered by the two Companies P and Q over the years is shown by the line graph
provided below.

6). A sum of Rs. 4.75 lakhs was invested in Company Q in 1999 for one year. How
much more interest would have been earned if the sum was invested in Company P?
a) A.Rs 19,000
b) B.Rs.14, 250
c) C.Rs.11, 750
d) D.Rs. 9,500
e) E. None of these

www.bankingpdf.com
7). If two different amounts in the ratio 8:9 are invested in Companies P and Q
respectively in 2002, then the amounts received after one year as interests from
Companies P and Q are respectively in the ratio?
a) 2:3
b) 3:4
c) 6:7
d) 4:3
e) None of these

8). In 2000, a part of Rs. 30 lakhs was invested in Company P and the rest was
invested in Company Q for one year. The total interest received was Rs. 2.43 lakhs.
What was the amount invested in Company P?
a) Rs.9 lakh
b) Rs.11 lakh
c) Rs. 12 lakh
d) Rs.18 lakh
e) None of these

9). An investor invested a sum of Rs. 12 lakhs in Company P in 1998. The total
amount received after one year was re-invested in the same Company for one more
year. The total appreciation received by the investor on his investment was?
a) Rs. 2, 96,200
b) Rs. 2, 42,200
c) Rs. 2, 25,600
d) Rs. 2, 16,000
e) None of these

www.bankingpdf.com
10). An investor invested Rs. 5 lakhs in Company Q in 1996. After one year, the entire
amount along with the interest was transferred as investment to Company P in 1997 for
one year. What amount will be received from Company P, by the investor?
a) Rs. 5, 94,550
b) Rs. 5, 80,425
c) Rs. 5, 77,800
d) Rs. 5, 77,500
e) None of these

Answers:
1).d) 2).d) 3).b) 4).d) 5).c) 6).d) 7).d) 8).d) 9).c) 10).b)

Solution:
1). Required number = (17% of 27300) + (14% of 24700)
= 4641 + 3458
= 8099.
Answer: d)

2). Required ratio = (21% of 24700)/(14% of 27300) = (21 x 24700)/ (14 x 27300)
=19/14.
Answer: d)

3). Students of institute M at graduate level= 17% of 27300 = 4641.


Students of institute S at graduate level = 14% of 27300 = 3822.
Therefore Total number of students at graduate in institutes M and S = (4641 + 3822) =
8463.

www.bankingpdf.com
Answer: b)

4). Required ratio = (21% of 24700)/ (13% of 27300) = (21 x 24700)/(13 x 27300) =
19/13.
Answer: d)

5). Required number = (15% of 24700) + (12% of 24700)


= 3705 + 2964
= 6669.
Answer: c)

6). DIFFERENCE = Rs. [(10% of 4.75) - (8% of 4.75)]


= Rs. (2% of 4.75) lakhs
= Rs. 0.095 lakhs
= Rs. 9500.
Answer: d)

7). Let the amounts invested in 2002 in Companies P and Q be Rs. 8x and Rs.
9xrespectively.
Then, interest received after one year from Company P = Rs. (6% of 8x)
= Rs. (48x/100)
and interest received after one year from Company Q = Rs. (4% of 9x)
= Rs. (36x/100)
Required ratio = 4/3
Answer: d)

8). Answer: d)

www.bankingpdf.com
9). Amount received from Company P after one year (i.e., in 199) on investing Rs. 12
lakhs in it
= Rs. [12 + (8% of 12)] lakhs
= Rs. 12.96 lakhs.
Appreciation received on investment during the period of two years
= Rs. (14.256 - 12) lakhs
= Rs. 2.256 lakhs = Rs. 2, 25,600
Answer: c)

10). Amount received from Company Q after one year on investment of Rs. 5 lakhs in
the year
1996
= Rs. [5 + (6.5% of 5)] lakhs
= Rs. 5.325 lakhs.
Amount received from Company P after one year on investment of Rs. 5.325 lakhs in
the year 1997
= Rs. [5.325 + (9% of 5.325)] lakhs
= Rs. 5.80425 lakhs
= Rs. 5, 80, 425
Answer: b)

www.bankingpdf.com
480-490 Questions :
1). Sum of present ages of A and B is 60 years. 5 years hence, their ages will be in the
ratio 3:4. Find A’s present age.
a) 25 years
b) 30 years
c) 35 years
d) 40 years
e) None of these

2). The sum of ages of Sunil and his father is 46 years. 3 years back, father’s age was 4
times sunil’s age. find present age of Sunil.
a) 7 years
b) 8 years
c) 11 years
d) 12 years
e) None of these

3). Sum of present age of A, B and C is 72 years. If 4 years ago, their ages were in the
ratio 1 : 2 : 3, find A’s present age.
a) 7 years
b) 10 years
c) 12 years
d) 14 years
e) None of these

4). Present ages of A and B are 50 years and 18 years respectively. In how many years
will A be twice as old as B?
a) 14 years
b) 15 years

www.bankingpdf.com
c) 16 years
d) 18 years
e) None of these

5). The sum of ages of Mohan and his father is 35 years. When Mohan’s age will be
equal to present age of his father, then sum of their ages will be 85 years. Find present
age of the father.
a) 20 years
b) 25 years
c) 30 years
d) 32 years
e) None of these

6). The sum of present ages of a father and his son is 36 years. When the son reaches
father’s present age, the sum of their ages will be 80 years. What is the present age of
the son?
a) 4 years
b) 7 years
c) 9 years
d) 12 years
e) None of these

7). The ages of A and B are in the ratio 8 : 5. If the sum of their ages is 39 years, what
will be the ratio of their ages after 9 years?
a) 3 : 2
b) 8 : 7
c) 10: 7
d) 11 : 8
e) None of these

www.bankingpdf.com
8). A is 2 years older than B who is twice as old as C. If sum of ages of A, B and C is 37
years, find the age of A.
a) 7 years
b) 9 years
c) 14 years
d) 16 years
e) None of these

9). At the time of marriage, a man was 6 years older to his wife. 12 years after their
marriage, his age is 6/5 times the age of his wife. What was wife’s age at the time of
marriage?
a) 18 years
b) 24 years
c) 30 years
d) 36 years
e) None of these

10). A and B are 3 years and 2 years old respectively. Their father is 40 years. After
how many years, father’s age would be twice of combined age of A and B?
a) 5 years
b) 10 years
c) 15 years
d) 20 years
e) None of these

Answers:
1).a) 2).c) 3).d) 4).a) 5).c) 6).b) 7).d) 8).d) 9).a) 10).b)

www.bankingpdf.com
Solution:
1). Sum of their present ages = 60 years
sum of their ages, 5 years hence = 60 + (5 × 2) = 70 years.
after 5 years, ratio of ages of A and B will be 3 : 4.
A’s age after 5 years = (3/7) × 70 = 30 years
A’s present age = 30 – 5 = 25 years
Answer: a)

2). Sum of their ages, 3 years ago = 46 – (2 × 3) = 40 years


sunil’s age, 3 years ago = 1/5 × 40 = 8 years
sunil’s present age = 8 + 3 = 11 years
Answer: c)

3). Sum of present ages of A, B and C is 72 years.


sum of their ages (4 years ago) = 72 – (3 × 4) = 60 years
4 years ago, ratio of ages of A, B and C was 1 : 2 :3.
A’s age, 4 years ago = (1/6) ×60 = 10 years
A’s present age = 10 + 4 = 14 years
Answer: d)

4). Let the required time be x years.


then (50+x) = 2 (18 + x) = 50 + x = 36 + 2x  x = 14 years.
Alternative method : Double of B’s age = 2 × 18 = 36 years
difference between A’s present age and double of B’s age = 50 – 36= 14 years.
A will be twice of B’s age after 14 years.
Answer: a)

www.bankingpdf.com
5). Sum of present ages = 35 years
sum of their ages will be 85 years after (85-35) / 2 = 25 years from now.
the father is 25 years older than his son.
Present age of father = (35 + 25)/2 = 30 years.
Answer: c)

6). Difference in two sums = 80- 36 = 44


Age of both are increased by 44/2 = 22 years
Difference in ages of father and son = 22 years
sum of present ages of father and son = 36 years
Son’s present age = 1/2 × (36 - 22) = 7 years
Answer: b)

7). Sum of ratios = 8 +5 = 13.


Sum of ages = 39 years
13 ratio = 39 years  1 ratio = 3 years
9 years = 3 ratio
new ratio = (8+3) : (5+3) = 11 : 8
Answer: d)

8). Let the age of C be x years. Then age of B = 2x and age of A = 2x + 2


sum of ages of A, B and C = (2x + 2) + (2x) + x = 37
(5x + 2) = 37  x = 7
Age of A = 2x + 2 = 16 years
Answer: d)

www.bankingpdf.com
9). ratio of their ages, 12 years hence = 6 : 5  Difference in ages = 1 ratio = 6 years
wife’s age (12 years hence) = 5 ratio = 5 × 6 years = 30 years
wife’s age at the time of marriage = 30 – 12 = 18 years
Answer: a)

10). Father’s age = 40 years. Children’s combined age = 3 + 2= 5 years


Double of their ages = 2 × 5 = 10 years
Difference between father’s age and combined age = 40 – 10 = 30 years
required time = 1/ (2 + 1) × 30 = 10 years
Answer: b)

490-500 Questions :

1).The cost of 3 horses is same as the cost of 5 cows. If total cost of 4 horses and 6
rows is Rs.1900.Find the cost of one horse?
a) Rs.50
b) Rs.150
c) Rs.200
d) Rs.250
e) None of these

2). 6 pens and 14 exercise books cost Rs.162. 5pens and 8 exercise books
cost Rs.102. The ratio of cost of 1 pen to that of 1 exercise book is :
a) 10 : 3
b) 2 : 3
c) 3 : 2
d) 5 : 6
e) None of these

www.bankingpdf.com
3). The price of 10 chairs is equal to that of 4 tables. The price of 15 chairs and 2 tables
together is Rs.4000. The total price of 12 chairs and 3 tables is:
a) Rs.3750
b) Rs.3840
c) Rs.3500
d) Rs.3900
e) None of these

4). If 6x2-17xy+12y2 = 0 ,find x : y


a) 3 : 2
b) 4 : 3
c) Both
d) 1 : 2
e) None of these

5). A man plays 100 games with cards. He gets Rs.5 if he wins and pays Rs.2 if he
loses. If he wins Rs.3 on the whole, in how many games did he win?
a) 25
b) 28
c) 29
d) 32
e) None of these

6). There are 200 questions on a 3 hour examination. Among 200 questions,50 are from
Maths, 100 are in G.K, and 50 are in Science. He spent twice as much as time on each
Mathematics question as for each other question. How many minutes did he spend on
Mathematics question?

www.bankingpdf.com
a) 100
b) 60
c) 36
d) 72
e) None of these

7). A milkman mixes 20 litres of water with 80 litres of milk. After selling one-fourth of
this mixture, he adds water to replenish the quantity he has sold, what is the current
proportion of water to milk?
a) 1 : 3
b) 2 : 3
c) 3 : 4
d) 4 : 3
e) None of these

8). When a ball bounces, it rises to 2/3 of the height from which it fell. If the ball is
dropped from a height of 36m, how high will it rise at the third bounce?
a) 10 2/3 m
b) 13 2/3 m
c) 12 1/3 m
d) 12 2/3m
e) None of these

9). A vessel contains 100 litres of milk. From this, 10 litres of milk was taken out and
replaced by water. The process was repeated further two times. How much milk is now
contained by the vessel?
a) 72.9 litres
b) 60.4 litres
c) 71.6 litres

www.bankingpdf.com
d) 81 litres
e) None of these

10). From a vessel containing 100 litres of wine,10 litres are drawn out and an equal
amount of water is added. From the mixture,10 litres is again drawn out and same
quantity of water is added. What is the final ratio of wine and water?
a) 91 : 9
b) 81 : 19
c) 80 : 20
d) 90 : 10
e) None of these

Answers:
1).d) 2).b) 3).d) 4).c) 5).c) 6).d) 7).b) 8).a) 9).a) 10).b)

Solution:
1). Ratio of cost of horse and cow = 1/3 : 1/5 =5 : 3
Let cost of horse and cow are 5x and 3x respectively:
Then 4 × 5x +6 × 3x = Rs.1900 =>38x = 1900 => x = 1900/38 = Rs.50
Cost of one horse = 5 × Rs.50 = Rs.250
Answer: d)

2). Let cost of 1 pen be Rs.x and that of 1 exercise book be Rs.y.
Then 6x +14 y = 162 =>3x + 7y =81……..(1)
5x + 8y =102…………………………………………..(2)
(1)× 5 – (2) × 3 =>11y=99=>y=9.Putting y=9 in equation (1),we get : x = 6
x : y =6 : 9 =2 : 3
Answer: b)

www.bankingpdf.com
3).Cost of 4 tables =Cost of 10 chairs => Cost of 1 table =10 ÷ 4=2.5 chairs
=>15 chairs + 2 tables = Rs.4000 => 15 chairs + 2 × 2.5 chairs = Rs.4000
=>20 chairs =Rs.4000 => 1 chair = Rs.4000 ÷ 20 = Rs.200
=>cost of 1 tables =2.56 × Rs.200 = Rs.500
Cost of 12 chairs + cost of 3 tables =12 × Rs.200 + 3 × Rs.500 = Rs.2400 +
Rs.1500=Rs.3900
Answer: d)

4). 6x2-17xy +12y2 = 0 =>6x2-8xy-9xy + 12y2 = 0 => 2x(3x-4y) -3y (3x-4y)=0


=>(2x – 3y)(3x-4y) = 0 =>2x-3y = 0 or 3x-4y = 0 => 2x = 3y or 3x = 4y
=>x : y =3 : 2 or 4 : 3
Answer: c)

5). Suppose he wins the first game and loses the second. Then his gain =Rs 5- Rs 2 =
Rs 3 out of remaining 98 games, amount won =amount lost =>Games won : Games lost
2:5
In last 98 Games, he wins =2/7 × 98 =28 games =>Total games he won =28 + 1 =29
Answer: c)

6). Ratio of questions on Mathematics and others =50 : 150 =1 : 3


Ratio of time spent on Mathematics and other questions = (1 × 2) : (3 × 1) = 2 : 3
Time spent on Mathematics =2/5 × 180 minutes =72 minutes
Answer: d)

7). Milk remaining after selling 1/4 of mixture = 80 ×(1- 1/4)=60 litres
=>water =100-60 = 40 litres => Required ratio = 40 : 60 =2 : 3

www.bankingpdf.com
Answer: b)

8). Height at third bounce =36 × (2/3)3 = 10 2/3 metre


Answer: a)

9). Milk after three replacement =100 × (90/100)3 = 72.9 litres


Answer: a)

10).Quantity of wine after two replacements = 100 l × (90/100)2 =81 l


Required ratio = 81 : ( 100 – 81 )= 81 : 19
Answer: b)

500-510 Questions :
Directions (Q.1-5): What should come in place of question mark(?) in the following
questions?
1). √24 + √96 + √216 + √384 =5√2 × ?
a) 6√3
b) 4√3
c) 2√3
d) 5√3
e) 3√3

2).189 2/9 +189 3/9+189 5/9+189 6/9+189 7/9 =?


a) 1138
b) 1037
c) 1237
d) 1238
e) 1137

www.bankingpdf.com
3). √1369 ×√1444 +√?=1420
a) 14
b) 196
c) 198
d) 194
e) 16

4). √6889+√3721+√1024-√2401=?
a) 129
b) 128
c) 127
d) 124
e) 123

5).3001 × 99 ÷ 11 -6001 × 8 + 401 × 11 + (303)2 = ?


a) 76125
b) 78129
c) 75000
d) 75221
e) 74532

Directions (Q.6-10): What approximate value should come in place of question mark (?)
in the following questions?
6).38% of 3976 +(32)2 – 13% of 8271 + 7/6 × 3400 =?
a) 5427
b) 5325
c) 5537

www.bankingpdf.com
d) 5612
e) 5554

7).987.67 × 123.35 ÷ 9 = ?
a) 13411
b) 13621
c) 13489
d) 13551
e) 13721

8). √80 × (35/6) × (21)2 +343 =?


a) 21125
b) 22981
c) 20781
d) 23159
e) 21230

9).4×(√3 + √4)2 + 6(√5 +√6)2 -3(√2 + √3)2 =?


a) 167
b) 123
c) 157
d) 153
e) 149

10). (331/30)+( 661/60) +(704/11) -35.013 + 36.026 =?


a) 69
b) 67
c) 83

www.bankingpdf.com
d) 89
e) 85

Answers:
1).b) 2).e) 3).b) 4).c) 5).d) 6).a) 7).c) 8).b) 9).c) 10).e)

Solution:
1). √24 + √96 + √216 + √384 =5√2 × ?
Or, √(6 × 4) + √(6 × 16) + √(6 × 36) + √(6 × 64) = 5√2 × ?
Or, 2√6 + 4√6 + 6√6 + 8√6 =5√2 × ?
Or, √6(2+4+6+8) = 5√2 × ?
? = 20√6 / 5√2 = 4√3
Answer: b)

2). ? = 189 2/9 + 189 3/9 +189 4/9 +189 5/9 + 189 6/9 + 189 7/9
=(189 × 6) +(2/9 + 3/9 +4/9+5/9+6/9+7/9 )
=1134 + (2+3+4+5+6+7)/9 =113 + (27/9)
=1134 + 3=1137
Answer: e)

3). √1369 ×√1444 - √? =1420


Or, √(37)2 ×√(38)2×√? = 1420
Or,37 × 38 ×√? = 1420
Or,1406 + √? =1420
Or, √? = 1420 -1406 =14
? = 196
Answer: b)

www.bankingpdf.com
4). √6889 + √3721 +√1024 - √2401 = ?
Or,?= √(83)2 + √(61)2 + √(32)2 - √(49)2
=83+61+32 – 49 =127
Answer: c)

5). ? =3001× 99 ÷ 11 – 6001 × 8 + 401 × 11 +(303)2


=3001 × 9 – 6001 × 8 + 401 × 11 + (303)2
=27009 -48008 + 4411 + 91809 = 75221
Answer: d)

6). ?=3976 × (38/100) + 1024 – 8271 × (13 / 100) + (7/6) × 3400


=1510.88 + 1024-1075.23 + 3966.66
=5426.31 ≈ 5427
Answer: a)

7). ? = (987 × 123) / 9 = 13489


Answer: c)

8). ?= √80 × (35/6) × (21)2 + 343


=√(16×5) × (35/6) × 441 + 343
=4√5 × (35/6) × 441 + 343
=2√5 ×35 ×147 + 343
=2 ×2.2 ×35 × 147 +343 = 22981
Answer: b)

9). ? =4(3+4+2√12) + 6(5+6+2√30)-3(2+3+2√6)

www.bankingpdf.com
=4(7+2√12)+6(11+2√30)-3(5+2√6)
=28 + 8√12 + 66 + 12√30 – 15 - 6√6
=(28+66-15) + (8√12+12√30-6√6)
=79 + (8√4 × 3 + 12√30 - 6√6)
=79+16√3 + 12√30 - 6√6
=79 + 16 ×1.7 + 12 × 5.4 – 6 ×2.4
=79+27.2 + 64.8-14.4 = 156.6 = 157
Answer: c)

10). ?=331/30 + 661/60 + 704/11 – 35.013 + 36.026


=11 + 11 + 64 -1.013 =86-1 ≈ 85
Answer: e)

510-520 Questions :

Direction (Q.1-5): The following line graph shows the percentage increase in the
population of two cities A and B over the period 2009 to 2013.

The table shows the population of these cities at the beginning of 2009.

www.bankingpdf.com
City Population (in lakh)

A 12

B 10

1).What is the population of city A at the beginning of the year 2011?


a) 14.96 lakh
b) 15.18 lakh
c) 15.72 lakh
d) 16.04 lakh
e) 16.24 lakh

2).What is the difference between the population of city A and City B at the end of the
year 2011?
a) 53400
b) 54600
c) 55300
d) 56100
e) 57500

3).What is the ratio of the population of City A to that of City B at the end of 2009?
a) 1 : 2
b) 6 : 5
c) 8 : 5
d) 11 : 10
e) None of these

www.bankingpdf.com
4).What will be the population of City B at the end of the year 2013?
a) 17.325 lakh
b) 18.464 lakh
c) 19.0575 lakh
d) 20.040 lakh
e) None of these

5).The population of City B at the end of the year 2011 is approximately what per cent of
the population of City A at the beginning of the year 2009?
a) 72.7%
b) 87.5%
c) 112.5%
d) 125%
e) 137.5%

Directions (Q.6-10):In each of these questions, two equations(I) and (II) are given. you
have to solve both the equations and give answer
a) if x > y
b) if x≥y
c) if x < y
d) if x ≤ y
e) if x = y or relation cannot be established between ‘x’ and ‘y’.
6).I. x2 – x – 56 = 0
II.y=3√729
7).I.x2 + 7x + 10=0
II.y2 + 13y + 42 =0
8).I.14x2 - 55x + 50 = 0
II.2y2 – 13y + 20 = 0
9).I.7x + 8y = 5
II.6x-5y = 28

www.bankingpdf.com
10).I.5x2 – 52x + 96 =0
II.5y2 + 3y-36 = 0

Answers:
1).b) 2).d) 3).d) 4).c) 5).e) 6).c) 7).a) 8).d) 9).a) 10).b)

Solution:
1). Population of City A at the beginning of the year 2011
=12 × 110/100 × 115 / 100 =15.18 lakh
Answer: b)

2). Population of City A at the end of 2011


=12 × 110/100 × 115/100 ×105/100 =15.939 lakh
Population of City B at the end of 2011
=10 × 120/100 × 125/100 × 105/100 = 16.5 lakh
Difference = 16.5 – 15.939 = 0.561 lakh = 56100
Answer: d)

3). Ratio = [12 × (110/100)] /[10×1(20/100)] = (12× 11) / (10 × 12) = 11/10 = 11 : 10
Answer: d)

4). Population of City B at the end of 2013


=10 × (120/100) × (125/100) × (110/100) × (105/100) × (110/100) =19.0575 lakh
Answer: c)

5). Population of City B at the end of the year =10 × 1.20 × 1.25 ×1.1 = 16.5 lakh
Population of City A at the beginning of the year 2009 = 12 lakh

www.bankingpdf.com
Reqd % = (16.5 /12) × 100 = 137.5 %
Answer: e)

6). I . x2 – x – 56 = 0
Or,x2 + 7x – 8x-56 = 0
Or, x(x+7) -8(x+7) =0
Or,(x-8)(x+7)=0
Or, x= 8,-7
II. y=3√729
Or, y=9
Hence, x < y
Answer: c)

7). I. x2 + 7x + 10 = 0
Or,x2 + 2x +5x + 10 = 0
Or, x(x+2)+ 5(x+2)=0
Or,(x+5)(x+2) = 0
or,x= -2,-5
II.y2 + 13y + 42 =0
Or,y2 + 6y+7y+42 = 0
Or, y(y+6)+7(y+6) = 0
Or,(y+6)(y+7)=0
Y = -6,-7
Hence, x > y
Answer: a)

8). I. 14x2 – 55x+50 = 0


Or,14x2 -35x-20x+50 = 0
Or,7x(2x-5)-10(2x-5)=0

www.bankingpdf.com
Or,(7x-10)(2x-5) = 0
X=10/7,5/2
II.2y2-13y + 20 = 0
Or,2y2-8y-5y+20 =0
Or,2y(y-4)-5(y-4) = 0
Or,(y-4)(2y-5) = 0
Or, y=4,5/2
Hence, x ≤ y
Answer: d)

9). I. 7x + 8y = 5 …..(i)
II.6x - 5y = 28 …..(ii)
Now, eqn(i) × 5 + eqn (ii) × 8
35x + 40 y =25
48x – 40y = 224
Simplify above 2 eqns we get 83x = 249
X=3 and y=-2
Hence,x > y
Answer: a)

10). I. 5x2-52x + 96 = 0
Or,5x2-40x-12x + 96 =0
Or,5x(x-8)-12(x-8)=0
Or,(x-8)(5x-12)=0
Or,x=8,12/5
II. 5y2 + 3y -36=0
Or,5y2 +15y -12y -36 =0
Or,5y(y + 3)-12(y+3) =0
Or,(5y-12)(y+3) = 0

www.bankingpdf.com
Or,y=-3,12/5
Hence,x ≥ y
Answer: b)

520-530 Questions :

1). A man sells two houses at the rate of Rs.1.995 lakh each. On one he gains 5% and
on the other he loses 5%. His gain or loss per cent in the whole transaction is
a) 0.25% loss
b) 0.25% gain
c) 2.5% loss
d) 2.5% gain
e) 1.25% loss

2). A reduction of 20% in the price of sugar enables a person to get 7.8 kg more sugar
for Rs.3.90. The original price of sugar per kg is
a) Rs. 10
b) Rs.11.50
c) Rs. 12.5
d) Rs. 15 ,
e) Rs. 10.5

3). In a college election a candidate who got 35% of the total votes polled was defeated
by his rival by 270 votes. Assuming that there were only two candidates in the election,
the total number of votes polled was
a) 10000
b) 8000

www.bankingpdf.com
c) 9000
d) 12000
e) 12500

4). In a bag there arc coins of 25 paise and 10 paise in the ratio of 6 : 17. If the bag
contains Rs. 16 then the number of 10-paise coins is
a) 30
b) 40
c) 70
d) 85
e) 65

5). If six men working 8 hours a day earn Rs.840 per week, then 9 men working 6 hours
per day will earn per week
a) Rs. 940
b) Rs. 945
c) Rs. 1620
d) Rs. 1680
e) Rs. 1245

6). 60 kg of an alloy A is mixed with 100 kg of alloy B. If alloy A has lead and tin in the
ratio of 3 : 2 and alloy B has tin and copper in the ratio of I : 4 the amount of tin in the
new alloy is
a) 36 kg
b) 44 kg
c) 80 kg
d) 53 kg

www.bankingpdf.com
e) 45 kg

7). A and B enter into a partnership. A contributes Rs. 16,000 and B Rs. 12,000. At the
end of 6 months C joins them with a capital of Rs. 20,000. In what ratio should they
divide their annual profit?
a) 8:6:5
b) 4 :3 :5
c) 5:6:8
d) 5:3:4
e) 5:7:8

8). Three cubes of side 3 cm. 4 cm, 5 cm respectively are melted to form a new cube.
The side of the new cube will be
a) 6cm
b) 6.5 cm
c) 70cm
d) 5 cm
e) 4.5 cm

9). The area of a circle is 154 sq cm. Its circumference is


a) 44cm
b) 88cm
c) 154cm
d) 176cm
e) 164 cm

www.bankingpdf.com
10). Two pipes fills a tank in 12 hours and 15 hours respectively while a third pipe
empties the tank in 18 hours. !f all three pipes operate simultaneously, in what time will
the tank be full?
a) 10 hours
b) 10 10/17 hours
c) 11 hours
d) 9 hours
e) 12 1/17 hours

Answers:

1).a) 2).c) 3).c) 4).d) 5).b) 6).b) 7).a) 8).a) 9).a) 10).b)

530-540 Questions :

Directions (Q. 1-5): Study the following graph and table carefully and answer the
questions given below.
Time taken to travel (in hours) by six trains on three different days

Distance covered (in kilometres) by six trains each day

www.bankingpdf.com
Train Day 1 Day 2 Day 3

P 980 704 1127

Q 720 1012 1120

R 1044 1008 1254

S 1026 855 741

T 1140 1144 918

U 871 1224 1518

1). Which of the following trains travelled at the same speed on all three days?
a) S
b) P
c) R
d) T
e) U

2). What was the difference between the speed of Train P on Day 1 and the speed of
Train S on Day 2?
a) 7 km/hr
b) 9 km/hr
c) 7.5 km/hr
d) 8.5 km/hr
e) 8 km/h

www.bankingpdf.com
3). What was the speed of Train R on Day 2 in terms of metre per second?
a) 17.80 m/s
b) 17.5 m/s
c) 18 m/s
d) 17.88 m/s
e) 18.8 m/s

4). The distance travelled by Train U on Day 3 was approximately what per cent of the
distance travelled by it on Day 1?
a) 95%
b) 92%
c) 91%
d) 98%
e) 96%

5). What is the ratio of the speeds of Train T to Train U on Day 2?


a) 13 : 17
b) 13 : 15
c) 17 : 15
d) 19 : 17
e) None of these

Directions (Q. 6-10): Study the following information and answer the questions that
follow.
The premises of an institute are to be renovated. Only the floor is to be renovated either
with marble or with wood. All rooms, halls and pantry are rectangular. The area to be
renovated comprises a hall measuring 33m by 39m. The director's room measures 13m
by 12m and the pantry measures 14m by 12m. A record keeping-cum-server room
measures 23m by 13m and the accounts room measures 12m by 23m. The total area of

www.bankingpdf.com
the institute is 2500 square metres. The cost of wooden flooring is 2170 per square
metre and the cost of marble flooring is 2190 per square metre. The accounts room, the
record keeping-cum-server room, and the pantry are be floored with marble. The
director's room and the hall are to I), floored with wood.

6). What is the ratio of the total cost of wooden flooring to the total cost of marble
flooring?
a) 1443: 735
b) 8177: 4655
c) 1443 : 4655
d) 24531 : 14117
e) 9177 : 4655

7). If four walls and ceiling of the room (the height of the room is 15 metres) are to be
painted at the cost of Rs.190 per square metre, how much will be the total cost of
renovation of the director's room, including the cost of flooring?
a) Rs.198660
b) Rs.178680
c) Rs.198880
d) Rs.222876
e) Rs.188680

8). If the remaining area of the institute is to be carpeted at the rate of Rs.210 per
square metre, by how much will the cost of renovation of institute premises increase?
a) Rs.75000
b) Rs.72840
c) Rs.65940
d) Rs.75940
e) Rs.64940

www.bankingpdf.com
9). What is the percentage area of the institute that is not to be renovated?
a) 16.44%
b) 13.56%
c) 14.55%
d) 12.56%
e) 11.44%

10). What is the total cost of renovation of the hall and the accounts room?
a) Rs.287700
b) Rs.277230
c) Rs.266600
d) Rs.298870
e) Rs.271230

Answers:
1).a) 2).e) 3).b) 4).d) 5).a) 6).d) 7).a) 8).c) 9).d) 10).e)

Solution:
Speed of train P
On Day 1 – (980/20) = 49km/h
On Day 2 – (704/22) = 32 km/h
On Day 3 – (1127/23) = 49 km/h
Similarly, for train Q, the speed
On Day 1 – (720/15) = 48 km/h
On Day 2 – (1012/22) = 46 km/h
On Day 3 – (1120/20) = 56 km/h
Similarly, for train R, the speed

www.bankingpdf.com
On Day 1 – (1044/18) = 58 km/h
On Day 2 – (1008/16) = 63 km/h
On Day 3 – (1254/22) = 57 km/h
Similarly, for train S, the speed
On Day 1 – (1026/18) = 57 km/h
On Day 2 – (855/15) = 57 km/h
On Day 3 – (741/13) = 57 km/h
Similarly, for train T, the speed
On Day 1 – (1140/20) = 57 km/h
On Day 2 – (1144/22) = 52 km/h
On Day 3 – (918/17) = 54 km/h
Similarly, for train U, the speed
On Day 1 – (871/13) = 67 km/h
On Day 2 – (1224/18) = 68 km/h
On Day 3 – (1518/23) = 66 km/h
1). Train S has the same speed on all three days.
Answer: a)

2). The speed of train P on 1st day = 49 km/h


The speed of train S on 2nd day = 57 km/h
Difference = 57 - 49 = 8 km/hr
Answer: e)

3). The speed of train Ron 2nd day = 63 km/h


Speed in metre per second = 63 x (5/18) = 17.5 m /s
Answer: b)

4). On the 3rd day the speed of Train U = 66 km/h


On 1st day the speed of Train U = 67 km/h

www.bankingpdf.com
Reqd% = (66/67) × 100 = 98.5 = 98%
Answer: d)

5). Speed of Train Ton Day 2 = 52 km/h


Speed of Train U on Day 2 = 68 km/h
Reqd ratio = 52/68 = 13 : 17
Answer: a)

6). Total flooring area with wood = 1287 + 156 = 1443 sq m


Cost of flooring area with wood =1443 x 170 = Rs. 245310
Total flooring area with marble = accounts room + record keeping cum-server room +
pantry
Cost of flooring area with marble =743 x 190 = Rs.141170
Reqd ratio = (245310/141170) = (24531/14117) = 24531 : 11417
Answer: d)

7). Area of wall = 2(15 x 13+ 15 x 12)= 750


Area of director's room = 13 x 12 = 156
Cost of painting = 190 x (750 + 156) = Rs.172140
Cost of flooring= 170 x 156 = Rs.26520
Total cost = 172410 + 26520 = Rs. 198660
Answer: a)

8).Total area of the institute = 2500 sq m


Remaining area = 2500 -2186 =314 sq m
Cost of renovation of the remaining area =314 x210 = Rs. 65940
Answer: c)

www.bankingpdf.com
9). Area not to be renovated = 314 sq m
Reqd % = (3.14/2500)x100 = 12.56
Answer: d)

10). Cost of renovation of the hall =1287 x 170 = 218790


Cost of renovating the accounts room = 276 x 190=52440
Total cost = 218790 +52440 =Rs. 271230
Answer: e)

540-550 Questions :

Directions (Q. 1-5): Find the missing number in the following number series:

1). 280 600 ? 2504 5036


a) 1364
b) 1236
c) 1362
d) 1361
e) None of these

2). 18 119 708 ? 14136 42405


a) 3434
b) 3535
c) 3636
d) 3737

www.bankingpdf.com
e) None of these

3). 5 7.5 11.25 17.5 ? 50


a) 28.75
b) 26.50
c) 27.50
d) 28.25
e) None of these

4). 264 262 271 243 ?


a) 302
b) 304
c) 306
d) 308
e) None of these

5). 1.5 2.5 7 24 100 ?


a) 500
b) 504
c) 505
d) 503
e) None of these

6). The average temperature of Monday, Tuesday, Wednesday and Thursday was 38∘
and that of Tuesday, Wednesday, Thursday and Friday was 40∘. If the temperature on
Monday was 30∘, the temperature of Friday was :

www.bankingpdf.com
a) 40∘
b) 39∘
c) 38∘
d) 30∘
e) 42∘

7). The average of weight of three men A,B and C is 84 kg. Another man D joins the
group and the average now becomes 80 kg. If another man E, whose weight is 3
kg.more than that of D, replaces A, then the average weight of B,C,D and E becomes
79 kg. The weight of A is :
a) 70 kg
b) 72 kg
c) 75 kg
d) 80 kg
e) 78 kg

8). The average age of 30 students in a class is 12 years. The average age of a group
of 5 of the students is 10 years and that of another group of 5 of them is 14 years. The
average of the remaining students is :
a) 8 years
b) 10 years
c) 12 years
d) 14 years
e) 15 years

www.bankingpdf.com
9). A train overtakes two persons who are walking in the same direction in which the
train is going, at the rate of 2 kmph and 4 kmph and passes them completely in 9 and
10 seconds respectively. The length of the train is :
a) 72 metres
b) 54 metres
c) 55 metres
d) 45 metres
e) 50 meters

10). A train travelling at 36 kmph completely crosses another train having half its length
and travelling in the opposite direction at 54 kmph, in 12 seconds. If it also passes a
railway platform in 112 minutes, the length of the platform is :
a) 560 metres
b) 620 metres
c) 700 meres
d) 750 metres
e) 720 meters

Answers:
1).b) 2).b) 3).a) 4).d) 5).c) 6).c) 7).c) 8).c) 9).e) 10). c)

Solution:
1). 118 – 35 = 83
280 – 118 = 162
=>162 – 83 = 79
280 – 118 = 162
600 – 280 = 320

www.bankingpdf.com
=>320 – 162 = 158
Answer: b)

2). 18*7 -7 = 119


119*6 -6 = 708
708*5 -5 = 3535
Answer: b)

3). 7.5 – 5 = 2.5


11.25 – 7.5 = 3.75
=> 3.75 – 2.5 = 1.25
11.25 – 7.5 = 3.75
17.5 – 11.25 = 6.25
=> 6.25 – 3.75 = 2.5
Answer: a)

4). 264 – 1³ + 1 = 262


262 + 2³ + 1 = 271
271 – 3³ + 1 = 243
Answer: d)

5). 1.5 * 1 + 1 = 2.5


2.5 * 2 + 2 = 7
7 * 3 + 3 = 24
Answer: c)

www.bankingpdf.com
6).
M+T+W+Th=(4×38)=152
Monday temparature is 30. So T+W+Th=(152-30)=122
T+W+Th+F=(4×40)=160
F = (160-122)= 38∘
Answer: c)

7).
A+B+C = 3×84=252
A+B+C+D= (4×80)=320
D = (320-252)=68 and E = (68+3)=71
Now, B+C+D+E = (4×79)=316
B+C+D=(316-71)=245 kg
So, A = (320-245)=75 kg
Answer: c)

8).
Let it be x . Then :
5×10+5×14+20×x=30×12
20x=360-120 or 20x=240 or x = 12
Answer: c)

9).
Answer: e)

www.bankingpdf.com
10).
Answer: c)

550-560 Questions :

1). The distance between two cities A and B is 330km. A train starts from A at 8 (a)m.
and travels towards B at 60 km/hr. Another train starts from B at 9 (a)m. and travels
towards A at 75 km/hr. At what time do they meet?
a) 10 am.
b) 10 : 30 am.
c) 11 am.
d) 11 : 30 am.
e) None of these

2). Busses start from a bus terminal with a speed of 20 km/hr at intervals of 10 minutes.
What is the speed of a man coming from the opposite direction towards the bus terminal
if he meets the buses at intervals of 8 minutes?
a) 3 km/hr
b) 4 km/hr
c) 5 km/hr
d) 7 km/hr
e) None of these

3). In covering a distance of 30 km, Abhay takes 2 hours more than Sameer. If Abhay
doubles his speed, then he would take 1 hour less than Sameer. Abhay's speed is?

www.bankingpdf.com
a) 5 kmph
b) 6 kmph
c) 6.25 kmph
d) 7.5 kmph
e) None of these

4). It takes eight hours for a 600 km journey, if 120 km is done by train and the rest by
car. It takes 20 minutes more, if 200 km is done by train and the rest by car. The ratio of
the speed of the train to that of the cars is?
a) 2 : 3
b) 3: 2
c) 3 : 4
d) 4 : 3
e) None of these

5). If a train runs at 40 km/hour, it reaches its destination late by 11 minutes. But if it
runs at 50 km/hour, it is late by 5 minutes only. The correct time (in minutes) for the train
to complete the journey is?
a) 13
b) 15
c) 19
d) 21
e) None of these

Directions (Q. 6-10): what approximate value should come in place of question mark (?)
in the following question?
6). (32.3)2 ÷ 4 + √361 = ?2 + 50
a) 15
b) 13

www.bankingpdf.com
c) 11
d) 17
e) None of these

7). 256×256 + 173×173 = ?


a) 96432
b) 94465
c) 95465
d) 90510
e) None of these

8). √ (191×7+231-839) = ?
a) 27
b) 33
c) 23
d) 37
e) None of these

9). [3/2 + 2(1/5) – 7/10] of ? = 1098


a) 463
b) 326
c) 276
d) 366
e) None of these

10). (1675÷5) + (5328÷12) * (8430÷15) = ?2 – 235


a) 152
b) 138

www.bankingpdf.com
c) 158
d) 142
e) None of these

Answers:
1).c) 2).c) 3).a) 4).c) 5).c) 6).a) 7).c) 8).a) 9).d) 10). c)

SOLUTION:
1). Distance travelled by first train in one hour
= 60 x 1 = 60 km
Therefore, distance between two train at 9 a.m.
= 330 – 60 = 270 km
Now, Relative speed of two trains = 60 + 75 = 135 km/hr
Time of meeting of two trains =270/135=2 hrs.
Therefore, both the trains will meet at 9 + 2 = 11 A.M.
Answer: c)

2). Distance covered in 10 minutes at 20 kmph = distance covered in 8 minutes at


(20+x) kmph
20× 10/60=8/60(20+x)
200 = 160 + 8x
8x = 40
x=40/8=5 kmph
Answer: c)

3). Let Abhay's speed be x km/hr.


Then, 30/x-30/2x= 3
6x = 30

www.bankingpdf.com
x = 5 km/hr.
Answer: a)

4). Let the speed of the train be x km/hr and that of the car be y km/hr.
Then, 120/x+480/y= 8 1/x+4/y=1/15 ....(i)
And, 200/x+400/y=25/3 1/x+2/y=1/24 ....(ii)
Solving (i) and (ii), we get: x = 60 and y = 80.
Ratio of speeds = 60 : 80 = 3 : 4.
Answer: c)

5). If the distance be x km, then x/40-x/50=6/60


x/4-x/5=1
x=20 km.
Required time = (20/40) hour – 11 minutes
= (1/2×60-11) minutes = 19 minutes
Answer: c)

6). 32*32 = 1024


1024/4 + 19 = 256+19= 275
Answer: a)

7). a2+b2 = [(a+b)2 +(a-b)2 ] / 2


? = 184041 + 6889
? = 95465
Answer: c)

8). 1337+231-839 = 729

www.bankingpdf.com
27*27 = 729
Answer: a)

9). [15+22-7/10] * x = 1098


X = 1098/3 = 366
Answer: d)

10). 335+444*56 = 24864+335 = 25199 – 235 = 24964


158*158 = 24964
Answer: c)

560-570 Questions :

Directions (Q. 1-10): What should come in place of the questions mark (?) in the
following questions?
1). 117.5 x (90 x 9.5 x 7.4) ÷ 9 =?
a) 82602.5
b) 86706.5
c) 83503.5
d) 84432.3
e) 87687.2

2). (138)2 + (511)2 – (132)2 = (85)2 + ?


a) 410416
b) 255516
c) 265918
d) 345622
e) 546056

www.bankingpdf.com
3). 119 + 784 ÷ 32 x 116 = ?
a) 2961
b) 3121
c) 2564
d) 2861
e) 2941

4). (37÷44) + (78÷34) x (480÷85) = ?


a) 11.769
b) 12.756
c) 13.796
d) 12.796
e) none

5). 19/14 x 12/72 x 18/29 x 767 = ? (Take 2 digits after the decimal point)
a) 110.69
b) 118.78
c) 111.56
d) None
e) 108.98

6). 49/55 of 365/517 of 5111 = ? (Take 4 digits after the decimal point)
a) 3015.6001
b) 3148.8123
c) 3214.2379
d) 3348.6179
e) 3358.6279

www.bankingpdf.com
7). 5005 x 87 – 22115 = ? x 320
a) 1291.625
b) 1276.635
c) 1256.735
d) 1326.685
e) 1593.625

8). [( 5 √11 + √11 ) x ( 4 √11 + 8 √11)] – (17)2 =?


a) 403
b) 655
c) 503
d) 483
e) e ) 568

9). (3333÷30) + (665 ÷ 25) + (3771 ÷ 27) = ?


a) 357.637
b) 458.558
c) 229.357
d) 277.367
e) 498.627

10). (37) 0.75 x [(7056) 0.25] ÷ (64) – 2 = (8) ? (Take 3 digits after the decimal point)
a) 95.357
b) 69.567
c) 96.257
d) 78.584
e) 88.598

www.bankingpdf.com
Answers:
1).a) 2).b) 3).a) 4).c) 5).d) 6).c) 7).a) 8).c) 9).d) 10).a)

Solution:
1). 117.5 x (90 x 9.5 x 7.4) ÷ 9 =?
Sol: 117.5 x 6327 ÷ 9 = ?
117.5 x 703 = ?
? = 82602.5
Answer: a)

2). (138)2 + (511)2 – (132)2 = (85)2 + ?


Sol: 19044 + 261121 – 17424 = 7225 + ?
19044 + 261121 – 17424 – 7225 = ?
? = 255516
Answer: b)

3). 119 + 784 ÷ 32 x 116 = ?


Sol: 119 + 24.5 x 116 =?
119 + 2842 = ?
? = 2961
Answer: a)

4). (37÷44) + (78÷34) x (480÷85) = ?


Sol: 0.841 + 2.29412 x 5.64706 =?
0.841 + 12.955 =?
? = 13.796
Answer: c)

www.bankingpdf.com
5). 19/14 x 12/72 x 18/29 x 767 =?
Sol: 1.35 x 0.16 x 0.62 x 767 = ?
102.71 = ?
Answer: d)

6). 49/55 of 365/517 of 5111 = ?


Sol: 0.8909 x 0.7059 x 5111 = ?
0.62888631 x 5111 =?
? = 3214.2379
Answer: c)

7). 5005 x 87 – 22115 = ? x 320


Sol: 435435 – 22115 = ? x 320
413320/ 320 = ?
? = 1291.625
Answer: a)

8). [( 5 √11 + √11 ) x ( 4 √11 + 8 √11)] – (17)2 =?


Sol: 792 – 289 = ?
? = 503
Answer: c)

9). (3333÷30) + (665 ÷ 25) + (3771 ÷ 27) = ?


Sol: 111.1 + 26.6 + 139.667 = ?
? = 277.367
Answer: d)

www.bankingpdf.com
10). (37) 0.75 x [(7056) 0.25] ÷ (64) – 2 = (8) ?
Sol: 27.75 x 1764 ÷ 64 – 2 = 8 x ?
27.75 x 27.5625 – 2 = 8 x ?
762.859/8=?
? = 95.357
Answer: a)

570-580 Questions :

Directions (Q. 1-5): What will come in place of the question mark (?) in the following
series
1). 40 39 32 37 16 ? -8
a) 35
b) 36
c) 33
d) 32
e) 40

2). 13 6 7 9 20 47.5 ?
a) 149.5
b) 140.5
c) 150.5
d) 147.5
e) 145.5

3). 121 ? 113 138 89 170 49


a) 125

www.bankingpdf.com
b) 110
c) 122
d) 120
e) 133

4). 256 128 ? 192 96 240 80


a) 64
b) 156
c) 96
d) 128
e) 176

5). 9 -1 1 -1 -11 -285 ?


a) -10155
b) -10255
c) -10555
d) -10755
e) -10355

Directions (Q. 6-10): Study the following bar graph carefully and answer the questions
given below:
Investments (in lakh rs.) of two business states Rajasthan & Assam over the years

www.bankingpdf.com
6). What was the percent rise/fall in the total investment of Rajasthan & Assam together
from the year 1992 to 1995? (Rounded off to two digits after decimal)
a) 8.33%fall
b) 9.09% rise
c) 8.33% rise
d) 9.09%fall
e) None of these

7). What was the percent rise in Rajasthan’s investment in the year 1994 from the
previous year?
a) 25%
b) 20%
a) c) 33 1/3 %
b) d) 33 2/3 %
c) e) None of these

www.bankingpdf.com
8). Investment of Assam in the year 1993 is approximately what percent of his total
investment for all the years together?
a) 12
b) 18
c) 20
d) 17
e) 14

9). What was the percent rise in investment of Assam in the year 1994 from 1991?
a) 45.6
b) 37.5
c) 30
d) 60
e) None of these

10). What is the ratio between total investment of Rajasthan in the year 1991, 1992 and
1993 together and the total investment of Assam in these three years together
respectively?
a) 5:6
b) 6:5
c) c)15:17
d) d)17:15
e) e) None of these

Answers:
1).a) 2).e) 3).c) 4).d) 5).b) 6).b) 7).e) 8).e) 9).d) 10).a)

Solution:
1). The series is + (1^2 - 2), – (2^2 + 3), + (3^2 - 4), –(4^2 + 5), .....

www.bankingpdf.com
Answer: a)
2). The series is × 0.5 – 0.5, × 1 + 1, ×1.5 – 1.5, ×2 + 2, ...
Answer: e)
3). The series is +1^2 , -3^2 , +5^2 , - 7^2 , ....
Answer: c)
4). The series is ×0.5, ÷1, ×1.5, ÷2, ×2.5....
Answer: d)
5). The series is ×1^2 -10, ×2^2 +5, ×3^2 -10, ×4^2 +5 , .....
Answer: b)

6). Total investment of Rajasthan & Assam in 1992 = 40+70 = 110


Total investment of Rajasthan & Assam in 1995 = 50+70 = 120
= 120 – 110 × 100 / 110
= 10/110 × 100 = 9.09% increase
Answer: b)

7). % rise in Rajasthan's investment in 1994 from previous year


= 50 – 70 × 100 / 50 = 40%
Answer: e)

8). Investment of Assam in 1993 = 60


Total investment of Assam for all the years together
= 50+70+60+80+50+50+60
= 420
= 60 × 100 / 420
= 14% (approx.)
Answer: e)

www.bankingpdf.com
9). % rise investment of Assam in 1994 from 1991
= 80 – 50 × 100 / 50
= 30/50 × 100 = 60%
Answer: d)

10). Total investment of Rajasthan in 1991, 1992 &1993


= 60+40+50 = 150
Total investment of Assam in 1991, 1992 &1993
= 50+70+60 = 180
Ratio = 150 : 180
= 15 : 18
=5:6
Answer: a)

580-590 Questions :

1). A man on tour travels first 160 km at 64 km/hr and the next 160 km at 80 km/hr. The
average speed for the first 320 km of the tour is:
a) 35.55 km/hr
b) 36 km/hr
c) 71.11 km/hr
d) 71 km/hr
e) 69.69km/hr

2). The present age of Roshan’s father is four times Roshan’s present age. Five years
back, Roshan’s father seven times as old as Roshan was at that time. What is the
present age of Roshan’s father?
a) 43 years

www.bankingpdf.com
b) 67 years
c) 40 years
d) 69 years
e) 72 years

3). A portion of Rs.6600 is invested at a 5% per annum, while the remainder is invested
at a 3% per annum. If the annual income from the portion earning a 5% per annum is
twice that of the other portion, what is the total income from the two investments after
one year?
a) Rs.270
b) Rs.370
c) Rs.250
d) Rs.280
e) None of these

4). The rate of Simple Interest in UBI & BOI are in the ratio of 5:4. Mr.Naveen wants to
deposit his total savings in two banks in such a way that he receive equal half-yearly
interest from both banks.He should deposit in both banks UBI & BOI in the ratio of
a) 2:5
b) 1:5
c) 4:5
d) 3:5
e) None of these

5). Five years ago, the age of Arun was 4 times the age of Sarmi after 10 years, Arun
will be twice as old as Sarmi. Find the Present ages of Arun and sarmi?
a) 30 years, 10.5 years
b) 32 years, 11.5 years

www.bankingpdf.com
c) 34 years, 14.5 years
d) 35 years, 12.5 years
e) None of these

Direction (Q. 6-10)


Marks obtained by P, Q, R, S and T in Physics and Chemistry

6). Marks obtained by S in Chemistry is what percent of the total marks obtained by all
the students in Chemistry?
a) 26
b) 28.5
c) 35
d) 31.5
e) 22
7). If the marks obtained by T in Physics were increased by 14% of the original marks,
what would be his new approximate percentage in Physics, if the maximum marks in
Physics were 140?
a) 57
b) 32
c) 38

www.bankingpdf.com
d) 48
e) 41
8). Fill in the blank space in order to make the sentence correct as per the given
information. Total marks obtained by T in both the subjects together is more than the
marks obtained by_____
a) Q in Chemistry
b) R in Physics
c) S in Chemistry
d) P in Physics
e) R in both the subjects together
9). What is the respective ratio between the total marks obtained by P in Physics and
Chemistry together to the total marks obtained by T in Physics and Chemistry together?
a) 3 : 2
b) 20 : 11
c) 5 : 3
d) 2 : 1
e) None of these
10). What is the respective ratio between the total marks obtained by Q and S together
in Chemistry to the total marks obtained by P and R together in Physics?
a) 23 : 25
b) 23 : 21
c) 17 : 19
d) 17 : 23
e) None of these

Answers:
1).c) 2).c) 3).a) 4).c) 5).d) 6).a) 7).e) 8).b) 9).b) 10).b)

Solution:

www.bankingpdf.com
1). Total time taken = (160/64) + (160/80) = 9/2 hrs.
Avg Speed – 320 × (2/9) km/hr = 71.11km/hr
Answer: c)

2). Present age of Roshan – x


Present age of Roshan’s father = 4x
Five years ago,
Roshan’s father age = 7 * Ravi’s age
4x – 5 = 7(x – 5)
x = 10
Present age of Roshan’s father = 4x = 40 years.
Answer: c)

3). 5x + 3y = z (Total Income)


x + y = 6600 -(1)
5x= 2(3y)
5x – 6y = 0 -(2)
By solving (1) and (2) we get, x = 3600 so y = 3000
(3600*5*1)/100 + (3000*3*1)/100 = 180 + 90 = 270
Answer: a)

4). R1 =5x R2=4x T1=T2=1/2 yr


[P1*5x*(1/2)]/100 = [P2*4x*(1/2)]/100
P1:P2 = 4:5
Answer: c)

5). Five years ago,

www.bankingpdf.com
x – 5/y – 5 = 4/1
x – 4y = – 15 —-(i)
After 10 years,
x + 10/y + 10 = 2/1
x – 2y = 10 —–(ii)
From eqn (i) and (ii) => x = 35, y = 12.5
Answer: d)

6). Required percentage mark = [120/(90 + 110 + 100 + 120 + 60 )]* 100
= (120 / 460)* 100 = 26%
Answer: a)

7). New marks of T in physics = (114/100)* 50 = 57


T’s new percentage = (57/140)* 100 = 41
Answer: e)

8). Marks obtained by T in both subjects together is more than the marks obtained by R
in Physics.
Answer: b)

9). Required ratio = 130 + 70 : 50 + 60


= 200 : 110 = 20: 11
Answer: b)

10). Required ratio = 110 + 120 : 130 + 80 = 230 : 210 = 23 : 21


Answer: b)

www.bankingpdf.com
590-600 Questions :

Directions (Q. 1-5): In the following questions two equations numbered I and II are
given. you have to solve both the equations and choose the correct option.
a) x < y
b) x ≥ y
c) x > y
d) x ≤ y
e) x = y or the relationship between x and y cannot be determined

1). I. x2 – 25x + 154 = 0


II. y2 – 20y + 99 = 0

2). I. 4x2 – 56x + 192 = 0


II. 5y2 – 80y + 315 = 0

3). I. 6x – 5y = 11
II. 3y – 4x = -5

4). I. 3x = 2187
II. 2y = 512

5). I. 45x2 – 43x + 10 = 0


II. 72y2 – 103y + 35 = 0

www.bankingpdf.com
Directions (Q. 6-10): Study the table carefully to answer the questions that follow:
Total number of employees in different departments of an organization and (of these)
percentage of females and males
Department Total Number of Percentage of females Percentage of males
Employees

IT 840 45 55

Accounts 220 35 65

Production 900 23 77

HR 360 65 35

Marketing 450 44 56

Customer Service 540 40 60

6). What is respective ratio of the number of females in production department to the
number of females in the Marketing department?
a) 22 : 23
b) 35 : 33
c) 23 : 22
d) 33 : 35
e) None of these

7). What is the ratio of the number of female in the HR and Accounts departments
together to the number of males in the same departments together?

www.bankingpdf.com
a) 311 : 269
b) 268 : 319
c) 296 : 311
d) 319 : 268
e) None of these

8). What is the total number of employees in all the departments together?
a) 3260
b) 3310
c) 3140
d) 3020
e) None of these

9). The total number of employees in the HR department forms approximately what
percent of the total number of employees in the Accounts department?
a) 149
b) 178
c) 157
d) 164
e) 137

10). What is the total number of males in the IT and Customer Service departments
together?
a) 687
b) 678
c) 768
d) 876
e) None of these

www.bankingpdf.com
Answers:
1). b) 2). e) 3). c) 4). a) 5). d) 6). c) 7). a) 8). b) 9). d) 10). e)

Solution:
1). From I, (x-11)(x-14) = 0
x = 11, 14
from II, (y-11)(y-9) = 0
y = 11, 9
hence x ≥ y
Answer: b)

2). From I, 4(x2 – 14x + 48) = 0


4 (x-6)(x-8) = 0
x = 6, 8
From II, 5(y2 – 16y + 63) = 0
5(y - 7)(y-9) = 0
y = 7, 9
x = y or the relationship between x and y cannot be determined
Answer: e)

3). Equ I × 2 + II ×3  12x – 10 y + 9y – 12x = 22-15


y = -7
and x = -4
hence x > y
Answer: c)

www.bankingpdf.com
4). From I, 3x = 37
x=7
from II, 2y = 512
y=9
hence x < y
Answer: a)

5). From I, 45x2 – 25x – 18 x + 10 = 0


5x (9x - 5) – 2(9x - 5) = 0
(9x - 5)(5x - 2) = 0
x = 5/9, 2/5
From II, 72y2 – 40y – 63y + 35 = 0
8y (9y - 5) -7(9y - 5) = 0
(9y - 5)(8y - 7) = 0
y = 5/9, 7/8
hence x ≤ y
Answer: d)

6). The number of females in Production department = 0.23 × 900


The number of females in Marketing department = 0.44 × 450
ratio = 0.23 × 900 : 0.44 × 450 = 0.46 : 0.44 = 23 : 22
Answer: c)

7). The number of females in the HR and Accounts departments together = 360 × 0.65
+ 220 × 0.35 = 331
The number of males in the HR and Accounts departments together = 360 × 0.35 + 220
× 0.65 = 269
ratio = 311 : 269
Answer: a)

www.bankingpdf.com
8). The total number of employees in all the departments together = 840 + 220 + 900 +
360 + 450 + 540 = 3310
Answer: b)

9). The total number of employees in the HR department = 360


The total number of employees in the Accounts department = 220
Required percentage = (360/220) × 100 = 163.63%
Answer: d)

10). The total number of males in the IT and Customer service departments together =
840 × 0.55 + 540 × 0.6 = 786
Answer: e)

610-620 Questions :

Directions (Q.1-5) : Based on the data given in the table, answer the following
questions:
The table below shows the runs scored by 6 different batsmen in 5 different
tournaments.
Each entry is of the Runs Scored followed by matches played in brackets. For example,
Deepak scored 721 runs in 14 matches in Champions Trophy.

www.bankingpdf.com
1).Which batsmen has the highest average among the players in all tournaments put
together?
a) Venkat
b) Sandeep
c) Deepak
d) Kumar
e) Rajesh

2). Which tournament had the lowest cumulative batting average?(This will be total runs
scored in the tournament divided by sum of total matches played by all batsmen. For
eg, In FEFA, the average will be
(477+112+141+177+221+77)/(8+2+8+8+9+2)
a) World cup
b) ICCI
c) FEFA
d) Champions trophy
e) Asia Cup

3). What is the difference in runs scored by kumar and Deepak in Champions Trophy
and World cup to that of Rajan and Sandeep together in the same two tournaments?

www.bankingpdf.com
a) 588
b) 493
c) 563
d) 383
e) None of the above

4). What are the minimum number of 100’s scored by batsmen across all the
tournaments?
a) 2
b) 3
c) 4
d) 1
e) None of the above

5). What is the overall average of all batsmen across the 5 tournaments (Overall
average =Total runs scored by all batsmen /total matches played by all batsmen)?
a) 29
b) 33
c) 38
d) 27
e) None of the above

Directions (Q.6-10): The chart below shows the percentage contribution in sugar
production by 7 states A, B, C, D, E, F, G. The total output across these seven states is
13500kg. (The production of sugar in each state is a multiple of 250kg.)

www.bankingpdf.com
The table below shows the selling price per kg of sugarcane in each of these states.
State Cost per kg

A 14

B 12

C 16

D 15

E 13

F 16

G 12

Sugar revenue for a state =Sugar Production in kgs × Cost per kg of sugarcane.

6). What is the difference in Sugar revenue of State C and D?


a) 22500
b) 12500

www.bankingpdf.com
c) 15000
d) 27500
e) 32500

7). If transportation losses of sugar in States E,G are 15% and 20% of the production,
what is the difference of their sugar revenues?
a) Between 1,000 and 2,500
b) Between 2,501 and 5,000
c) Between 5,001 and 7,500
d) Between 7,500 and 10,000
e) More than 10,000

8). What is the combined sugar revenue for all the seven states?
a) 173250
b) 193250
c) 193750
d) 173750
e) None of the above

9). States A, B, C, D export 10%,10%,15%,15% of their total production. If the export


price per kg is same the price in the respective state, what is the net revenue from
exports for these 4 states?
a) 19225
b) 17500
c) 12925
d) 15700
e) 23750

www.bankingpdf.com
10). If the cost of production per kg of sugar is 7Rs per kg in all the A, what is the net
profit in A?(Net Profit=Selling Price –Cost of Production)
a) 28000
b) 18000
c) 14000
d) 21000
e) 35000

Answers:
1). c) 2). b) 3). d) 4). a) 5). b) 6). a) 7). c) 8). b) 9). a) 10). c)

Solution:
Directions (Q. 1-5):

www.bankingpdf.com
1). From the table, Deepak has the highest average among the players in all
tournaments put together.
Answer: c)

2). From the table, we have ICCI had the lowest average.
Answer: b)

3). Deepak + Kumar = 721+187+190+187 =1285


Rajan + Sandeep = 408+135+203+156 = 902
Required Difference=1285-902=383
Answer: d)

4).The only cases where there are definite hundreds are where the average is greater
than 100 for a batsmen in any tournament.
So we have 1 hundred in ICCI for Rajesh (101 in 1 match)
So we have 1 hundred in Asia Cup for Rajan (210 in 2 matches means at least 1
hundred, for the scores can be 190,20).
So, minimum number of 100’s scored by batsmen across all the tournaments is 2.
Answer: a)

5).From the table, we have total average = 5525/167 =33.08~ =33.


Answer: b)

Directions (Q. 6-10): The data shown in Pie chart is shown in the table below:
State Production

www.bankingpdf.com
A 2000

B 1500

C 3750

D 2500

E 1750

F 750

G 1250

6). C=3750 × 16 = 60,000


D=2500 × 15=37,500
Difference=22,500
Answer: a)

7).E=1750 × 0.85 × 13 =
G=1250 × 0.8 × 12 =
E-G=19337.5 – 12000=7337.50
Answer: c)

8).Table below show the revenue. Total =193250


State Revenue

A 28000

www.bankingpdf.com
B 18000

C 60000

D 37500

E 22750

F 12000

G 15000

Total 193250

Answer: b)

9).
State Revenue

A 28000

B 18000

C 60000

D 37500

10% of A +10% of B + 15% of C +15% of D =2800+1800+9000+5625=19225.


Answer: a)

10).Revenue =Selling Price × Production


Cost = Cost × Production

www.bankingpdf.com
Profit =2000 × (14-7)=14000

Answer: c)

620-630 Questions :

Directions (Q. 1-5): What approximate value will come in place of question mark(?) in
the following questions?(you are not expected to calculate the exact value.)
1). 3599 ÷ 224.95 × 23.04 = ? + 16.98 × 11.03
a) 130
b) 210
c) 180
d) 115
e) 155

2). 8 1/4 ×5 2/3 – 4 1/6 × 4 1/5 = ?


a) 31
b) 18
c) 14
d) 27
e) 23

3). (18.97)2 + (20.03)2 + (20.98)2 + (22.04)2 = ?


a) 1680
b) 1720
c) 1620
d) 1590
e) 1650

www.bankingpdf.com
4).32.02 × 28.04 × (27.97)2 = ? × 24.12
a) 95
b) 40
c) 60
d) 115
e) 70

5). 79.93 × 35.02 -59.93 × 85.08 + 74.98 × 44.04 =?


a) 1420
b) 1000
c) 1580
d) 870
e) 1260

Directions (Q. 6-10): What value will come in place of question mark (?) in the following
questions?
6). (45)3 ÷ 175 × 189 ÷405 = 3?+6
a) -1
b) 7
c) -3
d) 5
e) 4

7).17 × 24 – 108 + 386 – 621 ÷ 9 + 53= ?


a) 564
b) 836
c) 684

www.bankingpdf.com
d) 432
e) 742

8).162 + 132 + 102 + 73 = ?


a) 2602
b) 2896
c) 2770
d) 2474
e) 2342

9). ?% of 725 + 14% of 2100 = 700


a) 64
b) 80
c) 72
d) 56
e) 48

10).1357 + 294 + 487 -3412 +8765 -5793= ?


a) 1444
b) 2588
c) 1698
d) 2155
e) 1241

Answers:
1). c) 2). d) 3). a) 4). e) 5). b) 6). a) 7). e) 8). b) 9). d) 10). c)

www.bankingpdf.com
Solution:
1). 3600 ÷ 225 × 23 = ? +17 × 11
=>16 × 23 = ? + 187
=> ? = 368 – 187 =181 ≈ 180
Answer: c)

2). ? = (33/4) × (27/5) – (25/6) × (21/5) =891/20 -35/2 ≈ 44.5 – 17.5 =27
Answer: d)

3). ? ≈192 + 202 +212 +222


=361+400+441+484 = 1686 ≈ 1680
Answer: a)

4). 32 × 28 + 282 = ? × 24
=>302 – 22 + 282 = ? × 24
=>900 -4 +784 =? × 24
=> ? = 1680/24 = 70
Answer: e)

5). ? ≈ 80 × 35-60 × 85 + 75 × 44
=2800 – 5100 + 3300 = 1000
Answer: b)

6). 3?+6 =(32 × 5)3 ÷ 52 × 7 × 33 × 7 ÷ 34×5


=35×50 × 70
=>? + 6 =5
=> ? = -1

www.bankingpdf.com
Answer: a)

7). ? = 408-108 +386-69 +125


=300+317 +125=742
Answer: e)

8). ?=256 + 2197 +100 +343 =2896


Answer: b)

9). ?% × 725 +14 × 21=700


=>?% × 725 +294 =700
=> ?% × 725 =406
=>?=406/725 × 100 =56
Answer: d)

10). ? =(1357 + 294 +487+8765)-(3412+5793)


=10903-9205=1698
Answer: c)

630-640 Questions :

Directions (Q.1-5)
Price Variation of 5 Pulses over a 5 year Period. All Prices are for 1kg of the Pulse.

www.bankingpdf.com
Note :Price of Toor Dal and Til were the same in 2009.

1).If the price of Moong Dal and Toor Dal remain the same for 2014(compared to
2013),and price of other 3 items go up by 15% each (per kg compared to 2013), what
will be the cost of 2kg of Moong Dal, and 4 kg of Poha in 2014?
a) 300
b) 320
c) 360
d) 340
e) 280

www.bankingpdf.com
2).Pulse Factor is defined as the cost of 1kg each of the 5 pulses. Which year had the
3rd highest Pulse Factor?
a) 2009
b) 2010
c) 2011
d) 2012
e) 2013

3).Which Pulse showed the highest percentage increase in price over the 5 year
period?
a) Moong Dal
b) Rajma
c) Toor Dal
d) Til
e) Poha

4). If the Pulse Index is defined as Cost of 5kg of Toor Dal, 9 kg of Moong Dal, 13kg of
Rajma, 4 kg of Til, and 1 kg of Poha, which year had the 2nd highest Pulse Index?
a) 2009
b) 2013
c) 2010
d) 2012
e) 2011

5).A family consumes 100 kg of Toor Dal, 90 kg of Moong Dal and 75 kg of Rajma every
year. What is the percentage increase in expenditure of the family from 2009 to 2012?
a) 50.7%
b) 49.8%
c) 52.3%

www.bankingpdf.com
d) 54%
e) 53.5%

Directions (Q.6-10): 980 families in a township use 7 brands of toothpaste: Coolgate,


Farup, Cibacal, Palmolie, Pepsodebt, Meswik and Krystal. It is known that families that
use Pepsodebt is more than the number of families using Faurp. The number of families
using Meswik is 25% more than that of Cibacal. Coolgate has 20% more families using
it than Palmolie. The numbr of families that use Krystal is twice the difference between
number of families using Pepsodebt and Farup. The number of families using Cibacal is
twice that of Krystal. Families using Farup are 20 less than those using Cibacal. The
number of familes using any toothpaste is a multiple of 10, and it is known that number
of families using Krystal is less than 120.

6). How many families use Pepsodebt?


a) 120
b) 140
c) 160
d) 180
e) Cannot be determined

7).What is the difference between number of families using Cibacal and Farup?
a) 20
b) 40
c) 60
d) 80
e) Cannot be determined

www.bankingpdf.com
8). If 20 families using Pepsodebt start using Krystal, what is the ratio of families using
Meswik to those using Krystal?
a) 1:2
b) 2:1
c) 1:3
d) 3:1
e) Cannot be determined

9). If the cost of Meswik is Rs.40 per packet, and all families using Meswik use 2
packets a month, what is the total expenditure of all families using Meswik on
toothpastes?
a) 12000
b) 14000
c) 16000
d) 180000
e) Cannot be determined

10).How many families use Palmolie or Coolgate?


a) 110
b) 220
c) 330
d) 440
e) Cannot be determined

Answers:
1). c) 2). c) 3). c) 4). d) 5). a) 6). d) 7). a) 8). b) 9). c) 10). b)

Solution:

www.bankingpdf.com
Directions (Q. 1-5):
2009 2010 2011 2012 2013

Toor Dal 25 30 35 40 45

Moong Dal 40 50 60 60 65

Rajma 35 40 45 50 55

Til 25 30 30 30 35

Poha 30 40 40 40 50

1). Moong in 2014 = 65,Poha in 2014 = 50 ×1.15 = 57.5


Required Value = 2 × 65 + 4×57.5 =360
Answer: c)

2).3rd Highest Pulse Factor was in 2011


2009 2010 2011 2012 2013

Pulse Factor 155 190 210 220 250

Answer: c)

3).
2009 2013 Increase

Toor Dal 25 45 80.00%

www.bankingpdf.com
Moong Dal 40 65 62.50%

Rajma 35 55 57.14%

Til 25 35 40.00%

Poha 30 50 66.67%

Answer: c)

4). 2nd Highest Index=2012(2013 was the highest)


2009 2010 2011 2012 2013

Index 1070 1280 1460 1550 1715

Answer: d)

5). 2009 Expenditure = 100 × 25 + 90×40+75×35=8725


2012 Expenditure =100×40+90×60+75×50=13150
Percentage Increase =(13150-8725) ×100/8725~=50.7%
Answer: a)

Directions (Q. 6-10):


Toothpaste Families

Coolgate 120

Farup 140

Cibacal 160

www.bankingpdf.com
Palmolie 100

Pepsodebt 180

Meswik 200

Krystal 80

6). From the table,we have 180 users use Pepsodebt.


Answer: d)

7).Cibacal-Farup=160-140=20.
Answer: a)

8).Krystal =80+20=100
Meswik = 200
Ratio of Meswik to Krystal=200:100=2:1
Answer: b)

9).Per Family Cost = 40 × 2 =80


Expenditure=80×200=16,000
Answer: c)

10).Palmolie or Coolgate =100+120=220

Answer: b)

www.bankingpdf.com
640-650 Questions :

Directions (Q. 1-5): In the following questions two equations numbered I and II are
given. You have to solve both the equations and choose the correct option.
a) x<y
b) x≥y
c) x>y
d) x≤y
e) x=y or the relationship between x and y cannot be determined

1).I.x=√529
II.5y-3x=11

2).I.5x2 -12x+18=0
II.3.5y2+7y-28=0

3).I.x2-14x+48=0
II.y2-21y+108=0

4).I.x4=1296
II.y2-14y+48=0

5).I.20x2-27x+9=0
II.6y2-7y+2=0
x=y or the relationship between x and y cannot be determined

Directions (Q. 6-10): Find the missing number in the following number series:

www.bankingpdf.com
6). 67,74,78,?,92,94
a) 86
b) 85
c) 83
d) 84
e) 88

7).25,?,45,58,73,90,109
a) 33
b) 34
c) 35
d) 38
e) 36

8).11,66,462,3696,?
a) 32364
b) 33364
c) 32264
d) 33264
e) 32664

9).520, 130, 32.5, 8.125, 2.03125,?


a) 0.5087125
b) 0.5071875
c) 0.5012875
d) 0.5028125
e) 0.5078125

www.bankingpdf.com
10).97,122,?,307,503,792
a) 171
b) 158
c) 186
d) 203
e) 178

Answers:
1). c) 2). b) 3). a) 4). d) 5). e) 6). a) 7). b) 8). d) 9). e) 10). c)

Solution:
1).From I,x=23
From II,5y-69=11
=>5y=80
=>y=16
Hence x>y
Answer: c)

2).From I,1.5(x2-8x+12)=0
=>1.5(x-2)(x-6)=0
=>x=2,6
From II,3.5(y2+2y-8)=0
=>3.5(y-2)(y+4)=0
=>y=2,-4
Hence x≥y
Answer: b)

3).From I,(x-6)(x-8)=0
=>x=6,8

www.bankingpdf.com
From II,(y-9)(y-12)=0
=>y=9,12
Hence x<y
Answer: a)

4).From I,x=±6
From II,(y-8)(y-6)=0
=>y=8,6
Hence x≤y
Answer: d)

5).From I,(4x-3)(5x-3)=0
=>x= 3/4,3/5=0.75,0.6
From II,(2y-1)(3y-2)=0
=>y=1/2,2/3=0.5,0.66
One value of x lies between the two values of y and vice versa, hence the relation
cannot be determined.
Answer: e)

6).The next term is obtained by adding the unit’s digit to the number.
Hence the missing number =78+8=86
Answer: a)

7).The series is 3×5+10=25,4×6+10=34,5×7+10=45,6×8+10=58,7×9+10=73,8×


10+10=90,9×11+10=109
Hence missing term=34
Answer: b)

www.bankingpdf.com
8).11×6=66,66×7=462,462×8=3696,3696×9=33264
Answer: d)

9).The series is a geometric progression with common ratio 1/4


Hence the next term is 2.03125/4=0.5078125
Answer: e)

10).The pattern is 97+52=122,122+82=186,186+112=307,307+142=503,503+172=792


Answer: c)

650-660 Questions :

1).Rubesh generally takes 80 minutes to reach home from office. Due to rain, he
travelled at 2/3rd of his usual speed for the 1st half of the distance. At what speed should
rubesh travel the remaining distance to reach home on time?
a) 1.5 times the original speed
b) 2 times the original speed
c) 2.5 times the original speed
d) 3 times the original speed
e) Cannot reach on time

2).If x:y =3:4,in the ratio of (7x-4y) : ( 3x + y)


a) 2:11
b) 4:9
c) 5:13
d) 6:5

www.bankingpdf.com
e) 13:5

3).Shweta deposits a certain amount on 1st January and 1st July (Same amount on both
days) in a bank that offers 5% compound interest, compounded half-yearly. Shweta
gains interest of Rs.121 at the end of the year. How much money did Shweta deposit?
a) 1200
b) 1300
c) 1400
d) 1500
e) 1600

4).10 years ago the average age of all the 25 teachers of a college was 45 years. 4
years ago, a teacher retired from the post at the age of 60 years, and was replaced by a
new teacher whose age was 54 years. What is the present average age of all the
teachers?
a) Between 52 and 53
b) Between 53 and 54
c) Between 54 and 55
d) Between 55 and 56
e) None of the above

5).A copper wire is bent to form an equilateral triangle of area 16√3 sq.cm. What will be
the area (in sq.cm) if the same copper wire was bent to form a square?
a) 16
b) 24
c) 32
d) 40
e) None of the above

www.bankingpdf.com
6).A box contains 15bolts of which 5 are defective. If 5 bolts are selected at random
from the box, what is the probability that at least one of them is defective?
a) 91/143
b) 101/143
c) 111/143
d) 121/143
e) 131/143

7).By selling 45 lemons for Rs 40, a man loses 20%.How many should he sell for Rs 24
to gain 20% in the transaction?
a) 19
b) 22
c) 20
d) 18
e) 14

8).Ram, Suresh and Shilpa, working together, can do a piece of work in 1 day. Ram is
thrice as efficient as Suresh, and Suresh takes twice the number of days as Shilpa
takes to do it alone. What is the difference between the number of days taken by Ram
and Shilpa, each working alone, to complete the work?
a) 1
b) 2
c) 3
d) 4
e) 5

www.bankingpdf.com
9).Anjana walked at 4 kmph for a certain part of his journey and took an auto, travelling
at 25 kmph for the rest of the journey. If he took 42 minutes for the entire journey, what
part of the journey did he walk given that his average speed for the entire journey was
10 kmph?
a) 2 km
b) 1.5 km
c) 3.5 km
d) 4 km
e) 1 km

10).When the integers from 1 to 999 are written on a paper, the number of zeros used in
writing it is:
a) 192
b) 189
c) 200
d) 203
e) 216

Answers:
1). b) 2). c) 3). e) 4). c) 5). e) 6). e) 7). d) 8). a) 9). a) 10). b)

Solution:
1). Under the normal speed, 40 minutes for half the distance
Due to rain, time taken will be 40 / (2/3) = 40 × (3/2) = 60 minutes
So, in 20 minutes he has to cover the distance he usually covers in 40 minutes.
So, he has to travel at twice the original speed.
Answer: b)

www.bankingpdf.com
2). An easy way to solve this question is use x = 3, y=4
7x – 4y = 21 – 16 = 5
3x + y = 9+4 = 13
Required ratio = 5:13
Answer: c)

3). Let the amount be P


Half yearly rate = 5/2 = 2.5%
For the deposit on January, Amount = P (1 + 2.5 / 100)2
For the deposit on July, Amount = P (1 + 2.5 /100)
We can solve the question by substituting options now.
If P = 1600, Amount in January = 1600 × 1.025 × 1.025 = 1681
Amount in July = 1600 × 1.025 = 1640
So total interest = 81 + 40 = 121
Required answer is 1600
Answer: e)

4). Let current year be 2015


In 2005, sum of all ages = 25×45 = 1125
In 2015, sum of all ages = 1125 + (25×10) = 1375
Now 60 year old teacher was replaced by a 54 year old teacher. So we need to subtract
6 from the total.
Sum of all ages in 2015 considering the replacement teacher = 1375 – 6 = 1369
Average = 1369/25 = 54.76
Answer: c)

5). Area of equilateral triangle = 16√3 = √3 (side)2 /4


Or side2 = 16×4 = 64
So each side = 8cm

www.bankingpdf.com
Perimeter = 8×3 = 24 cm
Since perimeter of the square is 24 cm, each side = 6cm
So area = 6×6 = 36 sq cm
Answer: e)

6). Probability that at least one is defective = 1 – probability that none is defective
Probability that none is defective = 10C5 / 15C5 = 12/143
Required Probability = 1-12/143 = 131/143
Answer: e)

7). Selling price = Rs. 40


Loss = 20%
Therefore, cost price = 40×100 /(100 - 20) = Rs. 50
Therefore, cost price of each lemon = Rs. 50/45
Now, selling price = Rs. 24
Profit = 20%
Cost price = 24×100/120 = Rs. 20
Number of lemons for Rs. 20 = 20/(50/45) = 900/50 = 18
Answer: d)

8). Let the time taken by Ram, Suresh and Shilpa to finish the work alone be A, B and C
respectively
Therefore, 1/A + 1/B + 1/C = 1
Also it is given that,
A = B/3 and C =B/2
3/B + 1/B + 2/B = 6/B = 1, or B = 6
Hence, time taken by Ram, Suresh and Shilpa to finish the work alone is 2 days, 6 days
and 3 days respectively

www.bankingpdf.com
Hence, difference between the number of days taken by Ram and Shilpa, each working
alone, to complete the work = 3-2 = 1
Answer: a)

9). Total time = 42 minutes = 0.7 hours.


Average speed = 10 kmph
Total distance = 0.7×10 = 7 km
Let distance walked = W
W/4 + (7-W)/25 = 0.7
Solving W = 2
Answer: a)

10). From 1 to 10 : 1 zero


From 11-100 : 10 zeros (100 has 2 zeros)
From 101 – 900 : 8×20 = 160 zeros
From 901-999 : we have 18 zeros
Total = 1+10+160+18 = 189 zeros
Answer: b)

660-670 Questions :

Directions (Q.1-5):
The table below shows the sale of multiple Cracker items in 5 shops during Diwali.
Answer the questions that follow. All Units Prices are in Rs.

www.bankingpdf.com
Revenue from a Cracker Item =Unit Price of Item × Units Sold of Item
For eg, Revenue in Shop 1 from Flowerpots = 45 × 40=1800.

1).What is the percentage revenue from Flowerpots in Shop2 compared to overall


revenue from shop2?
a) 25%
b) 27.5%
c) 30%
d) 32.5%
e) 35%

2).What was the total Revenue of Shop 1?


a) 4925
b) 7350
c) 8175
d) 5025
e) 5145

3).Shops 3 and 4 make up for what percentage of the total revenue from all 5 shops?
a) 40%

www.bankingpdf.com
b) 43%
c) 51%
d) 46%
e) 50%

4).The total revenue from sale of Zamin Chakras across all the 5 shops is ?
a) 8050
b) 8950
c) 8450
d) 7850
e) 7900

5).If the price of Rockets were 10% more, and the number of items sold remained the
same, what would have been the revenue from selling rockets in Shop 5?
a) 880
b) 800
c) 720
d) 640
e) 920

Directions (Q.6-10): Study the following pie-chart and bar diagram and answer the
following questions. Percentage-wise distribution of Students in six different Schools.
Total number of Students =6000
Percentage of students

www.bankingpdf.com
6).What is the sum of the number of girls in School C, the number of girls in School E
and the number of boys in School D together?

a) 1700
b) 1900
c) 1600
d) 1800
e) None of these

www.bankingpdf.com
7).What is the ratio of the number of boys in School C, the number of girls in School B
and the total number of students in School E?
a) 45:7:97
b) 43:9:97
c) 45:7:87
d) 43:9:87
e) None of these

8).What is the difference between the total number of students in School F and the
number of boys in School E?
a) 820
b) 860
c) 880
d) 900
e) None of these

9).In which of the following schools is the total number of students equal to the number
of girls in School E?
a) A
b) B
c) C
d) D
e) F

10).The number of girls in School A is approximately what percentage of the total


number of students in School B?
a) 55
b) 50
c) 35

www.bankingpdf.com
d) 45
e) 40

Answers:
1)b 2)a 3)d 4)c 5)a 6)d 7)c 8)e 9)b 10)e

Solution:
Directions (Q.1-5):

Percentage =1750 ×100/6350~=27.5%


1).Shop2 Flowerpot =1750,Total =6350
Answer: b)

2).From the table,Shop 1 Revenue =4925.


Answer: a)

3).Shop 3 and 4 =7900+8175=16075


Percentage =16075 × 100/34700~=46%
Answer: d)

4).Zamin Chakras Revenue=8450


Answer: c)

www.bankingpdf.com
5).W just need to find 1.1 × Rocket Revenue in Shop 5=1.1 × 800 =880.
Answer: a)

Directions (Q. 6-10):


School Total number of Students Number of boys Number of girls

A 0.12×6000=720 500 720-500=220

B 0.09×6000=540 400 540-400=140

C 0.26×6000=1560 900 1560-900=660

D 0.18×6000=1080 600 1080-600=480

E 0.29×6000=1740 1200 1740-1200=540

F 0.06×6000=360 100 360-100=260

6).Number of girls in school C=660


Number of girls in school E=540
Number of boys in school D=600
Sum =660+540+600=1800
Answer: d)

7). Number of boys in school C=900


Number of girls in school B=140
Number of students in school E=1740
Their ratio =900:140:1740=45:7:87
Answer: c)

www.bankingpdf.com
8).Number of students in school F=360
Number of boys in school E=1200
Difference =1200-360=840
Answer: e)

9).Number of students in school B= number of girls in school E =540


Answer: b)

10).number of girls in school A=220


Number of students in school B=540
required percentage = 220/540 × 100=40.7%
Answer: e)

670-680 Questions :

Directions (Q. 1-5): In the following questions three equations numbered I, II and III are
given. You have to solve all the equations either together or separately, or two together
and one separately, or by any other method and—

Give answer
a) X<Y=Z
b) X≤Y<Z
c) X<Y>Z
d) X=Y>Z
e) X=Y=Z or if none of the above relationship is established.

1). I. 8X – 245=3X – 41
II. 17Y= (14)2+√225 +(17)2 +3Y

www.bankingpdf.com
2). I. 18X – 1445= - 13X -140
II. 21Y = (21)2 + √961+ (26)2+4Y

3). I. 118X – 1145 = - 15X – 141


II. 121Y = (23)2 + √1156 + (16)2+5Y

4). I. 216X – 1020 = -11X – 131


II. 211Y = (43)2 + √1089 + (13)2+13Y

5). I. 315X – 2010 = - 23X -231


II.299Y = (53)2+√1296+ (19)2+11Y

Direction (Q.6-10): What value should come in the place of question mark (?) in the
following number series?
6). 28, 42, 84, 210, 630, ?
a) 2675
b) 2515
c) 2445
d) 2375
e) 2205

7). 13, 15, 22, 50, 113, ?


a) 239

www.bankingpdf.com
b) 272
c) 287
d) 296
e) 306

8). 8, 17, 38, 85, 186, ?


a) 318
b) 354
c) 397
d) 408
e) 427

9). 5042, 5187, 5334, 5483, 5634, ?


a) 5716
b) 5748
c) 5787
d) 5793
e) None of these

10). 3, 9, 17, 27, 39, 53, ?


a) 64
b) 69
c) 74
d) 78
e) 81

Answers:

www.bankingpdf.com
1).a) 2).c) 3).a) 4).c) 5).c) 6).e) 7).a) 8).c) 9).c) 10). b)

Solutions:
1). I. 8X – 245=3X – 41
II. 17Y= (14)2+√225 +(17)2 +3Y
Sol: I. 8X – 245=3X – 41
Or, 5X = 204
Or, X = 40.8
II. 17Y= (14)2+√225 +(17)2 +3Y
Or, 14Y = 500
Or, Y = 35.7
Answer: a)

2). I. 18X – 1445= - 13X -140


II. 21Y = (21)2 + √961+ (26)2+4Y
Sol: I. 18X – 1445= - 13X -140
Or, 31X = 1305
Or, X = 42.1
II. 21Y = (21)2 + √961+ (26)2+4Y
Or, 17Y = 1131
Or, Y=66.53
Answer: c)

3). I. 118X – 1145 = - 15X – 141


II. 121Y = (23)2 + √1156 + (16)2+5Y
Sol: ) I. 118X – 1145 = - 15X – 141

www.bankingpdf.com
Or, 133X = 1004
Or, X = 7.55
II. 121Y = (23)2 + √1156 + (16)2+5Y
Or, 116Y= 819
Or, Y=7.1
Answer: a)

4). I. 216X – 1020 = -11X – 131


II. 211Y = (43)2 + √1089 + (13)2+13Y
Sol: I. 216X – 1020 = -11X – 131
Or, 227X= 889
Or, X= 3.92
II. 211Y = (43)2 + √1089 + (13)2+13Y
Or, 198Y= 2051
Or, Y =10.36
Answer: c)

5). I. 315X – 2010 = - 23X -231


II.299Y = (53)2+√1296+ (19)2+11Y
Sol: I. 315X – 2010 = - 23X -231
Or, 338X= 1779
Or, X=5.3
II.299Y = (53)2+√1296+ (19)2+11Y
Or, 288Y= 3206
Or, Y= 11.13
Answer: c)

www.bankingpdf.com
6). Series is × 1.5, ×2, ×2.5, ×3, ×3.5
Answer: e)

7). Series is + 13 +1, +23 -1, +33 +1, 43 -1


Answer: a)

8). Series is ×2+12 , ×2+22 , ×2+32 , ×2+42


Answer: c)

9). Series is (71)2 +1, (72)2 +3, (73)2 +5


Answer: c)

10). Series is 1× 2 + 1, 2 × 3 + 3, 3 × 4 + 5, 4 × 5 + 7, 5 × 6 + 9
Answer: b)

680-690 Questions :

Directions (1-5):
There are six companies which produce two types of TV (LED and LCD). The total
production cost of all six companies together is 8 crore rupees. The following pie
chart shows the percentage distribution of the total production, and the table shows
the ratio of production of LED to LCD TV and per cent profit for these two types

www.bankingpdf.com
1). What is the sum of the profit earned by Philips on LCD and that by LG on LED?
(Answer in lakh)
a) Rs 22.48
b) Rs 24.84
c) Rs 26.24
d) Rs 28.75
e) Rs 32
2). What is the total production cost (in Rs) of LCD TV by Samsung and Videocon
together?
a) 1.24 crore
b) 1.28 crore

www.bankingpdf.com
c) 1.32 crore
d) 1.36 crore
e) 1.4 crore
3). What is the total profit earned by Onida for both LED and LCD together? (Answer
in crore)
a) Rs 0.426
b) Rs 0.464
c) Rs 0.492
d) Rs 0.524
e) Rs 0.584
4). The profit earned by Videocon on LCD TV is what per cent of the total production
cost of Samsung on LED TV? (Answer in approximate value)
a) 7.5%
b) 10%
c) 12.5%
d) 15%
e) 17.5%
5). What is the ratio of the profit earned on LED to that on LCD TV by Sony?
a) 5 : 7
b) 12 : 25
c) 3 : 7
d) 3 : 5
e) None of these

Directions (6-10):
Study The Following Graph Carefully And Answer The Questions Given Below
Import and Export of spare parts by an automobile company over the given years

www.bankingpdf.com
6). During which year the percentage rise/fall in imports from the previous year is the
lowest?
a) 1999
b) 1998
c) 1997
d) 1995
e) None of these
7). What is the ratio of total imports to total exports to all the given years together?
a) 63:65
b) 64:66
c) 70:75
d) 40:45
e) None of these
8). What following pairs of years the total import is equal to total export in the same
pair of year?
a) 1996-1997
b) 1995-1996
c) 1997-1996
d) 1999-1996
e) None of these

www.bankingpdf.com
9). The total imports in the year 1995, 1996 and 1999 together are what percent of
the total export during the same period?
a) 157.41
b) 107.41
c) 172.45
d) 127.21
e) None of these
10). which of the following pairs of years and the percent increase in the export over
the previous year is the least?
a) 1999
b) 1998
c) 1997
d) 1995
e) None of these

1) c 2) b 3) c 4) c 5) e 6) d 7)a 8) b 9) b 10) a

Detailed Solutions:
1). Profit Philips = 8 × 16/100 × 7/16 × 24/100 = 0.1344
Profit LG = 8 × 18/100 × 4/9 × 20/100 = 0.128
Total profit = 0.1344 + 0.1280 = 0.2624 crore = 26.24 lakh
Answer: c)
2). LCD Samsung = 80000000 × 20/100 × 3/5 = Rs 9600000
LCD Videocon = 80000000 × 10/100 × 2/5 = Rs 32000000
Total cost of production = Rs 12800000
= 1.28 crore
Answer: b)
3). Total profit
= 8 × 24/100 { 3/8 × 35/100 + 5/8 × 20/100}
= 24 × {1.05 + 1} = 24 × 2.05/100 = Rs 0.492 crores

www.bankingpdf.com
Answer: c)
4). (LED cost) Samsung = 8 × 20/100 × 2/5 = 0.64 crore
(LCD profit)Videocon = 8 × 10/100 × 2/5 × 25/100 = 0.08 crore
Required % = 0.08 × 100/0.64 = 12.5%
Answer: c)
5). Profit LCD= 8 × 12/100 × 5/12 × 35/100
Profit LED = 8× 12/100 × 7/12 × 25/100
Ratio = 7 × 25 / 5 × 35 = 1/1 = 1:1
Answer: e)

6).
1994 = ((35-30)/35)*100 = 14.28% fall
1995 = ((40-30)/30)*100 = 33.33% rise
1996 = ((50-40)/40)*100 = 25% rise
1997 = ((55-50)/50)*100 = 10% rise
1998 = ((60-55)/55)*100 = 9.09% rise
1999 = ((60-45)/60)*100 = 25% fall
Ans 1995
Answer: d)
7).
Total imports: Total exports
315: 325
Ans 63:65
Answer: a)
8).
1995-1996
Answer: b)
9).
(145/135)*100 = 107.41
Answer: b)

www.bankingpdf.com
10).
1994 = ((45-40)/40)*100 = 12.5% rise
1995 = ((45-35)/45)*100 = 22.22% fall
1996 = ((40-35)/35)*100 = 14.28% rise
1997 = ((60-40)/40)*100 = 50% rise
1998 = ((60-50)/60)*100 = 16.66% fall
1999 = ((55-50)/50)*100 = 10% rise
Ans – 1999
Answer: a)

690-700 Questions :

1). 24 14 26 ? 28 16 30
a) 13
b) 17
c) 15
d) 11
e) 20

2). 5 6 16 57 ? 1245 7506


a) 240
b) 248
c) 244
d) 250
e) 260

3). 2 9 ? 105 436 2195 13182

www.bankingpdf.com
a) 32
b) 45
c) 40
d) 30
e) 25

4). 11 6 7 12 26 67.5 ?
a) 205.5
b) 195.5
c) 216.5
d) 200.5
e) 222.5

5). 3 ? 113 673 3361 13441 40321


a) 13
b) 14
c) 15
d) 16
e) 17

Directions (Q. 6-10): In each of these questions two equations (I) and (II) are given. You
have to solve both the equations and give answer
a) if p > q
b) if p ≥ q
c) if p < q
d) if p ≤ q
e) if p = q or no relation can be established between p and q.

www.bankingpdf.com
6).
I. p^2 - 26p + 168 = 0
II. q^2 - 25q + 156 = 0

7).
I. 6p - 5q = -47
II. 5p + 3q = 11

8).
I. 2.3p - 20.01 = 0
II. 2.9q - p = 0

9).
I. p = 1764
II. q^2 = 1764

10).
I. p^2 - 13p + 42 = 0
II. q^2 + q - 42 = 0

Answers:
1).c) 2).b) 3).d) 4).a) 5).e) 6).e) 7).c) 8).a) 9).b) 10). b)

SOLUTION:
1). The series consists of two series 1 and 2:

www.bankingpdf.com
Series 1 : 24 26 28 30
+2 +2 +2
Series 2 : 14 15 16
+1 +1
Answer: c)

2). The series is


5 × 1 + 12 = 6; 6 × 2 + 22 = 16; 16 × 3 + 32 = 57; 57
× 4 + 42 = 244; 244 × 5 + 52 = 1245;
1245 × 6 + 62 = 7506.
Answer: b)

3). The series is


(2 + 7) × 1 = 9; (9 + 6) × 2 = 30;
(30 + 5) × 3 = 105; (105 + 4) × 4 = 436;
(436 + 3) × 5 = 2195; (2195 + 2) × 6 = 13182
Answer: d)

4). The series is:


11 × 0.5 + 0.5 = 6; 6 × 1 + 1 = 7; 7 × 1.5 + 1.5 = 12;
12 × 2 + 2 = 26; 26 × 2.5 + 2.5 = 67.5; 67.5 × 3 +
3 = 205.5.
Answer: a)

5). The series is:


3 × 8 – 7 = 17; 17 × 7 – 6 = 113; 113 × 6 – 5 = 673;

www.bankingpdf.com
673 × 5 – 4 = 3361;
3361 × 4 – 3 = 13441; 13441 × 3 – 2 = 40321
Answer: e)

6).
I. p^2 - 26p + 168 = 0
P^2 - 12p - 14p + 168 = 0
p (p - 12) - 14(p - 12) = 0
(p - 12) (p - 14) = 0
p = 12, 14
II. q^2 - 25q + 156 = 0
q^2 - 13q - 12q + 156 = 0
q(q - 13) - 12(q - 13) = 0
(q - 12) (q - 13) = 0
q = 12, 13
Hence, no relation can be established between p and q
Answer: e)

7).
Equation (I) × 3 = 18p - 15q = -141
Equation (II) × 5 = 25p + 15q = 55
43p = -86
P = - 86/43 = 2
5p + 3q = 11
3q = 11 - 5p
3q = 11 + 10
3q = 21
q=7

www.bankingpdf.com
p<q
Answer: c)

8).
I. 2.3p - 20.01 = 0
P = 20.01 / 2.3 = 8.7
II. 2.9q - p = 0
or, p = 2.9q
q = 8.7 /2.9 = 8.7
p>q
Answer: a)

9).
I. p = √1764
p = 42
II. q^2 = 1764
q = + 42
p≥q
Answer: b)

10).
I. p^2 - l3q + 42 = 0
p^2 - 6p - 7p + 42 = 0
p(p - 6) - 7(p - 6) = 0
(p - 6) (p - 7) = 0
p = 6, 7
II. q^2 + q - 42 = 0

www.bankingpdf.com
q^2 + 7q - 6p - 42 = 0
q(q + 7) - 6(q + 7) = 0
(q - 6)(q + 7) = 0
q = 6, - 7
p≥q
Answer: b)

700-710 Questions :

Directions (Q. 1-5): What should come in the place of question mark (?) in the following
questions?

1). 7^3 * √7 * 3√7 * ÷ 7 ÷ 3√(√7) = (7)^?


a) 5/6
b) 5/3
c) 7/6
d) 8/3
e) 7/3

2). 8/15 of 3/17 of 12/13 of 15470 = ?


a) 1242
b) 1344
c) 1464
d) 1504
e) 1624

3). 136% of 105 – 84% of 165 = ?


a) 4.2

www.bankingpdf.com
b) 5.7
c) 6.4
d) 7.8
e) 8.6

4). 22.222 + 2220.2 + 202.20 + 0.0202 – 2.0022 = ?


a) 2244.24
b) 2442.64
c) 2424.44
d) 2444.64
e) 2224.84

5). 1275.42 + 8624.6 – 894.54 – 24.71 = ?


a) 8620.27
b) 8738.57
c) 8890.93
d) 8980.77
e) 9012.17

Directions (Q. 6-10): Refer to the table and answer the given questions.
Total Number of People = 12000
Professions %of People % of Females % of Males

Accounting 12 45 -

Architect - - 50

Singer 11 35 -

www.bankingpdf.com
Scientist - - 75

Dentist 22 25 -

Writer 14 - 80

6). If the total % of people in Architect Profession is 16%. Then the total number of
people in Scientist Profession is approximately what percent of the total number of
people in Singer Profession?
a) 227 %
b) 193 %
c) 168 %
d) 79 %
e) 44 %

7). If the difference between the % of people in Scientist and Architect is 9%. What is
the difference between the total number of males and the total number of females from
all the professions together?
a) 1720
b) 1684
c) 1368
d) 1316
e) None of the Above

8). What is the ratio of the number of males in Dentist profession to the number of
females in Writer profession
a) 17:7
b) 28:55

www.bankingpdf.com
c) 7:11
d) 165:112
e) 165:28

9). If the difference between the % of Accounting and Scientist is 13% then What is the
ratio of the total males in Accounting and Scientist profession to that of females in the
same professions?
a) 161:250
b) 195:256
c) 570:253
d) 507:233
e) 13:19

10). If the total % of people in Scientist Profession is 25%. Then the females in Architect
Profession are approximately what percent of the females in Writer Profession?
a) 276
b) 256
c) 286
d) 246
e) None of the Above

Answers:
1).d) 2).b) 3).a) 4).b) 5).d) 6).a) 7).c) 8).e) 9).d) 10). c)

Solutions:
6). Scientist = 100 – Others = 100 – 75 = 25%
Required Percentage = (25/11)*100 = 227%
Answer: a)

www.bankingpdf.com
7). % of People in Architect = x ; % of People in Scientist = y
x + y = 100% -59% = 41%
x + y = 41
y – x = 9 => y = 25 ; x = 16
Total number of Males = 12000*(12*55 + 16*50 + 11*65 + 25*25 + 22*75 + 14*80) / 100
* 100 = 6684
Total number of Females = 12000 – 6684 = 5316
Difference = 6684 – 5316 = 1368
Answer: c)

8). Ratio = 22*75/ 14*20 = 165:28


Answer: e)

9). [12*55 + 25*75]/[12*45 + 25*25] 507 : 233


Answer: d)

10). [12000*16*50/12000*14*20]*100 = 286


Answer: c)

www.bankingpdf.com
710-720 Questions :

1). In racing over a given distance, A can beat B by 20m, B can beat C by 10m, A can
beat C by 28m. The racing distance is?
a) 58 m
b) 116 m
c) 100 m
d) 120 m
e) cannot be determined

2). A wire can be bent to form a circle of radius 56 cm. if it is instead bent in the form of
a square, then its area will be:
a) 6400 sq.cm
b) 2025 sq.cm
c) 7744 sq.cm
d) 6561 sq.cm
e) none of these

3). A man drives from his house to the station. If he drives at the rate of 10 kms per
hour, he reaches the station at 6 pm. if he drives at 15 kms per hour, he would reach
the station at 4 pm. At what speed, in kms per hour, should he drives so as to reach the
station at 5 pm?
a) 12
b) 12.33
c) 5√5
d) 12.5
e) 13

www.bankingpdf.com
4). The sum of 5 natural numbers A, B, C, D, E is 126. It is known that A:B = 7:8, B:C =
2:3, C:D = 4:5, D:E = 5:7.What is the difference between A and E?
a) 14
b) 22
c) 24
d) 28
e) 32

5). The difference between the amount of compound interest and simple interest
accrued on an amount of 26000 at the end of 3 years is 28994. 134. What is the rate of
interest p.c.p.a.?
a) 22
b) 17
c) 19
d) cannot be determined
e) none of these

Directions (Q. 6-10): In the following two equations numbered I and II are given. you
have to solve both the equations and choose the correct option.
a) x < y
b) x ≥ y
c) x > y
d) x ≤ y
e) x = y or the relationship between x and y cannot be determined

6). I. x2 - 19x + 72 = 0
II. y2 + 23y + 126 = 0

www.bankingpdf.com
7). I. 7x2 + 63x + 126 = 0
II. 9y2 + 36y + 27 = 0

8). I. x7 = 128
II. y10 = 1024

9). I. x3/2 = 252 + 104


II. y1/4 = 42 – 13

10). I. x2 – 5.8x + 8.41 = 0


II. y2 - 3.8y + 3.12 = 0

Answers:

1)c 2)c 3)a 4)d 5)c 6)c 7)d 8)e 9)e 10)c

Solution:
1).Let the uniform speeds of A,B,C be vA,vB,vC, respectively.
Let t1 be the time taken for the race between A and B and t2 be the time taken for race
between B and C
Since A covers the distance in time t1, race between A and C also takes time
t1.Therefore,
VAt1-VBt1 =20
VBt2 –Vct2=10
VAt1 - Vct1=28
Also, VAt1=VBt2 = d. Thus VBt1 =d-20, Vct2=d – 10 and VCt2= d – 10 and VCt1=d-28.
(d-20)/(d-28) = VB/ VC = d/(d-10)
Thus (d-20)(d-10)=d(d-28).Hence, d=100.

www.bankingpdf.com
Answer: c)

2). Length of the wire = circumference of the circle


2 × (22/7) × 56 = 2 × 22 × 8 = 352 cm
side of square = 352/4 = 88cm
area of square = 88 × 88 = 7744 sq.cm
Answer: c)

3). Let ‘x’ be the distance between house and station.


Time taken to travel a distance of x at the speed of 10km/hr, T1 = x/10 hours
Time taken to travel a distance of x at the speed of 15km/hr, T2 = x/15 hours
we know that T1 – T2 = 2 hours
therefore, we get x = 60 km
also, we can conclude that starting time is 12 pm [60/15 = 4, since he reach station at 4
pm travelling at 15 km/hr]
hence, to reach the station at 5 pm, speed = 60/5 = 12 km/hr [12 pm – 5pm, time = 5
hours]
Answer: a)

4). A:B = 7:8


B:C = 2:3 = 8:12
SO, A:B:C = 7:8:12
C:D = 4:5 = 12:15
A:B:C:D = 7:8:12:15
D:E = 5:7 = 15:21
SO, A:B:C:D:E = 7:8:12:15:21
given x(7+8+12+15+21) = 126
or 63x = 126 or x =2
E – A = 21X – 7X = 14X = 28

www.bankingpdf.com
Answer: d)

5). let the annual rate of interest be R%


26000[1+ (R/100)]3 – (26000×3×R) / 100 = 28994.134
Substituting the options in the above equation, rate of interest is 19%
Answer: c)

6). from eqn I. x2 - 19x + 72 = 0


(x - 24) (x + 3) = 0
x = 24, -3
from eqn II. y2 + 23y + 126 = 0
(x + 14) (x + 9) = 0
x = -14, -9
Hence, x > y
Answer: c)

7). from eqn I. 7(x2 + 9x + 18) = 0


(x+6) (x+3) = 0
x = -6, -3
from eqn. II. 9(y2 + 4y + 3) = 0
(y+3) (y+1) = 0
y = -3, -1.
Hence, x ≤ y.
Answer: d)

8). From I, x7 = 27
From II, y10 = 210
Hence x = y

www.bankingpdf.com
Answer: e)

9). From eqn I. x3/2 = 625 + 104 = 729


x = (729)2/3 = 81
from II, y1/4 = 16 – 13 = 3
y = 34 = 81
hence x = y
Answer: e)

10). From l,(x-2.9)2=0


=> x =2.9
From II,(y-1.2)(y-2.6)=0
=>y=1.2,2.6
Hence x > y
Answer: c)

720-730 Questions :

Directions (Q. 1-5): Find the missing number in the following number series:
1).15, 18, 21.6, 25.92, ?
a) 31.104
b) 31.014
c) 31.114
d) 30.114
e) 30.004

2).210, 154, 182, 168, 175,171.5, ?


a) 172.75

www.bankingpdf.com
b) 173.25
c) 172.25
d) 173.75
e) 174.75

3).72, 115, 244, ?, 760, 1147


a) 501
b) 431
c) 473
d) 459
e) 445

4).188, 212, 236, ?, 268, 284


a) 260
b) 256
c) 252
d) 248
e) 244

5).676, 400, 196, 64, 4, ?


a) 4
b) 36
c) 16
d) -4
e) -16

Directions (Q. 6-10): what value will come in place of question mark(?) in the following
questions?

www.bankingpdf.com
6). (512)8 × (1024)2 ÷ (256)? = 418
a) 7
b) 9
c) 5
d) 3
e) 6

7).53 × 47 + 64 × 56 -72 × 68 = ?
a) 1221
b) 1403
c) 1249
d) 1337
e) 1179

8).172 + 212 + 252 + 29 2 = ?


a) 2076
b) 2196
c) 2336
d) 2168
e) 2244

9).57% of 400 + 23% of 700 + 32% of 1100 – 28% of 1300 = ?


a) 401
b) 389
c) 377
d) 364
e) 352

www.bankingpdf.com
10).13.5 × 14 + 22.5 × 18 – 4.7 × 30 = ?
a) 405
b) 423
c) 435
d) 453
e) 481

Answers:
1)a 2)b 3)d 4)e 5)c 6)a 7)e 8)b 9)c 10)d

Solution:

1).The series is a geometric progression with a common ratio 1.2. Hence the next term
is 25.92 × 1.2 = 31.104
Answer: a)

2).Starting from the third term, every term is the average of the previous two terms.
Hence the next term is (175 + 171.5)/ 2 = 173.25
Answer: b)

3). The pattern is:


72+43×1=115, 115+43×3=244, 244+43×5=459, 459+43×7=760, 760+43×9=1147
Answer: d)

4).The pattern is 47×4=188, 53×4=212, 59×4=236, 61×4=244, 67×4=268, 71×4=284


47, 53, 59, 61, 69 and 71 are consecutive prime numbers.
Answer: e)

www.bankingpdf.com
5). The pattern is 262, 202, 142, 82, 22, (-4)2
Answer: c)

6).(29)8 × (210)2 ÷(28)?=236


=>272 × 220 ÷ 28×? = 236
=>28×? = 272+20-36=256
=> ? = 7
Answer: a)

7). ?=(50+3)(50-3)+(60+4)(60-4)-(70+2)(70-2)
=502 - 32 + 602 - 42 -702 + 22
=2500 – 9 +3600 -16 - 4900 + 4
=1179
Answer: e)

8). ? = 289 +441 +625 + 841=2196


Answer: b)

9). ? = 57×4 + 23×7 + 32×11 - 28×13


= 228 + 161 + 352 – 364 = 377
Answer: c)

10). ?=189+405-141 = 453


Answer: d)

www.bankingpdf.com
730-740 Questions :

Directions (Q. 1-5): The following bar-graph shows the number of adult Males and
Females of six cities and the line graph shows percentage of adult Males and Females
who voted in the last election:

www.bankingpdf.com
1). The total number of Females from Chennai and Hyderabad together who voted in
the last election is what percentage of the total number of Males from Chennai who
voted in the last election?
a) 75%
b) 80%
c) 90%
d) 120%
e) 150%

2). What is the total number of Females from all the six cities together who voted in the
last election?
a) 229060
b) 229160
c) 229260
d) 229360
e) 229460

www.bankingpdf.com
3). The total number of Females from Agra who voted in the last election is what
percentage less than the total number of Males from the same city who voted in the last
election?
a) 72%
b) 60%
c) 45%
d) 30%
e) 25%

4). In which pair of cities are the numbers of Males who voted in the last election equal?
a) Chennai and Mumbai
b) Mumbai and Hyderabad
c) Hyderabad and Bangalore
d) Chennai and Hyderabad
e) Mumbai and Bangalore

5). What is the difference between the total number of Males and the total number of
Males who voted in the last election?
a) 121750
b) 122850
c) 123740
d) 124550
e) None of these

Directions (Q. 6-10): In each of these questions, two equations (I) and (II) are given.
You have to solve both the equations and give answer

www.bankingpdf.com
a) if x > y
b) if x ≥ y
c) if x < y
d) if x ≤ y
e) if x = y or if there is no relation between ‘x’ and ‘y’.
6).
I. 3x2 - 23x + 40 = 0
II. 2y2 - 23y + 66 = 0

7).
I. x2 + 5x - 6 = 0
II. 2y2 - 11y + 15 = 0

8).
I. x2 + 42 = 13x
II. y = 4√1296

9).
I. x2 + x - 2 = 0
II. y2 + 7y + 12 = 0

10).
I. 15x2 - 46x + 35 = 0
II. 4y2 - 15y + 14 = 0

Answers:
1)e 2)a 3)e 4)b 5)b 6)c 7)c 8)b 9)a 10)c

www.bankingpdf.com
Solution:
1). Female (Chennai + Hyderabad) = 48000 + 36000 = 84000
Male Chennai = 56000
Required per cent = 84000 / 56000 × 100 = 150%
Answer: e)

2). Total Females = 64000 × 0.75 + 50000 × 0.72 + 72000 × 0.5 + 80000 × 0.65 +
72000 × 0.48 + 25000 × 0.9
= 48000 + 36000 + 36000 + 52000 + 34560 + 22500 = 229060
Answer: a)

3). Male Agra = 40000 × 75/100 = 30000


Female Agra= 25000 × 90/100 =22500
Required per cent = 30000 – 22500 / 30000 × 100 = 25%
Answer: e)

4). Male Mumbai = 48000 × 0.70 = 33600


Male Hyderabad = 60000 × 0.56 = 33600
Answer: b)

5). Total Males = 70 + 48 + 60 + 56 + 75 + 40 = 349 thousand


Total Male voters = 70 × 0.8 + 48 × 0.7 + 60 × 0.56 + 56 × 0.7 + 75 × 0.45 + 40 × 0.75
= 56 + 33.6 + 33.6 + 39.2 + 33.75 + 30
= 226.15 thousand
Difference = 349 - 226.15
= 122.85 thousand

www.bankingpdf.com
Answer: b)

6).
I. 3x2 - 23x + 40 = 0
or 3x2 - 15x - 8x + 40 = 0
or 3x(x - 5) - 8(x - 5) = 0
or (3x - 8) (x - 5) = 0
x = 5,8/3
II. 2y2 - 23y + 66 = 0
or 2y2 - 12y - 11y + 66 = 0
or 2y (y - 6) -11 (y - 6) = 0
or (y - 6)(2y - 11) = 0
y = 6, 11/ 2
x<y
Answer: c)

7).
I. x2 - x + 6x -6 = 0
or x(x - 1) + 6(x - 1) = 0
or (x - 1) (x + 6) = 0
x = 1, -6
II. 2y2 - 6y - 5y + 15 = 0
or 2y(y - 3) - 5(y - 3) = 0
or (y - 3) (2y - 5) = 0
y = 3, 5/2
Answer: c)

8).
I. x2 + 42 = 13x

www.bankingpdf.com
or x2 - 13x + 42 = 0
or x2 - 7x - 6x + 42 = 0
or x(x - 7) - 6(x - 7) = 0
or (x - 6) (x - 7) = 0
x = 6, 7
II. y = 4√1296
y=6
x≥y
Answer: b)

9).
I. x2 + x - 2 = 0
or x2 + 2x - x - 2 = 0
or x(x + 2) - 1(x + 2) = 0
or (x - 1) (x + 2) = 0
x = 1, - 2
II. y2 + 7y + 12 = 0
or y2 + 3y + 4y + 12 = 0
or y(y + 3) + 4(y + 3) = 0
or (y + 3) (y + 4) = 0
y = -3, -4
x>y
Answer: a)

10).
I. 15x2 - 25x - 21x + 35 = 0
or 5x(3x - 5) - 7(3x - 5) = 0
or (5x - 7) (3x - 5) = 0
x = 7/5, 5/3

www.bankingpdf.com
II. 4y2 - 8y - 7y + 14 = 0
or 4y(y - 2) - 7(y - 2) = 0
or (4y - 7) (y - 2) = 0
y = 2, 7/4
x<y
Answer: c)

740-750 Questions :

Directions (Q.1-5): Based on the data given in the table, answer the following questions:
The price of 6 metals across 5 cities is given below.
Price of Gold in the table is per 8g, Price of Silver are per 16g, and other prices are per
1g.

1). If the “Metal Index” of a city is defined as the combined cost of 1g of Gold, 3g of
Silver and 10g of copper, which city has the highest Metal Index?

www.bankingpdf.com
a) Chennai
b) Bangalore
c) Mumbai
d) Delhi
e) Kolkata

2). What is the difference between average price of 1g of Gold across all 5 Cities
compared to 1g of Silver across all 5 cities? (Average Price of Metal = Combined cost
of 1g of metal in all 5 cities/5).
a) 242.54
b) 283.76
c) 265.95
d) 255.98
e) 270.23

3). A tradesman in Delhi accepts Gold and returns Iron in return by taking 10%
margins.(So if gold costing Rs.100 is given to him, he return 90 Rs of Iron). How many
grams of Iron will a customer get on trading 45g of Gold In Delhi?
a) 691
b) 650
c) 552
d) 760
e) 730

4). If 1g of copper can be used in place of 0.8g of Aluminium in all the cities, Which city
will save the most on replacing 100kg of Aluminium by Copper?
a) Chennai
b) Bangalore
c) Mumbai
d) Delhi

www.bankingpdf.com
e) Kolkatta

5). How much will it cost for a customer who wishes to buy 4g of Gold, 6g of Silver and
50g each of Iron and Copper in Delhi?
a) 8523
b) 7137
c) 7752
d) 8160
e) 8245

Direction (Q.6-10): Study the following graph carefully to answer the Questions that
follow.
AMOUNT EARNED (IN THOUSANDS) BY THREE DIFFERENT PERSONS
IN SIX DIFFERENT MONTHS

6). What was the percentage increase in amount earned by persons-C in


March as ‘compared to the previous month?

www.bankingpdf.com
a) 145
b) 150
c) 125
d) 140
e) None of these

7). What was the average amount earned by Person-A in all months together?
a) Rs 8,520
b) Rs 9,520
c) Rs 8,500
d) Rs 9,500
e) None of these

8). What was the respective ratio between the amount earned by Person-C In May, the
amount earned by Person-A in March and amount earned by Person-B in April?
a) 6 : 7 : 2
b) 7 : 2 : 6
c) 6 : 7 : 4
d) 5 : 7 : 6
e) 5 : 7 : 3

9). What was the difference between the total amount earned by Person-B over All
the months together and the amount earned by per-son-C in April?
a) 41,000
b) 3.1 lacs
c) 48,000
d) 31,000
e) None of these

www.bankingpdf.com
10). Total amount earned by all the three persons together in January was
Approximately what percentage of the total amount earned by all the three Persons
together in March ?
a) 62
b) 81
c) 68
d) 72
e) 78

Answers:
1)a 2)c 3)a 4)c 5)b 6)b 7)d 8)a 9)d 10)c

Solution:
Directions (Q. 1-5):

1). The price per 1g of each metal in all cities is shown below:
Metal index

www.bankingpdf.com
From the table, we see that Chennai has the highest Metal Index.
Answer: a)

2). Average Price of Gold =(432.5+412.5+300+307+298.5)/5=1750.5/5=350.1


Average Price of Silver =(75+71.75+84.5+101.5+88)/5=420.75/5=84.15
Difference =350.1 – 84.15 =265.95
Answer: c)

3). 45g of gold in Delhi = 45 × 307 =13815


After margins, its worth 90% of 13815 = 12433.5
Iron returned =12433.5 /18= 690.75g ~=691g.
Answer: a)

4).

From the table, we have that Mumbai saves the most per 0.8g of aluminium, which
means it will save the most per 100kg as well.

www.bankingpdf.com
Answer: c)

5). Cost = 4 × 307 + 6 × 101.5 + 50×(18+88)=7137


Answer: b)

6). Amount earned by Person-C in February =4000


Amount earned by Person-C in March =10000
Percentage increase =[(10000 – 4000)/4000] ×100=150%
Answer: b)

7). Total amount earned by person A in all months


together=(8+12+14+3+9+11)thousands =57000
Average =57000/6 =Rs 9500
Answer: d)

8). the amount earned by Person –C in May =12000


The amount earned by Person-A in March =14000
Amount earned by Person-B in April=4000
Ratio =12 : 14 : 4 =6 :7:2
Answer: a)

9). The total amount earned by Person-B over all the months together
=(4+9+7+4+6+12)thousands=42000
The amount earned by person-C in April =11000
Difference =42000-11000=31000
Answer: d)

www.bankingpdf.com
10).Total amount earned by all the three persons together in January =8000 + 4000 +
9000 =21000
The total amount earned by all the three persons together in March =14000 + 7000 +
10000 =31000
Required percentage=21000/31000 × = 67.74%

Answer: c)

750-760 Questions :

Directions (Q. 1-5):


Revenue and Profits (in Lakhs of Rs) of 2 companies A and B over a 6 year period is
shown below.
Expenditure =Revenue – Profits

www.bankingpdf.com
1).What was the total expenditure (in lakhs) of both the companies over the 6 years?
a) 200
b) 250
c) 225
d) 240
e) 285

2).Which year did company A have the highest expenditure percentage?


(expenditure percentage =expenditure × 100 / revenue)
a) 2012
b) 2010
c) 2009
d) 2011
e) 2013

3).over the 6 year period, which year showed the highest percentage increase in profits
compared to the previous year for company B?
a) 2010
b) 2011
c) 2009
d) 2013
e) 2012

4).In which year was Profits as a percentage of Revenue was highest for companies A
& B together?
a) 2008
b) 2010

www.bankingpdf.com
c) 2012
d) 2011
e) 2009

5).In 2014, If revenue went up by 40% and expenditure went up 30% for company A
compared to 2013,what is the profit percentage in 2014 for company A?
a) 45%
b) 60%
c) 50%
d) 65%
e) 55%

6).A car is filled with fifteen and half gallons of fuel for a round trip journey. The forward
and return journey’s are completed by taking two different paths. The mileage ratio
during the forward and return journey’s is 3:4.And the distance ratio of the forward and
return journey is 4:5.What is the fuel consumed during the return journey?
a) 4.5 gallons
b) 6 gallons
c) 7.5 gallons
d) 9 gallons
e) None of these

7).Arun covers a total distance of 500 km by Bus, Train and Bike in the ratio of 3:4:2
respectively. The speeds of Bus, Bike and Train are in the ratio of 3:2:4. If the total
duration of the journey was 25 hours then find out the ratio of the time taken by Bus,
Train and Bike to cover the distance?
a) 1:2:3
b) 2:3:4
c) 2:5:8

www.bankingpdf.com
d) 3:5:8
e) None of the above

8).An amount at 5 % compound interest for the 1st 2 years,followed by simple interest of
10% for the subsequent 3 years becomes Rs.2579.85. Find the initial amount.
a) 1200
b) 1400
c) 1600
d) 1800
e) 2000

9).A tea merchant has 3 varieties of Tea. The first variety, called Mojo costs Rs.200per
kg. The 2ndvariety called Cojo costs Rs.250 per kg,and the third called Bojo costs
Rs.300per kg. He takes 10 kg each of Mojo and Cojo.He removes 5 kg of the mixture
and replaces it with 5 kg of Bojo. He then removes 8 kg of the mixture and replaces it
with 6 kg of Mojo and 2 kg of Bojo. What is the average price per kg of the 20 kg
mixture now?
a) Rs.212.5
b) Rs.221.25
c) Rs.231.75
d) Rs.236.25
e) None of the above

10).A rectangular sheet of paper, when halved by folding it at mid-point of its longer
side, results in a rectangle, whose longer and shorter sides are in the same proportion
as the longer and shorter sides of the original rectangle. If the shorter side of the original
rectangle is 2, what is the area of the smaller rectangle?
a) 4√3
b) 2√3

www.bankingpdf.com
c) 2√2
d) √2
e) None of the above

Answers:
1)b 2)b 3)c 4)e 5)b 6)c 7)e 8)d 9)d 10)c

Solution:
Directions (Q. 1-5):

1). Total = 105 + 145 = 250 Lakhs


Answer: b)

2). We see that expenditure percentage is maximum in 2010.

www.bankingpdf.com
Answer: b)
3). From the table, we see highest percentage increase was in 2009 compared to 2008.

Answer: c)

4). We see in 2009, the value is the highest.

Answer: e)

5). 2014 Revenue = 1.4 × 70 = 98


2014 Expenditure = 1.3 × 30 =39
Profits = 98-39=59
Percentage = 59 × 100/98~ = 60%
Answer: b)

6).Total fuel=15.5 gallons


Ratio of mileage = 3 : 4 ,Ratio of Distance = 4 : 5
Let the distances of forward and return journey’s be 4x and 5x respectively.

www.bankingpdf.com
So fuel consumed will be : ( 4x/3) + ( 5x/4) =15.5
Solving, x=6
So return journey consumption =30/4 =7.5 gallons.
Answer: c)

7).Distance on bus, train,bike be 3a,4a,2a respectively.


Speeds of the bus,bike, train be 3b,2b,4b respectively.
Time on bus =3a/3b
Time on bike=2a/2b
Time on train=4a/4b=a/b
So required ratio =1:1:1
Note: The speeds mentioned are in the order Bus,Bike and Train, while distance is in
the order Bus, Train and Bike.
Answer: e)

8).Let the initial amount be A


Amount with CI for 2 years = A × 1.05 ×1.05
Amount with SI for 3 years on this =A × 1.05 ×1.05(1+3×0.1)=A × 1.05 ×1.05×1.3
Given: A ×1.05×1.05×1.3=2579.85
Or A=2579.85/(1.05×1.05×1.3)=Rs.1800
Answer: d)

9).Initially:10 Mojo + 10 Cojo


Step 1: 5 kg of mixture removed : So we will have 7.5 kg Mojo,cojo and 5 kg of Bojo
Step 2:8 kg of mixture removed : So,3 kg of Mojo and cojo will be removed, 2 kg of Bojo
New Mojo =7.5 - 3 + 6 =10.5 kg
New Cojo=7.5 - 3 = 4.5kg
New Bojo =5 - 2 + 2 = 5 kg
Price =(10.5× 200+4.5×250+5×300)/20 = Rs.236.25

www.bankingpdf.com
Answer: d)

10).Let longer side of larger rectangle=a


Initial ratio of larger to shorter side=a/2
Ratio of longer to shorter side in case of second rectangle =2/(a/2)=4/a
We are given that 4/a=a/2
So, a=2√2
Area of smaller rectangle = 2 × a√2=a=2√2

Answer: c)

760-770 Questions :

Directions (Q. 1-5): What will come in place of the question mark (?) in the following
series?
1). 104 153 189 214 ?
a) 239
b) 225
c) 264
d) 235
e) None of these

2). 12 22 69 272 1365 ?


a) 8196
b) 8184
c) 8195
d) 6830
e) None of these

www.bankingpdf.com
3). 15 17 21 29 45 77 ?
a) 109
b) 125
c) 141
d) 173
e) None of these

4). 1 ? 27 64 125
a) 14
b) 4
c) 9
d) 8
e) None of these

5). 15 17 32 49 81 130 ?
a) 179
b) 211
c) 194
d) 226
e) None of these

Directions for Questions (6-10): The questions are based on the table and pie-chart
given below. The table has some missing information but it is known that every
shopkeeper sold books. Complete the missing information and answer the questions
using the information given in table and pie-chart.

Books sold by 6 shopkeepers in June 2016 (in thousands)

www.bankingpdf.com
6). If a total of 4000 books were returned in June 2016, then what was the number of
Economics books returned as a percentage of Economics books sold in June 2016?
a) 2.32%
b) 2.74%
c) 1.86%
d) 1.44%
e) 2.92%

www.bankingpdf.com
7). What is the average number of books sold of Commerce, Computer and Economics
together?
a) 29500
b) 30000
c) 30333
d) 28333
e) None of these

8). If the number of Commerce books returned in June 2016 is 780, then what is the
approximate number of English and Marketing books sold as a ratio of the number of
English and Marketing books returned in June 2016?
a) 33:2
b) 41:2
c) 47:3
d) 53:4
e) 65:3

9). In June 2016, the second highest number of books sold was of which subject?
a) Marketing
b) Commerce
c) Business Maths
d) Economics
e) Computer

10). Which shopkeeper sold maximum number of Commerce books as a percentage of


the total number of books sold by that shopkeeper?
a) C

www.bankingpdf.com
b) D
c) A
d) F
e) None of these

Answers:
1)e 2)b 3)c 4)d 5)b 6)a 7)e 8)b 9)d 10)b

Solution:
1). 104 + 72 = 153
153 + 62 = 189
189 + 52 = 214
214+ 42 = 230
Answer: e)

2). 12 × 2 - 2 = 22
22 × 3 + 3 = 69
69 × 4 - 4 = 272
272 × 5 + 5 = 1365
1365 × 6 - 6 = 8184
Answer: b)

3). 15 + 2 = 17
17 + 4 = 21
21 + 8 = 29
29 + 16 = 45
45 + 32 = 77
77 + 64 = 141

www.bankingpdf.com
Answer: c)

4). The series is


11, 22, 33, 44, 55
Hence, the question mark will be replaced by 4.
Answer: d)

5). 15 + 17 = 32
17 + 32 = 49
32 + 49 = 81
49 + 81 = 130
81 + 130 = 211
Answer: b)

Directions (Q. 6-10):

6). Number of Economics books returned = 18% of 4000 = 720


Required percentage = (720/31000)× 100 = 2.32% 720/31000
Answer: a)

www.bankingpdf.com
7). Required average = (27+29+31)/3= 29
= 29000
Answer: e)

8). Number of Commerce books returned = 780


Total number of books returned = 780/13 × 100 = 6000
Number of English and Marketing books returned = 37% of 6000 = 2220
Required ratio = 46000 : 2220 = 2300 : 111 = 41 : 2.
Answer: b)

9). Second highest number of books sold was Economics


Answer: d)

10). We need to check for which shopkeeper (s) the number of Commerce books sold
were the highest, for that shopkeeper the percentage of Commerce books sold as a
percentage of total number of books sold by that shopkeeper would be higher than for
the shopkeeper who has not sold maximum number of Commerce books out of all the
books sold, there is only one shopkeeper is D.
Answer: b)

www.bankingpdf.com
770-780 Questions :

1).In a class of 24 students, each student shakes hand with everyone els. In another
class of 32 students, each student shakes hand with all other students. What is the
difference between total number of hand shakes in the 2 classes?
a) 180
b) 200
c) 22
d) 240
e) 260
2).Which of the following series of discounts is the best for a customer?
(A) Successive discounts of 10% ,20%,30%
(B) Successive discounts of 20%,20%,20%
(C) Successive discounts of 15%,15%,15%,15%
(D) Successive discounts of 20%,40%
a) A
b) B
c) C
d) D
e) A and B
3).A,B and C can do piece of work in 11 days,20 days and 55 days respectively,
working alone. How soon can the work be done if a is assisted by B and C on
alternate days/
a) 6
b) 7
c) 8
d) 9
e) 10

www.bankingpdf.com
4).A,B are start walking around a Circular park of radius 70m. They start at the same
point, and A goes clockwise at 10m/s, while b goes anti-clockwise at 20m/s.How
many times will they cross each other at the starting point if they walk for 30
minutes?
a) 20
b) 40
c) 60
d) 80
e) None of the above
5).Ram is 3 times as old as his brother. Ram’s father is 3 times older than Ram. In 9
years , Ram will be 1.5 times the age of his brother. What will be the age of Ram’s
father when Ram is 1.5 times the age of his brother?
a) 27
b) 36
c) 45
d) 54
e) 63

Directions (6-10):Each of the questions below consists of a question and two


statements numbered I and II given below it.You have to decide whether the data
provided in the statements are sufficient to answer the question. Read both the
statements.
6).Is it more profitable for Company M to produce Q?
I. Product R sells at a price four times that of Q
II. One unit of Q requires 2 units of labour, while one unit of R requires 5 units of
labour. There is a no other constraint on production.
If the question can be answered with the help of statement I alone,
If the question can be answered with the help of statement II alone,
If both,statement I and statement II are needed to answer the question

www.bankingpdf.com
If the question cannot be answered even with the help of both the statements.
If the data in either statement I or statement II alone is sufficient to answer the
question
7). A man distributed 43 chocolates to his children. How many of his children are
more than five years old?
I. A child older than five years gets 5 chocolates
II. A child 5 years or younger in age gets 6 chocolates.
If the question can be answered with the help of statement I alone,
If the question can be answered with the help of statement II alone,
If both,statement I and statement II are needed to answer the question
If the question cannot be answered even with the help of both the statements.
If the data in either statement I or statement II alone is sufficient to answer the
question
8).What is the radius of the circle/
I.Ratio of its area to circumference is >7.
II.Diameter of the circle is ≤ 32.
If the question can be answered with the help of statement I alone,
If the question can be answered with the help of statement II alone,
If both,statement I and statement II are needed to answer the question
If the question cannot be answered even with the help of both the statements.
If the data in either statement I or statement II alone is sufficient to answer the
question.
9).X says to Y,”I am 3 times as old as you were 3 years ago”.How old is X?
I. Y ‘s age 17 years from now is same as X’s present age.
II. X’s age nine years from now is 3 times Y’s present age.
If the question can be answered with the help of statement I alone,
If the question can be answered with the help of statement II alone,
If both,statement I and statement II are needed to answer the question
If the question cannot be answered even with the help of both the statements.

www.bankingpdf.com
If the data in either statement I or statement II alone is sufficient to answer the
question
10). What is the value of prime number x?
I. X2 + x is a two digit number greater than 50
II. X3 is a three digit number.
If the question can be answered with the help of statement I alone,
If the question can be answered with the help of statement II alone,
If both,statement I and statement II are needed to answer the question
If the question cannot be answered even with the help of both the statements.
If the data in either statement I or statement II alone is sufficient to answer the
question

Answers:
1). c) 2). d) 3). c) 4). b) 5). c) 6). c) 7). c) 8). d) 9). a) 10). a)

Solution:
1).1 st class : Total Hand shakes = 24 × 23 /2 = 12 × 23 =276
2nd class :Total Hand shakes =32 × 31 / 2 = 16 × 31 = 496
Difference =220
Answer : c
2).consider the price as 100
A = 100 × 0.9 × 0.8 × 0.7 =50.4
B = 100 × 0.8 × 0.8 × 0.8=51.2
C = 100 × 0.85 × 0.85 × 0.85 ×0.85 ~ =52.2
D = 100 × 0.8 × 0.6 =48
We see that the maximum discount of 52% (100-48) is on D, which is the best for a
customer.
Answer : d
3).Work done by A and B in one day = 1/11 + 1/20 =31/220
Work done by A and C in one day = 1/11 + 1/55 =6/55

www.bankingpdf.com
Work done in 2 days = 31 /220+6/55=55/220 = ¼
Hence, 1/4 of the work is done in 2 days
Hence, it takes 8 days to finish the work
Answer : c
4).Radius = 70m, So circumference = 2 × (22/7) × 70 =440m
Relative speed = 10 +20 = 30m/s
They will meet everytime at the starting point when the distance covered by both put
together is a multiple of 440
LCM of (440,30) = 44 × 30
So, they will meet at starting point every 44 seconds.
In 30minutes, they will meet =1800/44 =40.9
So they will meet 40 times (we need to ignore the decimal part).
Answer : b
5). Let the age of Ram be R, brother be B
R=3B
And (R + 9 ) =1.5(B + 9)
3B + 9 =1.5B + 13.5
1.5B =4.5 or B = 3 years R= 9 years.
Father current age = 4R(Since father is 3 times older than Ram, his age is R + 3 R,
and not 3R)
Father age when Ram is 1.5 times the age of his brother = 4R + 9 = 45.
Answer : c
6). from statement I :
Let the price of product Q be ‘P’
Then the price of product R is 4P
From Statement II:
Cost of product Q =2
Cost of product R=5
Therefore,

www.bankingpdf.com
Profit percentage of product Q = (P-2 ) × 100/2 =50(P-2)=50P – 100
Profit percentage of product R = (4P -5) × 100/5=20(4P-5)=80P-100
Hence, product R is more profitable.
Both statements are required to answer the question
Answer :c
7).From both statement I and II:
Let the number of chocolates received by child older than 5 years be x and that of
for child than 5 years or younger be y
Therefore,
5x + 6 y = 43
Since x and y has to be natural numbers, the solution for the above equation will be
x=5 and y=3
Hence, 5 children are older than 5 years
Answer : c
8). Let the radius of the circle be r units
From statement 1:
[πr^2]/ [ 2 πr ] > 7 or r > 14
From statement II:
2r≤32
R ≤ 16
Hence, r can be 15 or 16 units
Hence,
Answer : d
9).Let the current ages of X and Y be a and b respectively
From the question:
A=3(a-b)
From statement I:
A=b + 17
B = a-17

www.bankingpdf.com
Therefore, a = 3(a-17-3)
a=3a-60
2a=60
A=30
Hence, X is 30 years old
From statement II:
a + 9 = 3b
b=(a+9)/3
Substituting this in first equation,
A=a+9-9
Hence, no result
Therefore, statement I alone is sufficient
Answer :a
10). From statement I :
(x2 +x) should be less than 100 and greater than 50
This is possible only for 3 numbers i.e.7,8,9
72 + 7 =56
82 + 8 = 72
92 + 9 = 90
But only 7 is prime number. Hence,x=7
From statement II:
The value of x can 5 or 7, since both 53 and 73 is a three digit number
Hence, only statement I alone is sufficient to answer the question
Answer : a

www.bankingpdf.com
780-790 Questions :

1).In a class of 24 students, each student shakes hand with everyone els. In another
class of 32 students, each student shakes hand with all other students. What is the
difference between total number of hand shakes in the 2 classes?
a) 180
b) 200
c) 22
d) 240
e) 260
2).Which of the following series of discounts is the best for a customer?
(A) Successive discounts of 10% ,20%,30%
(B) Successive discounts of 20%,20%,20%
(C) Successive discounts of 15%,15%,15%,15%
(D) Successive discounts of 20%,40%
a) A
b) B
c) C
d) D
e) A and B
3).A,B and C can do piece of work in 11 days,20 days and 55 days respectively,
working alone. How soon can the work be done if a is assisted by B and C on
alternate days/
a) 6
b) 7
c) 8
d) 9
e) 10

www.bankingpdf.com
4).A,B are start walking around a Circular park of radius 70m. They start at the same
point, and A goes clockwise at 10m/s, while b goes anti-clockwise at 20m/s.How
many times will they cross each other at the starting point if they walk for 30
minutes?
a) 20
b) 40
c) 60
d) 80
e) None of the above
5).Ram is 3 times as old as his brother. Ram’s father is 3 times older than Ram. In 9
years , Ram will be 1.5 times the age of his brother. What will be the age of Ram’s
father when Ram is 1.5 times the age of his brother?
a) 27
b) 36
c) 45
d) 54
e) 63

Directions (6-10):Each of the questions below consists of a question and two


statements numbered I and II given below it.You have to decide whether the data
provided in the statements are sufficient to answer the question. Read both the
statements.
6).Is it more profitable for Company M to produce Q?
I. Product R sells at a price four times that of Q
II. One unit of Q requires 2 units of labour, while one unit of R requires 5 units of
labour. There is a no other constraint on production.
If the question can be answered with the help of statement I alone,
If the question can be answered with the help of statement II alone,
If both,statement I and statement II are needed to answer the question

www.bankingpdf.com
If the question cannot be answered even with the help of both the statements.
If the data in either statement I or statement II alone is sufficient to answer the
question
7). A man distributed 43 chocolates to his children. How many of his children are
more than five years old?
I. A child older than five years gets 5 chocolates
II. A child 5 years or younger in age gets 6 chocolates.
If the question can be answered with the help of statement I alone,
If the question can be answered with the help of statement II alone,
If both,statement I and statement II are needed to answer the question
If the question cannot be answered even with the help of both the statements.
If the data in either statement I or statement II alone is sufficient to answer the
question
8).What is the radius of the circle/
I.Ratio of its area to circumference is >7.
II.Diameter of the circle is ≤ 32.
If the question can be answered with the help of statement I alone,
If the question can be answered with the help of statement II alone,
If both,statement I and statement II are needed to answer the question
If the question cannot be answered even with the help of both the statements.
If the data in either statement I or statement II alone is sufficient to answer the
question.
9).X says to Y,”I am 3 times as old as you were 3 years ago”.How old is X?
I. Y ‘s age 17 years from now is same as X’s present age.
II. X’s age nine years from now is 3 times Y’s present age.
If the question can be answered with the help of statement I alone,
If the question can be answered with the help of statement II alone,
If both,statement I and statement II are needed to answer the question
If the question cannot be answered even with the help of both the statements.

www.bankingpdf.com
If the data in either statement I or statement II alone is sufficient to answer the
question
10). What is the value of prime number x?
I. X2 + x is a two digit number greater than 50
II. X3 is a three digit number.
If the question can be answered with the help of statement I alone,
If the question can be answered with the help of statement II alone,
If both,statement I and statement II are needed to answer the question
If the question cannot be answered even with the help of both the statements.
If the data in either statement I or statement II alone is sufficient to answer the
question

Answers:
1). c) 2). d) 3). c) 4). b) 5). c) 6). c) 7). c) 8). d) 9). a) 10). a)

Solution:
1).1 st class : Total Hand shakes = 24 × 23 /2 = 12 × 23 =276
2nd class :Total Hand shakes =32 × 31 / 2 = 16 × 31 = 496
Difference =220
Answer : c
2).consider the price as 100
A = 100 × 0.9 × 0.8 × 0.7 =50.4
B = 100 × 0.8 × 0.8 × 0.8=51.2
C = 100 × 0.85 × 0.85 × 0.85 ×0.85 ~ =52.2
D = 100 × 0.8 × 0.6 =48
We see that the maximum discount of 52% (100-48) is on D, which is the best for a
customer.
Answer : d
3).Work done by A and B in one day = 1/11 + 1/20 =31/220
Work done by A and C in one day = 1/11 + 1/55 =6/55

www.bankingpdf.com
Work done in 2 days = 31 /220+6/55=55/220 = ¼
Hence, 1/4 of the work is done in 2 days
Hence, it takes 8 days to finish the work
Answer : c
4).Radius = 70m, So circumference = 2 × (22/7) × 70 =440m
Relative speed = 10 +20 = 30m/s
They will meet everytime at the starting point when the distance covered by both put
together is a multiple of 440
LCM of (440,30) = 44 × 30
So, they will meet at starting point every 44 seconds.
In 30minutes, they will meet =1800/44 =40.9
So they will meet 40 times (we need to ignore the decimal part).
Answer : b
5). Let the age of Ram be R, brother be B
R=3B
And (R + 9 ) =1.5(B + 9)
3B + 9 =1.5B + 13.5
1.5B =4.5 or B = 3 years R= 9 years.
Father current age = 4R(Since father is 3 times older than Ram, his age is R + 3 R,
and not 3R)
Father age when Ram is 1.5 times the age of his brother = 4R + 9 = 45.
Answer : c
6). from statement I :
Let the price of product Q be ‘P’
Then the price of product R is 4P
From Statement II:
Cost of product Q =2
Cost of product R=5
Therefore,

www.bankingpdf.com
Profit percentage of product Q = (P-2 ) × 100/2 =50(P-2)=50P – 100
Profit percentage of product R = (4P -5) × 100/5=20(4P-5)=80P-100
Hence, product R is more profitable.
Both statements are required to answer the question
Answer :c
7).From both statement I and II:
Let the number of chocolates received by child older than 5 years be x and that of
for child than 5 years or younger be y
Therefore,
5x + 6 y = 43
Since x and y has to be natural numbers, the solution for the above equation will be
x=5 and y=3
Hence, 5 children are older than 5 years
Answer : c
8). Let the radius of the circle be r units
From statement 1:
[πr^2]/ [ 2 πr ] > 7 or r > 14
From statement II:
2r≤32
R ≤ 16
Hence, r can be 15 or 16 units
Hence,
Answer : d
9).Let the current ages of X and Y be a and b respectively
From the question:
A=3(a-b)
From statement I:
A=b + 17
B = a-17

www.bankingpdf.com
Therefore, a = 3(a-17-3)
a=3a-60
2a=60
A=30
Hence, X is 30 years old
From statement II:
a + 9 = 3b
b=(a+9)/3
Substituting this in first equation,
A=a+9-9
Hence, no result
Therefore, statement I alone is sufficient
Answer :a
10). From statement I :
(x2 +x) should be less than 100 and greater than 50
This is possible only for 3 numbers i.e.7,8,9
72 + 7 =56
82 + 8 = 72
92 + 9 = 90
But only 7 is prime number. Hence,x=7
From statement II:
The value of x can 5 or 7, since both 53 and 73 is a three digit number
Hence, only statement I alone is sufficient to answer the question
Answer : a

www.bankingpdf.com
790-800 Questions :

Directions (Q. 1-5): In the following questions three equations numbered I,II and III are
given. Solve the equations and choose the correct option that gives the relation
between the variables.
a) x<y =z
b) x≤ y<z
c) x<y>z
d) x=y>z
e) None of these

1). I.z2 + z – 56 = 0
II.7y + 2x = 81
III.3x + y = 36

2).I.3x + 4y + 5z = 87
II.3x + z = 27
III. x + 3y =29

3). I.x2 + 3x +2 = 0
II.y2 + 8y + 15 = 0
III.z2 + 13z + 42 = 0

4).I. x= - √6.25
II. y 2 + 0.7y – 4.5 = 0
III.3x + 7z = 7.9

www.bankingpdf.com
5).I. x + 2y + z =23
II.3x + 5z = 35
III.y+2z =15

Directions (Q. 6-10): Find the missing number in the following number series:
6). 410,405,396,378,360,?
a) 351
b) 345
c) 342
d) 350
e) 339

7). 1/9, 4/9, 1 7/9, 7 1/9, 28 4/9, ?


a) 112(1/9)
b) 113(7/9)
c) 115(4/9)
d) 140(5/9)
e) 142(2/9)

8).1685,1712,1837, 2280, 2909, ?


a) 4119
b) 3775
c) 4094
d) 4240
e) 3656

9).3, 0, -4, -17, -74, ?


a) -309

www.bankingpdf.com
b) -302
c) -363
d) -290
e) -377

10). 2025, 2401, 2809, ?, 3721, 4225


a) 3225
b) 3281
c) 3249
d) 3301
e) 3369

Answers:
1). d) 2). a) 3). e) 4). b) 5). c) 6). a) 7). b) 8). d) 9). e) 10). c)

Solution:
1).From I,(z-7)(z+8) = 0
=>z = 7, -8
3 × II – 2 × III => 19y =171
=>y = 9
Putting in II, we get x=9
Hence x=y>z
Answer: d)

2).I – II => 4y + 4z = 60
=>y+z = 15(IV)
I – 3III => 5z -5y = 0
=>y=z

www.bankingpdf.com
Putting in IV, we get y = z = 7.5
Putting in II, x = 6.5
Hence x<y =z
Answer: a)

3).From I, (x+1)(x+2)=0
=>x=-1,-2
From II, (y+3)(y+5)=0
=>Y = -3,-5
From III,(z+6)(z+7)=0
=>z = -6,-7
Hence x>y>z
Answer: e)

4).From I, x = -2.5
Putting in III,-7.5 + 7z = 7.9
=>7z =15.4
=>z=2.2
From II, (y+2.5)(y-1.8)=0
=>y=-2.5,1.8
Hence x≤y<z
Answer: b)

5).II + 2xIII – 1 => 2x + 8z =42


Or x+4z = 21(IV)
3xIV – II =>7z=28
=>z=4
Putting in II,3x + 20 =35
=>x=5

www.bankingpdf.com
Putting these values in equation I,
We get, 5 + 2y + 4 =23
=>y=7
Hence x<y>z
Answer: c)

6).The next number is obtained by subtracting sum of digits from the number.
Hence the missing number is 360-(3+6+0)=351
Answer: a)

7).The pattern is 1/9,4/9,16/9,64/9,256/9,1024/9.


1024/9 = 113 7/9
Answer: b)

8).1685+33 =1712,1712+53 =1837,1837+73=2280, 2280+93=2909, 2909+113=4240


Answer: d)

9).The pattern is 3×1-3 = 0 ,0×2-4 = -4 ,-4×3-5 = -17,-17×4-6 =-74,-74×5-7=-377


Answer: e)

10).The pattern is 452,492,532,572, 612, 652


Hence missing term =572 = 3249
Answer: c)

www.bankingpdf.com
800-810 Questions :

Directions (Q. Nos.1-5) : study the following information to answer the given
questions.
Percentage of students in various courses (A, B, C, D, E, F) and Percentage of girls
out of these.
Total number of Students: 1200 (800 girls + 400 boys)
PERCENTAGE IN VARIOUS COURSES

Total Number of Girls: 800


PERCENTAGE OF GIRLS IN COURSES

www.bankingpdf.com
1). For which course is the number of boys the minimum ?
a) E
b) F
c) C
d) A
e) None of these

2). How many girls are there in course C?


a) 44
b) 16
c) 40
d) 160
e) None of these

3). For course D what is the respective ratio of boys and girls ?
a) 3 : 4
b) 4 : 5
c) 3 : 5
d) 5 : 6
e) None of these

4). For which pair of courses is the number of boys the same ?
a) E and F
b) A and D
c) C and F
d) B and D

www.bankingpdf.com
e) None of these

5). For course E, the number of girls is how much percent more than the boys for
course E ?
a) 250%
b) 350%
c) 150%
d) 80%
e) None of these

Directions (Q. Nos.6-9) : Study the following pie-diagrams carefully to answer these
questions:

Classification of appeared candidates in a competitive test from different States and


qualified candidates from those States.
Total appeared candidates = 45,000.

www.bankingpdf.com
Total qualified candidates = 9000.

6). What is the ratio between the number of appeared candidates from States C and
E together and the appeared candidates from States A and F together ?
a) 17 : 33
b) 11 : 13
c) 13 : 27
d) 17 : 27
e) None of these

7). In which State, the percentage of qualified candidates to that of appeared


candidates is minimum?
a) C
b) F
c) D
d) E
e) G

www.bankingpdf.com
8).What is the percentage of qualified candidates to that of appeared candidates
from States B and C taken together? (rounded to two decimal places).
a) 23.11%
b) 24.21%
c) 21.24%
d) 23%
e) None of these

9). What is the ratio between number of candidates qualified from States B and D
together and the number of candidates appeared from State ‘C’ respectively ?
a) 8 : 37
b) 11 : 12
c) 37 : 40
d) 7 : 37
e) None of these

Answers:
1).d) 2).b) 3).a) 4).c) 5).a) 6).a) 7).e) 8).b) 9).c)

Solution:

Directions :(Q. Nos. 1-5)


Number of students in course A = 1200 × (20 / 100) = 240
Number of girls = 800 × (30 / 100) = 240, Number of boys = 0
Number of students in course B = 1200 × (15 / 100) = 180
Number of girls = (800 × 10) / 100 = 80, Number of boys = 180 – 80 = 100

www.bankingpdf.com
Number of students in course C = 1200 × (5 / 100) = 60
Number of girls = (800 × 2) / 100 = 16, Number of boys = 60 – 16 = 44
Number of students in course D = 1200 × (35 / 100) = 420
Number of girls = (800 × 30) / 100 = 240, Number of boys = 420 – 240 = 180
Number of students in course E = (1200 × 12) / 100 = 144
Number of girls = (800 × 14) / 100 = 112, Number of boys = 144 – 112 = 32
Number of students in course F = (13 × 1200) / 100 = 156
Number of girls = (800 × 14) / 100 = 112, Number of boys = 156 – 112 = 44

1). Number of boys in course A is minimum i.e. zero.


Answer: d)

2). Number of girls in course C = 16


Answer: b)

3). Required ratio = 180 : 240 = 3 : 4


Answer :a)

4). Number of boys in each of courses C and F = 44


Answer: c)

5). Required percentage = [(112 - 32) / 32] × 100 = (80 / 32) × 100 = 250%
Answer: a)

www.bankingpdf.com
6). No. of candidates who appeared from states C and E together = (8% + 9%) of
45000 = 17% of 45000
No. of candidates who appeared from states A and F together = (15% + 18%) of
45000 = 33% of 45000
Required ratio = 17% of 45000 : 33% of 45000 = 17 : 33
Answer: a)

7). Let us compare the ratios for states given in option.


States C F D E G

No. of (8 / 100) (18/100) × (17/100) × (9/100) ×45000 (22/100) ×


candidates × 45000 45000 = 45000 = 7650 = 4050 45000 = 9900
who appeared = 3600 8100

No. of qualified (7/100) × (11/100) × (21/100) ×9000 (14/100) ×9000 (13/100) ×9000
candidates 9000 = 9000 = 990 = 1890 = 1260 = 1170
630

Ratio (630 / (990/8100) 1890/7650 1260/4050 1170/9900


3600) =
(11/90)<(1/6) (189/765)>(1/6) (126/405)>(1/6) (117/990)<(1/6)
(7/40) =
1/6

Since we have to find minimum percentage increase, compare 11/90 and 117/990
11/90 = 121/990, 121/990 > 117/990
Hence, state G has minimum percentage increase.
Answer: e)

8). Qualified candidates of B and C to together = (16% + 7%) of 9000 = 23% of 9000

www.bankingpdf.com
Appeared candidates of B and C together = (11% + 8%) of 45000 = 19% of 45000
Required % = [(23 × 9000) / (19 × 45000)] × 100 = [(23 × 100) / (19 × 5)] = (23 × 20)
/ 19 = 24.21%
Answer: b)

9). Required ratio = (16% + 21%) of 9000 : 8% of 45000


(37 / 100) × 9000 = (8/100) × 45000 = 37 × 9000 : 8×45000 = 37 : 8 ×5 = 37 : 40
Answer: c)

810-820 Questions :

1). A mixture contains milk and water in the ratio of 3:2 litter of water is added to the
mixture, milk, milk and water in the mixture become equal find the quantities of milk and
water in the mixture .
a) 12, 8 litter
b) 4,3 litter
c) 12, 6 litres
d) 10,8 litres
e) None of these

2). A and B working alone can finish a job in 5 days and 7 days respectively. They work
at it alternately for a day. If A starts the work, find in how many days the job will be
finished?
a) 29/5
b) 11/5
c) 24/5
d) 21/5
e) None of these

www.bankingpdf.com
3). The incomes of A of B are in the ratio 3:2 and their expenditure are in the ratio 5:3 if
each saves rupees 2000, what is their income?
a) 32000
b) 20000
c) 1190
d) 8000
e) None of these

4). The ratio between two number is 3:4 ,if each number be increased by 9, the ratio
becomes 18:23 find the sum of the number
a) 135
b) 105
c) 155
d) 165
e) None of these

5). 6 men and 8 women can complete a work in 10 days. 26 men and 48 women can
finish the same work in 2 days. 15 men and 20 women can do the same work in - days.
a) 4 days
b) 6 days
c) 2 days
d) 8 days
e) 7 days

Directions (Q. 6-10): Study the following bar graph and pie-chart and answer the
questions that follow:

www.bankingpdf.com
Sector wise export in each month

6). The export of Diamond in March is approximately how many times the export of
Diamond in April?
a) 2.212 times
b) 1.212 times
c) 1.732 times

www.bankingpdf.com
d) 17 times
e) 2 times

7). What is the average export (in billion dollars) of Cotton industry over the period
March to August?
a) 14.6.
b) 17.8
c) 18.9
d) 12.6
e) None of these

8). The export of Rice and Cotton together in the month of August is approximately what
per cent of the export of the other three categories in the pie-chart in the same month?
a) 84%
b) 180%
c) 186%
d) 86%
e) 190%

9). If the export in September increases by 15% in comparison to previous year, then
what is the approximate amount of increase in Rice industry?
a) $37 billion
b) $49 billion
c) $48 billion
d) Data inadequate
e) None of these

10). The export of Sugar in July is what per cent more than Iron in April?

www.bankingpdf.com
a) 21%
b) 24%
c) 23%
d) 22%
e) None of these

Answers:
1).a) 2).b) 3).d) 4).b) 5).a) 6).b) 7).d) 8).c) 9).d) 10). b)

Solution:
1). Let quantities of milk and water in the mixture be 3x and 2x. Then if 4 litres of water
is added to the mixture the ratio of milk and water become 1:1.
It can be written as (3x): (2x + 4) = 1/1
Thus, 3x = 2x +4
x=4
Therefore, the milk in the mixture is 4x3 = 12 litres and quantity of water = 4x2 = 8 liters
Answer: a)

2). Work done by A in one day = 1/5 and work done by B in one day = 1/7
They are working alternately.
Therefore, Work done in first two day = (1/5 + 1/7) = 12/35
Work done in first four day = 24/35
Work done in first 5 days = 24/35 + 1/5 = 31/35
Remaining work = 4/35
Day it will take B to complete = 4/35/1/7 = 4/5 of the day.
Therefore Total days taken = 5 + 4/5 = 29/5 days
Answer: b)

3). Let the income be 3x and 2x. It is given that the saving of each is Rs. 2000.

www.bankingpdf.com
Then, their expenditures are 3x – 2000 and 2x – 2000
Again, (3x – 2000)/(2x – 2000) = 5/3
=> 3(3x – 2000) = 5(2x – 2000)
=> 9x – 6000 = 10x – 10000
=> 9x -10x = -10000+ 6000
=> -x = -4000
=> x = 4000
Therefore, their salaries are 3 x 4000 = 12000 and 2 x 4000 = 8000
Answer: d)

4). Let the two numbers be 3x and 4x.


When they are increased by 9 they become 3x + 9 and 4x + 9.
It is given that the ratio is 18:23
Thus, 3x + 9/4x + 9 = 18:23
23(3x + 9) = 18(4x + 9)
69x + 207 = 72x + 162
69x – 72x = 162- 207
-3x = -45
X = 15
Thus two numbers are 3x15 = 45 and 4 x 15 = 60
And the sum is 60+45 = 105
Answer: b)

5). Let work done by 1 man in 1 day = m and work done by 1 woman in 1 day = b
Work done by 6 men and 8 women in 1 day = 1/10
=> 6m + 8b = 1/10
=> 60m + 80b = 1 --- (1)
Work done by 26 men and 48 women in 1 day = 1/2
=> 26m + 48b = ½

www.bankingpdf.com
=> 52m + 96b = 1--- (2)
Solving equation 1 and equation 2, We get m = 1/100 and b = 1/200
Work done by 15 men and 20 women in 1 day
= 15/100 + 20/200 =1/4
=> Time taken by 15 men and 20 women in doing the work = 4 days
Answer: a)

6). Export of Diamond in March


= 8% of 40 = 3.2 billion
Now, Export of Diamond in April
= 8% of 33 = 2.64 billion
Number of times = 3.2/2.64 = 1.212 times
Answer: b)

7). Total export of Cotton in the given period = 35% of (40 + 33 + 34 + 32 + 38 + 39)
= 35% of 216 = 75.6 billion
Average export of Cotton
= 75.6/6 = 12.6 billions
Answer: d)

8). Export of Rice and Cotton in August


= 65% of 39 = 25.35 billion
Total export in the other three sectors
= 35% of 39 = 13.65 billion
Required per cent = 25.35/13.65 × 100
= 185.714 = 186%
Answer: c)

www.bankingpdf.com
9). There is no data available for previous year, so we can’t find the solution.
Answer: d)

10). Export of Sugar in July = 14% of 38 = 5.32 billion


Now, export of Iron in April = 13% of 33 = 4.29 billion
% increase = (5.32 - 4.29)/4.29 × 100
= 1.03 × 100 / 4.29 = 24.009 = 24%.
Answer: b)

820-830 Questions :

Directions (Q. 1-5): What approximate value will come in place of question mark (?) in
the following questions?(you are not expected to calculate the exact value.)
1). 23.98 × 62.49 × 12.03 = ? × 29.97
a) 500
b) 600
c) 900
d) 450
e) 750

2). (?)2 + (9.99)3 = (45.02)2 – (4.01)3


a) 34
b) 19
c) 26
d) 31
e) 12

3). √730 + √840 - √960 + √1090 - √290 = ?

www.bankingpdf.com
a) 41
b) 33
c) 20
d) 28
e) 55

4). [(3/8) + (8/3)] – [(4/9) + (9/4)] = ?


a) 0.5
b) 0.65
c) 0.9
d) 1.25
e) 0.35

5). 39.98 × 85.01 – 79.9 × 34.9 + 175.03 × 12.08 = ?


a) 4800
b) 4300
c) 2700
d) 2100
e) 3400

Directions (Q. 6-10): what value will come in place of question mark(?) in the following
questions?
6). 1824 ÷ 32 + 5040 ÷ ? – 5600 ÷ 35 = 37
a) 36
b) 48
c) 60
d) 72
e) 84

www.bankingpdf.com
7). 3.25 × 16 + 7.25 × 8 = ? + 75
a) 83
b) 75
c) 61
d) 53
e) 35

8). 242 + 312 + 372 + 432 = ?


a) 4575
b) 4755
c) 5145
d) 5065
e) 5315

9). 15% of 1460 + 25% of ? = 81% of 500


a) 846
b) 928
c) 656
d) 712
e) 744

10). 72× 25 + 45× 96 = 163 + ?


a) 1912
b) 2024
c) 1844
d) 1908
e) 1960

www.bankingpdf.com
Answers:
1). b) 2). d) 3). a) 4). e) 5). c) 6). a) 7). e) 8). b) 9). e) 10). b)

Solution:
1). 24× 62.5× 12 = ? × 30
24× 750 = ? × 30
? = (24× 750) / 30 = 600
Answer: b)

2). ?2 + 1000 = 2025 – 64


?2 = 961  ? = 31
Answer: d)

3). ? = 27 + 29 – 31 + 33 – 17 = 41
Answer: a)

4). ? = [(64+9) / 24] – [(81+16)/36]


= (73/24) – (97/36) = (219 - 194)/72
= 25/72 = 0.347 = 0.35
Answer: e)

5). ? = 40 × 85 – 80 × 35 + 175 × 12 = 3400 – 2800 + 2100 = 2700


Answer: c)

6). 57 + 5040 ÷ ? – 160 = 37


5040 ÷ ? = 37 – 57 + 160 = 140

www.bankingpdf.com
= 5040 ÷ 140 = 36
Answer: a)

7). ? + 75 = 52 + 58 = 110
? = 110 – 75 = 35
Answer: e)

8). ? = 576 + 961 + 1369 + 1849 = 4755


Answer: b)

9). 219 + 25% of ? = 405


25% of ? = 405 – 219 = 186
? = 186×4 = 744
Answer: e)

10). 1800 + 4320 = 4096 + ?


? = 1800 + 4320 – 4096 = 2024
Answer: b)

www.bankingpdf.com
830-840 Questions :

Directions (Q. 1-5): In each of the following number series, a wrong number is given.
Find out that number.

1). 0 1 9 36 99 225 441


a) 9
b) 36
c) 99
d) 225
e) 441

2). 5 10 17 27 37 50 65
a) 10
b) 17
c) 27
d) 37
e) 50

3). 3 7.5 15 37.5 75 167.5 375


a) 167.5
b) 75
c) 37.5
d) 15
e) 7.5

4). 108 54 36 18 9 6 4
a) 54
b) 36
c) 18
d) 9

www.bankingpdf.com
e) 6

5). 2 3 5 8 14 23 41 69
a) 5
b) 8
c) 14
d) 41
e) 69

Directions (Q. 6-10): The following line graph shows the number of newspaper readers
in Tamil and English language in six years. The table gives the information about the
ratio of Male to Female readers among them. The values will be in 1000.

www.bankingpdf.com
Year Tamil English

M:F M:F

2011 2:1 8:3

2012 5:3 3:1

2013 3:2 7:2

2014 2:1 9:5

2015 1:1 3:2

2016 7:6 2:1

6). What is the average number of Females who read Tamil newspaper taking all the
years together?
a) 3740
b) 3850
c) 3960
d) 4080
e) 4120

7). The number of Females who read English newspaper in 2016 is what percentage
more than the number of Males who read English newspaper in the year 2011?
a) 7.5%
b) 10%
c) 12.5%
d) 15%
e) None of these

www.bankingpdf.com
8). What is the ratio of the number of Males who read Tamil newspaper in the year 2014
to the number of Females who read English newspaper in the year 2011?
a) 12 : 5
b) 15 : 4
c) 16 : 4
d) 17 : 3
e) 19 : 9

9). The number of Females who read English newspaper in the year 2013 is what
percentage of the number of Females who read Tamil newspaper in the same year?
a) 35%
b) 42%
c) 45%
d) 50%
e) 54%

10). What is the total number of Females who read Tamil newspaper in the year 2014?
a) 2700
b) 3200
c) 3400
d) 3600
e) 4000

Answers:
1)c 2)c 3)a 4)d 5)e 6)b 7)e 8)d 9)a 10)c

Solution:
1). The differences are 1 - 0 = 1 = 13 ; 9 - 1 = 8= 23 ; 36 - 9 = 27 = 33 ; 99 – 36 = 63 ≠
43 , but

www.bankingpdf.com
100 - 36 = 64 = 43 ;
225 – 100 = 125 = 53 ;
441 – 225 = 216 = 63
Answer: c)

2). The series is +5, +7, +9, +11, ....


Answer: c)

3). The series is ×2.5, ×2 alternately


Answer: a)

4). The series is ÷2, ÷1.5 alternately.


Answer: d)

5). The series is an alternate series, having


S 1 = 2 5 14 41; ×3 – 1 in each term
S 2 = 3 8 23 69; ×3 – 1 in each term
Answer: e)

6). The required average


= [6000 × 1/3 +6400 × 3/8 +10000 × +10200 × 1/3 +10600 × 1/2 +13000 × 6/13] ÷6
= [2000+2400+4000+3400+5300+6000] ÷ 6
= 23100/6 = 3850
Answer: b)

7). Male - 2011 = 4400/11 × 8 = 3200


Female - 2016 = 10500/3 × 1 = 3500
Required per cent = 3500 – 3200/3200 × 100 = 9.375%
Answer: e)

8). Male-Tamil - 2014 =10200/3 × 2 = 6800


Female-English - 2011 = 4400/11 × 3 = 1200

www.bankingpdf.com
Ratio = 6800/1200 = 17/3 = 17: 3
Answer: d)

9). Tamil - 2013 = 10000/5 × 2 = 4000


English - 2013 = 6300/9 × 2 = 1400
Required per cent = 1400/4000 × 100 = 35%
Answer: a)

10). Required number of females


=10200/3 × 1 = 3400
Answer: c)

840-850 Questions :

Directions (Q. 1-5): Evaluate

1). 7% of 77 + 77% of 777 + 777% of 7777


a) 58090.54
b) 60457.25
c) 68709.16
d) 71000.01
e) None of these

2). 19 + (12 ÷ 4 × (240 × 1 ÷ 180)) × 6 ÷ (101 + 98 ÷ 7 - 113) – (7 ÷ 2 + 14 – 3.5 – 9 ×


8.5) × 4
a) 201
b) 241
c) -241

www.bankingpdf.com
d) -281
e) 281

3). {(0.013)3 + 0.000000343} / {(0.013)2 – 0.000091 + 0.000049} =


a) 0.020
b) 0.002
c) 0.023
d) 0.021
e) None of these

4). 552 + 392 – 452 – 112


a) 1600
b) 1900
c) 2100
d) 2500
e) None of these

5). If [(156)2 ÷ 8 × 36] ÷ y = 117 × 24, what is the value of y?


a) 24
b) 39
c) 44
d) 32
e) 47

Directions (Q. 6-10): In the following questions two equations numbered I and II are
given. You have to solve the equations and select appropriate option.
a) x ≥ y
b) x < y

www.bankingpdf.com
c) x ≤ y
d) x > y
e) x = y or the relationship cannot be established

6). I. √(2025)x + √(4900) = 0


II. (81)1/4y + 3√(343) = 0

7). I. 15x2 – 8x + 1 = 0
II. y2 + 9.68 + 5.64 = 16.95 – 1.59

8). I. (113 - 35) / 6 = x3


II. 4y3 = (500 / 4) + 5y3

9). I. 3x2 + 8x + 4 = 0
II. 4y2 – 19y + 12 = 0

10). I. (4/√x) + (7/√x) = √x


II. y2 – [(11)5/2 / √y] = 0

Answers:

1)e 2)e 3)a 4)e 5)b 6)d 7)a 8)d 9)b 10)e

Solution:
1). 7% of 77 + 77% of 777 + 777% of 7777
= 0.07 × 77 + 0.77 × 777 + 7.77 × 7777
= 61030.97

www.bankingpdf.com
Answer: e)

2). 240×1 ÷ 180 = 4/3


12÷ 4 × (240 × 1 ÷ 180) = 3 × (4/3) = 4
101 + 98 ÷ 7 – 113 = 101 + 14 – 113 = 2
19 + (12 ÷ 4 × (240 × 1 ÷ 180)) × 6 ÷ (101 + 98 ÷ 7 - 113) = 19 + 4 × 6 ÷ 2
= 19 + 4 × 3 = 31
7÷2 + 14 – 3.5 – 9 × 8.5 = 3.5 + 14 – 3.5 – 76.5 = -62.5
(7÷2 + 14 – 3.5 – 9 × 8.5) × 4 = - 62.5 × 4 = -250
19 + (12 ÷ 4 × (240 × 1 ÷ 180)) × 6 ÷ (101 + 98 ÷ 7 - 113) – (7÷2 + 14 – 3.5 – 9 × 8.5) ×
4
= 31 – (-250) = 281
Answer: e)

3). (0.013)3 + 0.000000343 = 0.00000254


(0.013)3 – 0.000091 + 0.000049 = 0.000127
Therefore,
{(0.013)3 + 0.000000343} / {(0.013)2 – 0.000091 + 0.000049} = 0.00000254 / 0.000127
= 2.54 /127 = 0.02
Answer: a)

4). 552 – 452 = (55 + 45) × (55 - 45) = 1000


392 – 112 = (39 + 11) × (39 - 11) = 1400
Sum = 1000 + 1400 = 2400
Answer: e)

5). (156 × 156 × 36) / 8 = 117 × y × 24


y = (156 × 156 × 36) / (117 × 8 × 24)

www.bankingpdf.com
now, 156 = 13 × 21, 36 = 12×3, 117 = 13×9
y = (13 × 12 × 13 × 12 × 12 × 3) / (13 × 9 × 8 × 12 × 2)
= (13 × 12 × 12 × 3) / (9 × 8 × 2)  12 × 12 = 144 = 9 × 8 × 2
So, y = 13 × 3 = 39
Answer: b)

6). I. 45x + 70 = 0 or x = (-70/45) = (-14/9)


II. 3y + 7 = 0 or y = -(7/3)
Answer: d)

7). I. 15x2 – 5x – 3x + 1 = 5x(3x - 1) –1 (3x - 1) = (5x - 1) (3x -1)


So, x = 0.2, 1/3
II. y2 = 0.04
y = ± 0.2
Hence, x≥y
Answer: a)

8). I. 6x3 = 113 – 35 = 1296


x3 = 216 or x = 6
II. y3 = -125
y = -5
Hence, x>y
Answer: d)

9). I. 3x2 + 8x + 4 = 0
3x2 + 6x + 2x +4 = 0
3x(x + 2) + 2 (x + 2) = 0
(3x + 2) (x + 2)= 0

www.bankingpdf.com
x = -2 or -(2/3)
II. 4y2 – 19y + 12 = 0
4y2 – 16y – 3y +12 = 0
4y(y -4) – 3(y - 4) = 0
(y-4) (4y - 3) = 0
y = 4 or 3/4
Hence, x<y
Answer: b)

10). I. (4+7) / √x = √x or x = 11
II. (y)2 + ½ = (11)5/2
y = 11

Answer: e)

850-860 Questions :

Directions (1-5): What will come in place of the question mark (?) in the following
number series?

1). 12 28 75 316 1555 ?


a) 9366
b) 9294
c) 7811
d) 9336
e) None of these

2). 3 20 87 392 ?
a) 1612
b) 1963

www.bankingpdf.com
c) 847
d) 2025
e) None of these

3). 55 60 67 78 91 108 ?
a) 125
b) 121
c) 127
d) 89
e) None of these

4). 112 111 119 92 156 31 ?


a) 375
b) 287
c) 387
d) 247
e) None of these

5). 1 15 16 31 47 78 125 ?
a) 172
b) 203
c) 139
d) 167
e) None of these

Directions (Q. 6-10): Study the following table and pie-chart carefully and answer the
questions given below.

www.bankingpdf.com
The pie-chart shows the percentage of employees in various types of banks in 2012.

6). What is the percentage increase in the total number of Banks during 2009-12?
a) 125.5%
b) 123.8%

www.bankingpdf.com
c) 122.3%
d) 127.7%
e) 131.5%

7). What is ratio of the total number of Reserve Bank of India, Regional Rural Banks
and Public Sector Banks in the year 2009 to the total number of Reserve Bank of India
in the year 2012?
a) 11:7
b) 12:9
c) 12:7
d) 11:9
e) 13:5

8). In which of the following years is the increase in the number of banks the minimum
in comparison to the previous year?
a) 2009
b) 2010
c) 2011
d) 2012
e) None of these

9). The average of the number of employees working in Reserve Bank of India,
Regional Rural Banks and Public Sector Banks in the year 2012 is what percentage
more or less than the number of employees working in Private Banks in the same year?
a) 59.25% less
b) 61.27% more
c) 57.48% less
d) 63.37% more
e) 54.21% less

10). What is the percentage increase in the number of Reserve Bank of India and
Regional Rural Banks from 2011 to 2012?

www.bankingpdf.com
a) 57.63%
b) 55.87%
c) 54.54%
d) 53.32%
e) 52.72%

Answers:
1)a 2)d 3)c 4)d 5)b 6)b 7)c 8)b 9)a 10)c

Solution:
1). 12×2 = 24 + 4= 28
28×3 = 84 – 9 = 75
75×4 = 300 + 16 = 316
316×5 = 1580 – 25 = 1555
1555×6 = 9330 + 36 = 9366
Answer: a)

2). 3×2 + 14 (7×2) = 20


20×3 + 27(9×3) = 87
87×4 + 44 (11×4) = 392
392×5 + 65 (13×5) = 2025
Answer: d)

3). 55 + 5 = 60
60 + 7 = 67
67 + 11 = 78
78 + 13 = 91
91 + 17 = 108+108 + 19 = 127

www.bankingpdf.com
Answer: c)

4). 112 – 13 = 111


111 + 23 (8) = 119
119 – 33 (27) = 92
92 + 43 (64) = 156
156 - 53 (125) = 31
31 + 63 (216) = 247
Answer: d)

5). 15 + 1 = 16
15 + 16 = 31
16 + 31 = 47
31 + 47 = 78
47 + 78 = 125
78 + 125 = 203
Answer: b)

6). Total number of Banks in 2009 = 50 + 100 + 150 + 225 = 525


Total number of Banks in 2012 = 175 + 250 + 325 + 425 = 1175
Increase = 1175 - 525 = 650
Percentage increase = 650/525 × 100= 123.8%
Answer: b)

7). Total number of (Reserve Bank of India + Regional Rural Banks + Public Sector
Banks) in 2009
= 50 + 100 + 150 = 300
Number of Reserve Bank of India in 2012 = 175

www.bankingpdf.com
Required ratio = 300: 175 = 12: 7
Answer: c)

8). Total number of banks in 2009 = 525


Total number of banks in 2010 = 75 + 150 + 175 + 250 = 650
Percentage increase = increase/525 × 100 = 125/525 × 100 = 23.8%
Total number of banks in 2011 = 125 + 200 + 250 + 275 = 825
Percentage increase = 825 – 650/650 × 100
= 175/650 × 100 = 26.92%
Total number of banks in 2012 = 1175
Percentage increase = 1175 – 825/825 × 100
1175 – 825/825 × 100 = 350/825 × 100 = 42.42%
Answer: b)

9). Total number of employees working in (Reserve Bank of India + Regional Rural
Banks + Public Sector Banks) in 2012
200000 (10/100 + 15/100 + 30/100) = 55 × 2000 = 110000
Average of the number of employees working in (Reserve Bank of India + Regional
Rural Banks + Public Sector Banks)
= 110000/3 = 36666.7
Employees working in Private Banks in 2012 = 200000 × 45/100 = 90000
Required% = 90000 - 36666.7/900000 × 100 = 59.25%
Answer: a)

10). Number of Reserve Bank of India and Regional Rural Banks in 2010
= 125 + 150 = 275
Number of Reserve Bank of India and Regional Rural Banks in 2012 = 175 + 250 = 425
Percentage increase = 425 – 275/275 × 100% = 150/275 × 100 = 54.54%
Answer: c)

www.bankingpdf.com
860-870 Questions :

Directions (Q. 1-5): Students appearing in 5 exams over 5 years.


Each cell contains number of students that appeared, and pass percentage in brackets.
For eg, students who appeared in BPharm was 700 in 2009 and 40% of them = 700 × .4
= 280 passed the exam.

1). Of the total number of students who passed BPharm across all the years, how many
were from 2011?
a) 11%
b) 13%
c) 16%
d) 18%
e) 21%

2). How many students passed across all the exams in 2012?
a) 876
b) 902
c) 922
d) 998
e) 1045

www.bankingpdf.com
3). The percentage of students who passed in MPharm exam over the 5 year is:
a) 56%
b) 66%
c) 68%
d) 64%
e) 71%

4). The total number of students passing across all the exams was the highest in:
a) 2009
b) 2010
c) 2011
d) 2012
e) 2013

5). The number of students who appeared in BPharm increased by 20% in 2014
compared to 2013, and the pass percentage was 45%. How many students failed in
BPharm in 2014?
a) 363
b) 297
c) 333
d) 303
e) 307

Directions (Q. 6-10): Sales of 5 different Halwas at a sweet shop for 4 weeks

www.bankingpdf.com
6). What is the total sale from Beet Root Halwa over the 4 weeks?
a) 10500
b) 12000
c) 12500
d) 9600

www.bankingpdf.com
e) 10000

7). What percentage of sales in the 4 weeks is from Diet Pumpkin Halwa?
a) 20%
b) 24%
c) 28%
d) 30%
e) 26%

8). The difference in sales of Corn Halwa and Water Melon Halwa over the 4 weeks
was?
a) 650
b) 720
c) 950
d) 1000
e) 400

9). If the sales in 5th week (compared to week 4) for went up 30% for Beet Root Halwa,
20% for Diet Pumpkin Halwa, and dropped by 20% for other 3, what is the total sale of
all 5 Halwas together in week 5?
a) 12000
b) 14200
c) 14400
d) 16000
e) 16800

10). What percentage of sales in weeks 2 to 4 was from Papaya Halwa?


a) 21.1%

www.bankingpdf.com
b) 23.5%
c) 18.6%
d) 24.5%
e) 26.8%

Solutions:
Directions (Q. 1-5):

1). Total BPharm pass = 280 + 225 + 150 + 350 + 165 = 1170
Percentage = 150 × 100/1170 = 12.82% ≈ 13%.
Answer: b)

2). Total pass in 2012 = 350 + 175 + 260 + 72 + 65 = 922


Answer: c)

3). MPharm total = 250 + 300 + 300 + 350 + 300 +=1500


MPharm pass = 125 + 210 + 240 + 175 + 210 = 960
Percentage = 960 × 100/1500 = 64%

www.bankingpdf.com
Answer: d)

4).
2009 2010 2011 2012 2013

846 760 869 922 943

Answer: e)

5). Total = 550 × 1.2 = 660


Failed = (1-0.45) × 660 = 363
Answer: a)

Directions (Q. 6-10):

www.bankingpdf.com
6). Beet Root Halwa totel = 10500
Answer: a)

7). Diet Pumpkin Halwa percentage = 12000 × 100/50000 = 24%


Answer: b)

8). Difference = 8000 – 7000 = 1000


Answer: d)

9). Beet Root Halwa = 1.3 × 4000 = 5200, Diet Pumpkin Halwa = 1.2 × 3000 = 3600
Other 3 = 0.8 × (2000 + 3000 + 4000) = 7200
Total = 5200 + 3600 + 7200 = 16000
Answer: d)

10). Week 2 to 4 sales = 42500


Papaya weeks 2 to 4 = 10000
Percentage = 10000 × 100/42500 ≈ 23.5%

Answer: b)

www.bankingpdf.com
870-880 Questions :

Directions (Q. 1-5) : In each of these questions, two equations (I) and (II) are given.
You have to solve both the equations and give answer
a) if x > y
b) (b if x ≥ y
c) if x < y
d) if x ≤ y
e) if x = y or no relationship can be established between ‘x’ and ‘y’.

1).
I. 4x2 + 3x – l = 0
II. 6y2 – 5y + l = 0

2).
I. x =3√357911
II. y =2√ 5041

3).
I. 5x + 7y = -43
II. 9x – 17y = 41

4).
I. 3x2 + 15x + 18 = 0
II. 2y2 + 15y + 27 = 0

5).

www.bankingpdf.com
I. x2 + 11x + 30 = 0
II. y2 + 9y + 20 = 0

Directions (Q. 6-10): Study the following graph carefully and answer the questions
that follow:
The line graph shows the production of milk in various states in different years.

The pie-chart shows the percentage of total production used to make milk product.

www.bankingpdf.com
6). In which state is the production of milk maximum over six years?
a) HP
b) JK
c) Kerala
d) Rajasthan
e) Both Rajasthan and HP

7). The milk used for milk products in 2003 is what per cent of the milk used for milk
products in 2005?
a) 210%
b) 102.27%
c) 110.14%
d) 125.98%
e) 97.05%

8). Total production of milk in 2006 is what per cent more than that in 2001?
a) 64.56%
b) 72.84%
c) 89.29%
d) 56.15%

www.bankingpdf.com
e) 69.23%

9). What is the ratio of milk used for milk products in 2004 to 2001?
a) 3 : 7
b) 14 : 15
c) 2 : 5
d) 12 : 13
e) 7 : 11

10). What is the difference between the volume of milk used for milk products in
2006 and that in
2002?
a) 24 lakh litres
b) 28 lakh litres
c) 32 lakh litres
d) 35 lakh litres
e) 34 lakh litres

Answers:
1)c 2)b 3)e 4)b 5)d 6)b 7)b 8)e 9)d 10)a

Solution:
1). I. 4x2 + 4x - x - l = 0
or 4x(x+ 1)- l(x + 1) = 0
or (4x - 1) (x + 1) = 0
x = -1, ¼
II. 6y2 - 3y - 2y + 1 = 0
or 3y(2y - 1) - l(2y - 1) = 0

www.bankingpdf.com
or (3y - 1) (2y - 1) = 0
y = 1/2 ,1/3
x<y
Answer: c)

2). I. x = 3√357911 = x = 71
II. y = 2√5041 = y = 71
x=y
Answer: b)

3). Equation (l) × 9 - Equation (II) × 5


45x + 63y = -387
45x - 85y = 205
- + -
148y = -592
y = -4 and x = -3
x>y
Answer: e)

4).
I. 3x2 + 9x + 6x + 18 = 0
or 3x(x + 3) + 6(x + 3) = 0
or (x + 3)(3x + 6) = 0
x = -3, -2
II. 2y2 + 6y + 9y+ 27 = 0
or 2y(y + 3) + 9(y + 3) = 0
or (2y + 9)(y + 3) = 0

www.bankingpdf.com
y = -3, -9/2
Answer: b)

5).
I. x2 + 11x + 30 = 0
or x(x + 5) + 6(x + 5) = 0
or (x + 5) (x + 6) = 0
x = -5, -6
II. y2 + 4y + 5y + 20 = 0
or y(y + 4) + 5(y + 4) = 0
or (y + 4) (y + 5) = 0
y = -4,-5
x≤y
Answer: d)

6). Total production of milk in JK


= (60 + 60 + 70 + 80 + 60 + 70) lakh litres
= 400 lakh litres = 4 crore litres
Total production of milk in Kerala
= (40 + 70 + 50 + 30 + 70 + 60) lakh litres
= 320 lakh litres = 3.2 crore litres
Total production of milk in HP
= (10 + 50 + 10 + 20 + 40 + 50) lakh litres
= 1.8 crore litres
Total production of milk in Rajasthan
= (20 + 30 + 20 + 50 + 50 + 40) lakh litres
= 2.1 crore litres
In JK the production of milk is the maximum during the six years.

www.bankingpdf.com
Answer: b)

7). Total production of milk in 2003


= (10 + 20 + 50 + 70) lakh litres
= 1.5 crore litres
The milk used in milk products = 1.5 × 18/100
= 27 lakh litres
Total production of milk in 2005
= (40 + 50 + 60 + 70) = 2.2 crore litres
The milk used in milk products = 2.2 × 12/100
= 26.4 lakh litres
Required %= 27/26.4 × 100 = 102.27%
Answer: b)

8). Total production of milk in 2006


= (40 + 50 + 60 + 70) = 2.2 crore litres
Total production of milk in 2001
= (10 + 20 + 40 + 60) = 1.3 crore litres
Required % = (2.2 - 1.c)/1.3 × 100 = 69.23% more than the production of 2001.
Answer: e)

9). The milk used for milk products in 2004


= (20 + 30 + 50 + 80) × 8/100 = 14.4 lakh litres
The milk used for milk products in 2001 = 1.3 × 12/100 = 15.6 lakh litres
Required ratio = 14.4 : 15.6 = 12 : 13
Answer: d)

www.bankingpdf.com
10). The milk used for milk products in 2006
= 2.2 × 30/100
= 66 lakh litres
The milk used for milk products in 2002 = (30 + 50 + 60 + 70) × 20/100 = 210 ×
20/100 = 42 lakh litres
Required difference = (66 - 42) = 24 lakh litres
Answer: a)

890-900 Questions :

1). The width of a rectangular park is 10/21 of its length. If the area of the park is 3360
sq m, then what is the difference between the length and the width of the park?
a) 40m
b) 44m
c) 48m
d) 52m
e) 56m

2). My brother is 3 years elder to me. My father was 28 years of age when my sister
was born while my mother was 26 years of age when I was born. If my sister was 4
years of age when my brother was born, then what was the age my father when my
brother was born?
a) 35 years
b) 34 years
c) 33 years
d) 32 years
e) 30 years

www.bankingpdf.com
3). The profit earned after selling a wrist watch for `5080 is the same as the loss
incurred after selling the same wrist watch for `4650. What is the cost price of the wrist
watch?
a) `4685
b) `4875
c) `4695
d) `4785
e) `4865

4). A and B started a business jointly. A’s investment was thrice the investment of B and
the period of his investment was two times the period of investment of B. If B received
Rs.4000 as profit, then their total profit is :
a) Rs.16,000
b) Rs.20,000
c) Rs.24,000
d) Rs.28,000
e) None of these

5). Three pipes A, B and C can fill a tank in 6 hours. After working at it together for 2
hours, C is closed and A and B can fill the remaining part in 7 hours. The number of
hours taken by C alone to fill the tank is:
a) 10
b) 14
c) 12
d) 13
e) 15

www.bankingpdf.com
Directions (Q.6-10): Study the following graphs to answer the questions given below:
Number of applicants (in lakh) for three different banking exams, viz SBI, IBPS and
Insurance Exams over the years

The following graph shows the number of female applicants of IBPS and Insurance
Exam in thousands per one lakh.

www.bankingpdf.com
6). What is the percentage of the number of average applicants, for SBI Bank Exam
with respect to that of average applicants for IBPS over the given period 2006-2011?
a) 50%
b) b) 66 2/3%
c) 75%
d) 45%
e) None of the above

7). In which of the following years the percentage increase/decrease in the number of
applicants for Insurance Exam is maximum with respect to the previous year?
a) 2007
b) 2008
c) 2009
d) 2010
e) None of the above

www.bankingpdf.com
8). The number of female applicants, for Insurance Exam is what percentage of the
number of female applicants for IBPS in the year 2011?
a) 48.14%
b) 35.14%
c) 60.41%
d) 63.14%
e) Cannot be Determined

9). What is the approximate percentage increase or decrease in the number of male
applicants for Insurance Exam in the year 2010 with respect to the previous year?
a) 8%
b) 7%
c) 9%
d) 6%
e) Cannot be Determined

10). What is the ratio of the number of male applicants for SBI to that for IBPS in the
year 2009?
a) 51 : 99
b) 32 : 63
c) 43 : 55
d) 44 : 63
e) Cannot be determined

Answers:

1)b 2)d 3)e 4)d 5)b 6)b 7)b 8)a 9)d 10)e

www.bankingpdf.com
Solution:
1). Let the length of the park be x metres.
Width = x × 10 / 21 = 10x / 21
Area = x × 10x / 21 = 3360
or, x^2 = 3360 × 21 / 10 = 7056
or, x = 84
Hence, length = x = 84m and width = 84 × 10 / 21 = 40 metres
Difference = 84 – 40 = 44 metres
Answer: b)

2). Let my age = x


Then
My brother's age = x + 3
My mother's age = x + 26
My sister's age = (x + 3) + 4 = x + 7
My Father's age = (x + 7) + 28 = x + 35
=> age my father when my brother was born = x + 35 – (x + 3) = 32.
Answer: d)

3). Cost price of the watch


= 5080 + 4650 / 2
= 9730 / 2
= `4865
Answer: e)

4). Suppose B invested Rs. x for y months. Then,


A invested Rs.3x for 2y months.
So, A : B = (3x × 2y) : (x × y) = 6xy : xy = 6 : 1
So, B’s profit : total profit = 1 : 7

www.bankingpdf.com
Let the total profit be Rs. x. Then, 1 / 7 = 40000 / x
Or x = 28000.
Answer: d)

5). (A+B+C) - (A+B)


(A+B+C) =1/6 and (A+B+C) in 2hrs=2/6 and remaining part 1-2/6=2/3.
So (A+B) in 7 hrs is 2/ (3*7) =2/21.
1/6-2/21=1/14
so answer is 14.
Answer: b)

6). Average number of applicants for SBI = 1.5 + 2.5 + 3 + 2.5 + 3.5 + 5/6
= 18 /6 = 3 lakh
Average number of applicants for IBPS = 2.5 + 3.5 + 4.5 + 4 + 5.5 + 7/6 = 27/6 = 4.5
lakh
Required % = 3 / 4.5 × 100 = 66 2/3 %
Answer: b)

7). In the year 2008, % increase is the Maximum.


Answer: b)

8). Number of female applicants for Insurance Exam in 2011


= 4 × 22.75 × 1000 = 91000
Number of female applicants for IBPS in 2011 = 27000 × 7= 189000
Required % = 91000 / 189000 × 100 = 48.14
Answer: a)

www.bankingpdf.com
9). Number of male applicants for Insurance Exam in 2010 = 5 × 78000 = 390000
Number of male applicants for Insurance Exam in 2009 = 5.5 × 75000 = 412500
% decrease = 412500 – 390000 / 412500 = 22500/412500 = 5.45%
Answer: d)

10). Number of male applicants for SBI is not known; hence it cannot be determined
Answer: e)

900-910 Questions :

Directions (Q. 1-5): The production of Paddy, Barley and Oats over 5 years is as shown
below. All values are in 1000’s of Kgs.

www.bankingpdf.com
1). What was the combined production (in 1000’s of kg) of Paddy and Oats over the 5
year period?
a) 1190
b) 1170
c) 1210
d) 1230
e) 1250

2). Which year (compared to the previous year) showed the highest growth in
production of Barley and Oats taken together?
a) 2010
b) 2012
c) 2013
d) 2011
e) 2010 and 2011

3). If the cost of production of 1 kg of Barley in 2011 was 4 Rs per kg, 1 kg of Oats was
3 Rs per Kg throughout the 5 year period, what is the percentage increase in cost of
production of Barley and Oats in 2011 compared to 2009?
a) 75%
b) 88.67%
c) 84.25%
d) 81%
e) 82.75%

4). What percentage of total Barley production was in 2013 over the 5 year period?
a) 22%
b) 23%
c) 24%

www.bankingpdf.com
d) 25%
e) 26%

5). If the production of Barley in 2014 (compared to 2013) went by 25%, Oats 15% and
Paddy by 30%, What was the total production of all 3 items in 2014 in 1000’s of Kg’s?
a) 600
b) 612.5
c) 625
d) 630
e) 640.5

6). 5, 8, 12, 18, 24, 30, 36, 42, ____


a) 52
b) 46
c) 48
d) 54
e) 56

7). 21, 23, 29, 53, 173, _____


a) 210
b) 893
c) 720
d) 253
e) 293

8). 10, 221, ____, 44443, 555554


a) 3333
b) 3321

www.bankingpdf.com
c) 3332
d) 3341
e) 332

9). 6, 15, 35, ____, 143, 221, 323


a) 97
b) 112
c) 77
d) 75
e) 128

10). 1:1, 8:4, 9:27, 64:16, 25:125, ?:?, 49:343,


a) 36:316
b) 216:36
c) 316:16
d) 32:316
e) None of these

Answers:
1)c 2)d 3)e 4)a 5)b 6)a 7)b 8)c 9)c 10)b

Solution:

Directions (Q. 1-5):


2009 2010 2011 2012 2013 Total

Paddy 70 90 110 130 150 550

www.bankingpdf.com
Barley 100 130 160 140 150 680

Oats 60 80 140 180 200 660

230 300 410 450 500

1). Combined Production = 550 + 660 = 1210


Answer: c)

2).
2009 2010 2011 2012 2013

Barley 100 130 160 140 150

Oats 60 80 140 180 200

Combined 160 210 300 320 350

Growth 31.25% 42.86% 6.67% 9.38%

Answer: d)

3). Total cost in 2011 = 160×4 + 140×3 = 1060


Total cost in 2009 = 100×4 + 60×3 = 580
Increase = (1060 - 580) × 100 / 580 = 82.75%
Answer : e)

4). Required Percentage = (150×100) / 680 = 22.05%


Answer: a)

www.bankingpdf.com
5). 2014 Barley = 1.25×150
2014 Oats = 1.15×200
2014 Paddy = 1.3×150
Total = 612.5
Answer: b)

6). Each term in the series is the addition of successive prime numbers. Like Prime
numbers are2, 3, 5, 7, 11, 13, 17, 19, 23, 29 and so on. So first term is addition of (2 +
3). Ie 5, then second term is (3 + 5) ie 8, then (5+7) ie. 12, then (7+11) i.e. 18, then
(11+13) i.e. 24, then (13+17)ie. 30, then (17+19)ie.36, then (19+23)ie., 42, then
(23+29)i.e. 52.
Answer: a)

7). Difference between each successive tam is factorial of number starting from 2, like
23-21 = 2, then 29-23 = 6 means 3!, then 53-29 = 24 means 4!, then next difference is
173-53 = 120 means 5!, so next difference would be 6! i.e. 720 so term will be 173+720
= 893.
Answer: b)

8). This series is formed by subtracting 1 from the series of 11, 222, 3333, 44444,
555555 and so on. So the missing tam is 3333-1 = 3332.
Answer: c)

9). Number in each term is the product of the successive prime numbers starting from
two, for first term 2×3 = 6, for second term 3×5 = 15, for third term 5×7 = 35, so missing
term is 7×11 = 77.

www.bankingpdf.com
Answer: c)

10). Its alternatively ratio of n2 and n3


Now, after 25:125, it would be 63 : 62  216:36

Answer: b)

910-920 Questions :

Directions (Q. 1-5): Study the following information to answer the given questions:
The pie-chart shows the percentage of different types of employees in a bank and the
table shows the percentage of employees recruited through two modes for the different
posts among them in the bank.

www.bankingpdf.com
1). Promotee in Officer Scale II is what per cent of direct-recruits in Officer Scale II?
a) 120%
b) 130%
c) 150%
d) 160%
e) None of these

2). What is the total number of direct-recruit Clerk?


a) 945
b) 968
c) 975
d) 960
e) None of these

3). Find the total number of employees of direct-recruit Probationary Officer, Officer
Scale II and Clerk.
a) 2055
b) 2035
c) 2045
d) 2065

www.bankingpdf.com
e) 2040

4). Find the total number of employees in direct-recruit Officer Scale III and Promotee
Clerk cadre.
a) 1400
b) 1470
c) 1685
d) 1800
e) 1600

5). What is the difference between direct-recruit Probationary Officers and promotee
Probationary Officers?
a) 700
b) 800
c) 900
d) 600
e) None of these

Directions (Q. 6-10): What will come in place of the question mark(?) in the following
number series?

6). 5 ? 4 7.5 17 45
a) 3.5
b) 3
c) 2.5
d) 2
e) None of these

7). 2 9 30 105 ? 2195


a) 432
b) 426
c) 440

www.bankingpdf.com
d) 436
e) None of these

8). 1 3 9 31 ? 651
a) 97
b) 127
c) 129
d) 109
e) None of these

9). 3 4 12 45 ? 1005
a) 152
b) 198
c) 144
d) 192
e) None of these

10). 15 30 ? 40 8 48
a) 10
b) 20
c) 18
d) 12
e) None of these

Answers:

1)c 2)d 3)e 4)e 5)b 6)b 7)d 8)c 9)e 10)a

Solution:

1). The number of promotee Officer Scale II

www.bankingpdf.com
= 15/100 × 8000 × 60/100 = 720
The number of direct-recruit Officer Scale II
= 15/100 × 8000 × 40/100 = 480
Required% = 720/480 × 100 = 150%
Answer: c)

2). The number of direct-recruit Clerk


= 1/5 × 8000 × 3/5 = 960
Answer: d)

3). Required number


= ¼ × 8000 × 3/10 + 15/100 × 8000 × 2/5 + 1/5 × 8000 × 3/5
= 2040
Answer: e)

4). Required number


= 30/100 × 8000 × 40/100 + 20/100 × 8000 × 40/100
= 960 + 640 = 1600
Answer: e)

5). Number of Probationary Officers = 1/4 × 8000 = 2000


Required difference = 7/10 × 2000 – 3/10 × 2000 = 1400 - 600 = 800
Answer: b)

6). The series is ×0.5 + 0.5, ×1 + 1, ×1.5 + 1.5, ...


Answer: b)

www.bankingpdf.com
7). The series is ×1 + 1 × 7, ×2 + 2 × 6, ×3 + 3 × 5...
Answer: d)

8). The series is ×1 + 2, ×2 + 3, ×3 + 4, ...


Answer: c)

9). The series is ×1+12, ×2 + 22 , ×3 + 32 , ...


Answer: e)

10). The series is ×2, ÷3, ×4, ÷5, ...


Answer: a)

920-930 Questions :

1). A man can row 15 km/hr in still water. It takes him twice as long to row upstream as
to row downstream. Find the rate of stream?
a) 5 km/hr
b) 10 km/hr
c) 7.5 km/hr
d) 6 km/hr
e) 8 km/hr

2). The monthly incomes of two persons are in the ratio 5 : 4 and their monthly
expenditures are in the ratio 9 : 7. If each person saves Rs. 500 per month, then what
are their monthly incomes?
a) Rs 8000 and Rs 10000
b) Rs 3750 and Rs 3000

www.bankingpdf.com
c) Rs 5000 and Rs 4000
d) Rs 4500 and Rs 4000
e) None of the above

3). Two equal sums are lent at the same time at 6% and 5 % simple interest
respectively. The former is recovered 24 months earlier than the latter. And the amount
in each case is Rs. 2400. Find the sum of money lent initially.
a) 1500
b) 2000
c) 2400
d) 3000
e) 3600

4). The average monthly expenditure of a family was Rs. 2200 during the first 3 months;
Rs. 2250 during the next 4 months and Rs. 3120 during the last 5 months of a year. If
the total savings during the year were Rs. 1260, then the average monthly income was:
a) Rs. 2605
b) Rs. 2680
c) Rs. 2705
d) Rs. 2745
e) Rs. 2595

5). What is the size of the longest pencil that can fit in a box with dimensions 12 cm× 10
cm× 8cm?
a) 15
b) 15.5
c) 17
d) 17.5
e) 18

www.bankingpdf.com
6). A committee of 5 members is to be formed from 8 men and 5 women. How many
committees can be formed if at least 2 women and 1 man have to present in the
committee?
a) 800
b) 840
c) 880
d) 920
e) 996

7). A man buys milk at Rs. 2.40 per litre. He adds one-third water to it and sells the
mixture at Rs. 2.88 per litre. Find his profit percentage. (Assume water is free).
a) 21%
b) 30%
c) 40%
d) 50%
e) 60%

8). A can build up a wall in 8 days while B can break it in 3 days. A has worked for 4
days and then B joined to work with A for another 2 days only. In how many days will A
alone build up the remaining part of the wall?
a) 13/3 days
b) 22/3 days
c) 7/4 days
d) 7 days
e) None of these

www.bankingpdf.com
9). The sum of 2 numbers is 216, and their HCF is 27. How many such pairs of numbers
exist? (A pair a unique set of 2 numbers, so (a, b) and (b, a) will be considered as 1
pair).
a) 0
b) 1
c) 2
d) 3
e) 4

10). Ajay reads at an average rate of 30 pages per hour, while Vikram reads at an
average rate of 40 pages per hour. If Ajay starts reading a novel at 4:30 PM and Vikram
begins reading an identical copy of the same book at 5:20 PM, at what time will they be
reading the same page?
a) 7:50 PM
b) 8:15 PM
c) 8:40 PM
d) 9:00 PM
e) 9:25 PM

Answers:
1)a 2)c 3)a 4)c 5)d 6)c 7)e 8)b 9)c 10)a

Solution:
1). Let the man’s upstream speed be x km/hour and downstream speed will be 2x
km/hour
Speed in still water = 1/2 (2x + x) = 3x/2 km/hour
3x/2 = 15
X = 10
Hence, his downstream speed = 20km/hour

www.bankingpdf.com
Rate of stream = 1/2 (20-10) = 5km/hour
Answer: a)

2). Let the incomes of two persons be 5x and 4x


let the expenditures of them be 9y and 7y
Each person saves Rs. 500
Therefore,
5x – 9y = 500
4x – 7y = 500
Solving the above equations,
20x – 36y = 2000
20x – 35y = 2500
Y = 500
Therefore,
5x = 500 + 9y
5x = 500 + 9(500) = 5000
X = 1000
Hence, their salaries are Rs. 5000 and Rs. 4000
Answer: c)

3). Let the sum be S, time be ‘n’ years for 6%. n+2 years for 5%
2400 = S + S ×n × 6/100
2400 = S + S × (n+2) × 5/100
So, 6n = 5(n+2) or n = 10 years.
2400 = S + S × 10 × 6/100 = 1.6S
S = 2400/1.6 = 1500
Answer: a)

4). Total expenditure for first 3 months = 2200 × 3 = Rs 6600

www.bankingpdf.com
Total expenditure for next 4 months = 2250 × 4 = Rs9000
Total expenditure for last 5 months = 3120 × 5 = Rs 15600
Total expenditure = 6600 + 9000 + 15600 =Rs 31200
Total income = expenditure + savings = 31200 + 1260 = Rs 324060
Average monthly income = 32460/12 = Rs 2705
Answer: c)

5). The longest pencil that can be fit will be the diagonal length
Required length = (122 + 102 +82)1/2 ≈17.5cm
Answer: d)

6). The various possible cases are:


i. 2 women and 3 men = 5C2 × 8C3 = 10 × 56 = 560
ii. 3 women and 2 men = 5C3 × 8C2 = 10 × 28 = 280
iii. 4 women and 1 men = 5C4 × 8C1 = 5 × 8 = 40
Total = 560 + 280 + 40 = 880
Answer: c)

7). Let say he purchases 3 litres of milk whose cost is Rs. 7.20
He adds 1 litre of water and the mixture of 4 litres is sold at Rs 2.88 × 4 = Rs11.52
Profit percentage = (11.52 – 7.20) × 100.0/7.20 = 4.32 × 100.0/7.2 = 60%
Answer: e)

8). Amount of work done by A in one day = 1/8th of the total work
B can break 1/3rd of the work in one day
Therefore, if both of them are working together then the work done in one day = 1/8 –
1/3 = -5/24
Negative sign indicates that, work is more broken than built in one day

www.bankingpdf.com
Amount of work done by A in four days = 1/2 of the total work
Work done by both A and B for two days = -10/24 = -5/12
Total work done by the end of 6 days = 1/2 - 5/12 = 1/12
Remaining work = 1 – 1/12 = 11/12
Time taken by A to finish 11/12th of the work = 11/12 × 8 = 22/3 days
Answer: b)

9). Let the numbers be a,b


A = 27m, b = 27n where m, n are co-prime
27(m + n) = 216, or m+n = 8
M = 1, n = 7 and m = 3, n = 5 are the only 2 co – prime pairs possible.
So we have 2 pairs: (27 , 189) and ( 81, 135)
Answer: c)

10). Ajay reads at average rate of 30 pages per hour ie. 1 page in every two minutes.
Therefore, by 5:20 PM, he would have covered 25 pages
By 6:20 PM, Ajay would have read 25+30 pages and Vikram would have read 40 pages.
By 7:20 PM, Ajay would have read 85 pages and Vikram would have read 80 pages
In next half an hour, Ajay would cover 15 pages and he will finish 100 pages
Vikram would cover 20 pages and he will also finish 100 pages
Hence, both of them will be on same page at 7:50
Answer: a)

www.bankingpdf.com
930-940 Questions :

Directions (Q. 1-5): In each of these questions, two equations (I) and (II) are given. You
have to solve both the equations and give answer
a) if x > y
b) if x ≥ y
c) if x < y
d) if x ≤ y
e) if x = y or no relation can be established between x and y.

1). I. 15x2 – 41x + 14 = 0


II. 2y2 – 13y + 20 = 0

2). I. x2 – 7x = 0
II. 2y2 + 5y + 3 = 0

3). I. 3x2 – 29x + 56 = 0


II. 3y2 – 5y – 8 = 0

4). I. 5x2 + 26x – 24 = 0


II. 5y2 – 34y + 24 = 0

5). I. 7x – 4y = 40
II. 8x + 8y = 8

Directions (Q. 6-10): Study the following pie-chart and table carefully and answer the
questions given below:
A survey was conducted on 6800 students studying in various colleges having various
favourite fruits.

www.bankingpdf.com
The pie-chart shows the percentage-wise distribution among the people.

The table shows the ratio of male to female


Male Female

Cherry 3 5

Jackfruit 3 4

Lemon 5 3

Papaya 1 3

Pears 7 5

Pineapple 1 5

6). What is the ratio of the number of males whose favourite fruit is Lemon to that of the
number of females whose favourite fruit is Jackfruit?
a) 268:179
b) 255:272

www.bankingpdf.com
c) 274:341
d) 265:465
e) 284:514

7). What is the numbers of females who like Cherry the most?
a) 1384
b) 1380
c) 1275
d) 1470
e) 1290

8). What is the ratio of the number of males whose favourite fruit is Jackfruit to the
number of females whose favourite fruit is Pears?
a) 418:425
b) 425:408
c) 408:425
d) 204:425
e) 510:408

9). The number of females whose favourite fruit is Pineapple is by what per cent more
than the number of females whose favourite fruit is Papaya?
a) 81.81%
b) 83.01%
c) 82.52%
d) 82.78%
e) 85.21%

www.bankingpdf.com
10). What is the difference between the number of males whose favourite fruit is Cherry
and the number of females whose favourite fruit is Papaya?
a) 535
b) 504
c) 420
d) 204
e) 468

Answers:
1)c 2)a 3)b 4)d 5)a 6)b 7)c 8)c 9)a 10)d

Explanation:
1). I. 15x2 - 4!x + 14 = 0
or 15x2 - 6x - 35x + 14 = 0
or 3x(5x - 2) - 7(5x - 2) = 0
or (3x - 7)(5x - 2) = 0
x = 7/3 , 2/5
II. 2y2 - 13y + 20 = 0
or 2y2 - 8y - 5y + 20 = 0
or 2y(y - 4) - 5(y - 4) = 0
or (2y - 5) (y - 4) = 0
y = 4,5/ 2
x<y
Answer: c)

2). I. x2 - 7x = 0

www.bankingpdf.com
or x (x - 7) = 0
x = 0, 7
II. 2y2 + 5y + 3 = 0
or 2y2 + 2y + 3y + 3 = 0
or 2y(y + 1) + 3(y + 1) = 0
or (2y + 3) (y + 1) = 0
y = -1, -3/2
x>y
Answer: a)

3). I. 3x2 - 29x + 56 = 0


or 3x2 - 21x - 8x + 56 = 0
or 3x(x - 7) - 8(x - 7) = 0
or (3x - 8) (x - 7) = 0
x = 8/3 ,7
II. 3y2 - 5y - 8 = 0
or 3y2 + 3y - 8y - 8 = 0
or 3y(y + 1) - 8(y + 1) = 0
or (3y - 8) (y + 1) = 0
or (3y - 8) (y + 1) = 0
y =-1, 8/3
x≥y
Answer: b)

4). I. 5x2 + 26x - 24 = 0


or 5x2 + 30x - 4x - 24 = 0
or 5x(x + 6) - 4(x + 6) = 0
or (5x - 4) (x + 6) = 0

www.bankingpdf.com
x = 4/5, 6
II. 5y2 - 30y - 4y + 24 = 0
or 5y(y - 6) - 4(y - 6) = 0
or (5y - 4) (y - 6) = 0 4
y = 4/5, 6
x≤y
Answer: d)

5). 7x - 4y = 40 ...(i)
and 8x + 8y = 8
or x + y = 1 ...(ii)
Solving (i) and (ii), we have
x = 4, y = -3
x>y
Answer: a)

6). Number of males whose favourite fruit is Lemon


= 6800 × 12/100 × 5/8 = 510
Number of females whose favourite fruit is Jackfruit
= 6800 × 14/100 × 4/7 = 544
Required ratio = 510 : 544 = 255 : 272
Answer: b)

7). Number of females whose favourite fruit is Cherry


= 6800 × 30/100 × 5/8 = 1275
Answer: c)

www.bankingpdf.com
8). Required ratio = 408/425 = 408 : 425
Answer: c)

9). Number of females whose favourite fruit is Pineapple


= 6800 × 18/100 × 5/6 = 1020
Number of females whose favourite fruit is Papaya
= 6800 × 11/100 × ¾ = 561
Required % = 1020 – 561/561 × 100 = 45900/561 = 81.81% more
Answer: a)

10). Number of males whose favourite fruit is Cherry


= 6800 × 30/100 × 3/8 = 765
Number of females whose favourite fruit is Papaya
= 6800 × 11/100 × ¾ = 561
Difference = 765 - 561 = 204
Answer: d)

940-950 Questions :

1). In a class there are 15 boys and 10 girls. Three students are selected at random.
The difference between the probability that 2 boys and 1 girl are selected compared to 1
boy and 2 girls are selected is:
a) 23/78
b) 19/88
c) 15/92

www.bankingpdf.com
d) 4/23
e) 7/46

2). Ravi borrowed some money at the rate of 4 p.c.p.a. for the first three years, at the
rate of 8 p.c.p.a. for the next two years and at the rate of 9 p.c.p.a. for the period
beyond 5 years. If he pays a total simple interest of 19,550 at the end of 7 years, how
much money did he borrow?
a) 39,500
b) 42,500
c) 41,900
d) 43,000
e) None of these

3). Suresh and Ramesh are twins. In a form by mistake, Suresh reverses the last 2
digits of his year of birth, and this makes him 9 years older than Ramesh. With this
mistake, the sum of their ages in 2015 becomes 43. How old will Ramesh be 2021?
a) 17
b) 19
c) 21
d) 23
e) 25

4). A dealer sold a radio at a loss of 2.5%. Had he sold it for Rs. 120 more, he would
have gained 7.5%. In order to gain 12.5% after a 25% discount, the marked price
should be:
a) 1750
b) 1800
c) 1857.75
d) 1925

www.bankingpdf.com
e) None of these

5). A father runs after his son, who is 1000 meters ahead. The father runs at a speed of
1 kilometre every 8 minutes, and the son runs at a speed of 1 kilometer every 12
minutes. How much distance has the son covered at the point when the father
overtakes him?
a) 2500 meters
b) 2000 meters
c) 1500 meters
d) 1000 meters
e) 1200 meters

6). 24+(72 ÷ 48×8+14) × 36 ÷ (3×4) + 36 ÷ 3 ×4


a) 120
b) 140
c) 160
d) 180
e) None of these

7). Evaluate: 2+√2 + 1/(2√2) + 1/(√2 - 2)


a) 1 + 0.75√2
b) 2 + 1.5√2
c) 2 + √2
d) 2 - √2
e) None of these

8). Evaluate: 48×52 + 61×59 + 77×83


a) 12486

www.bankingpdf.com
b) 10996
c) 13406
d) 13206
e) None of these

9). Evaluate: (0.625 × 0.0729 × 28.9)/(0.0081 × 0.025 × 1.7)


a) 382.5
b) 3725
c) 3625
d) 3825
e) None of these

10). Evaluate: (4 + 4×18 – 6 - 8) / (123×6 - 146×5)


a) 7.5
b) 7.75
c) 8
d) 8.25
e) 8.5
Answers:
1)c 2)b 3)d 4)b 5)b 6)e 7)a 8)a 9)d 10)b

Solution:
1). 2 boys, 1 gi(Prl = (15c2×10c1) / 25c3 = 1050/2300
1 boy, 2 girls = (15c1×10c2) / 25c3 = 675/2300
Difference = (1050 - 675)/2300 = 375/2300 = 15/92
Answer: c)

2). Let the amount borrowed be Rs P

www.bankingpdf.com
Total amount of simple interest = (P×3×4)/100 + (p×2×8)/100 + (p×2×9)/100 = 19550
0.12P + 0.16P + 0.18P = 19550
0.46p= 19550
P = Rs. 42500
Answer: b)

3). Let the correct ages of Ramesh and Suresh be X in 2015 (since they are twins, they
will have the same age)
Given: 2x + 9 = 43
Or x = 17 years in 2015.
Age in 2021 = 17 + 6 = 23
Note: year of birth = 2015 – 17 = 1998. Reversing, Suresh = 1989 or 9 year older.
Answer: d)

4). 120 = 10% of the CP (2.5% loss to 7.5% profit)


Or CP = Rs. 1200
To gain 12.5%, SP = 1200 × 1.125 = Rs. 1350
Rs. 1350 = 25% discounted of Marked Price
Or marked Price = 1350/0.75 = Rs. 1800
Answer: b)

5). Time taken is same for both father and son


Let x be the distance son travelled before being overtaken by his father
Time taken by son = x/5 km/hour
Time taken by father = (1000 + x) / 7.5 km/hr
Since, time taken is same
x/5 = (1000 + x) / 7.5 or x = 2000m
Answer: b)

www.bankingpdf.com
6). 72÷48×8+14 = (72/48) × 8 + 14 = (3/2) × 8 + 14 = 12 +14 = 26
24 + (72 ÷ 48 × 8 + 14) ×36 ÷ (3 × 4) + 36 ÷ 3 × 4
= 24 + 26 × 36÷12 + 36 ÷ 3 × 4
= 24 + 26 × 3 + 12 × 4
= 24 + 78 + 48 = 150
Answer: e)

7). 1/(√2 - 2) = (√2 + 2)/(2 - 4) = (√2 + 2)/ -2


1/(2√2) = √2/4
The given equation becomes: 2 + √2 +√2/4 – (√2 + 2) / 2
= (4(2 + √2) + √2 – 2(√2 + 2)) / 4
= (4 + 3√2) / 4 = 1 + 0.75√2
Answer: a)

8). 48×52 = (50 - 2) × (50 + 2) = 502 – 22 = 2496


61 × 59 = (60 + 1) × (60 - 1) = 602 – 12 = 3599
77 × 83 = (80 - 3) × (80 + 3) = 802 – 32 = 6391
Sum = 2496 + 3599 + 6391 = 12486
Answer: a)

9). (0.625×0.0729×28.9) / (0.0081×0.025×1.7) = (625/25) × (729/81) × (289/17) = 25×9×


17 = 3825.
Answer: d)

10). Numerator = 4 + 72 – 14 = 62
Denominator = 738 – 730 = 8

www.bankingpdf.com
Required value = 62/8 = 7.75
Answer: b)

950-960 Questions :

Directions (Q. 1-5): Study the following line graph and the table and answer the
questions given below:
Percentage of population below poverty line in different states of India from 2011 to
2016

The bar chart shows the sex ratio per 10 males in different states below poverty line

www.bankingpdf.com
1). The population of Karnataka in the year 2012 is 40 lakh. If there is an annual growth
of 10% in the population of Karnataka from year 2012 to 2014 then what is the
percentage increase or decrease in the number of males below poverty line in the year
2014 with respect to that in the year 2012?
a) 21% increase
b) 15% increase
c) 14% increase
d) 18% decrease
e) None of these

2). What is the percentage of the population below poverty line in the year 2013 in
Kerala with respect to that in all the years from 2011 to 2016 ?
a) 18.66%
b) 20.33%
c) 40.66%
d) 30.66%
e) Can’t be determined

www.bankingpdf.com
3). If the population of Kerala and C in the year 2015 was 55 lakh and 62 lakh
respectively then what will be the ratio of the females below poverty line in Kerala to that
of the females below poverty line in Karnataka in the year 2015?
a) 85:99
b) 82:97
c) 109:124
d) 97:123
e) None of these

4). If there is an increase of 10% in the population of Tamil Nadu in the year 2013, then
how many females are there who are below poverty line in that state in the year 2012, if
the population in 2013 was 55 lakh in that state?
a) 4 lakh
b) 5.2 lakh
c) 4.9 lakh
d) 3.05 lakh
e) None of these

5). If in the year 2015 the population of Tamil Nadu, B and C was 60 lakh, 55 lakh and
62 lakh respectively, then what is the total population below poverty line in the year
2015 in all three states?
a) 75.60 lakh
b) 64.9 lakh
c) 74.9 lakh
d) 66.50 lakh
e) None of these

www.bankingpdf.com
Directions (Q. 6-10): What should come in place of question mark (?) in the following
sums?
6). 6000 ÷ 15 × 225 – 70000 = ?
a) 20000
b) 18000
c) 16000
d) 14000
e) 15000

7). 36 – 0.036 × 100 = ?


a) 3.6
b) 36.6
c) 29.6
d) 32.4
e) None of these

8). 3 1/7 + 4 2/5 - ? = -2 3/5


a) a) 10 1/7
b) b) 7 1/10
c) c) 8 8/7
d) d) 9 8/7
e) None of these

9). 0.003 × ? × 0.0003 = 0.00009


a) 10
b) 3
c) 0.03
d) 0.003
e) None of these

www.bankingpdf.com
10). 5287 – 176.22 – 78.584 = ?
a) 5023.196
b) 5032.196
c) 5302.196
d) 5203.196
e) None of these

Answers:
1)a 2)e 3)c 4)a 5)b 6)a 7)d 8)a 9)e 10)b

Solution:
1). Population of Karnataka in the year 2012 = 40 lakh
Number of males below poverty line in Karnataka in the year 2012
= 40 × 45/100 × 10/15 = 12 lakh
Population of Karnataka in 2014 = 40 + 40 × 21/100 = 48.4 lakh
Number of males below poverty line in Karnataka in 2014
= 48.4 × 42/100 × 10/14 = 14.52 lakh
Required % increase = (14.52 - 1b)/12 × 100 = 2.52/12 × 100 = 21%
Answer: a)

2). Total population in any year is not given, so we cannot determine the population of
all the states in 2015.
Answer: e)

3). The number of females below poverty line, in Kerala in the year 2015
= 55 × 38/100 × 10.9/20.9

www.bankingpdf.com
= 20.9 × 10.9/20.9 = 10.90 lakh
Again, In Karnataka in the year 2015 = 62 × 40/10 × 10/20 = 12.4 lakh.
Required ratio =109/124 = 109 : 124
Answer: c)

4). Population of Tamil Nadu in the year 2013 = 55 lakh


Population of Tamil Nadu in the year 2012 = 50 lakh
The number of females below poverty line in Tamil Nadu in the year 2012
= 50 × 24/100 × 5/15 = 4 lakh
Answer: a)

5). Population of A below poverty line in the year 2015


= 60 × 32/100 = 19.2 lakh
Population of B below poverty line in the year 2015
= 55 × 38/100 = 20.9 lakh
Population of C below poverty line in the year 2015
= 62 × 40/100 = 24.8 lakh
Total population below poverty line in the year 2015 = 19.2 + 20.9 + 24.8 = 64.9 lakh
Answer: b)

6). ? = 6000 – 225 / 15 - 70000 = 90000 – 70000 = 20000


Answer: a)

7). ? = 36 – 3.6 = 32.41


Answer: d)

8). 3 1/7 + 4 2/5 - ? = -2 3/5

www.bankingpdf.com
22/7 + 22/5 + 13/5 = 355/35 = 71/7 = 10 1/7
Answer: a)

9). ? = 0.00009 / 0.003 × 0.0003 = 100


Answer: e)

10). 5287 – 176.22 – 78.584 = 5032.196


Answer: b)

960-970 Questions :

1). Is X divisible by 12?


a. X leaves a remainder 2 when divided by 8
b. X is divisible by 3.
c. X is divisible by 6.
a) If the data in statement a is sufficient to answer the question, while the data in
statement b and c are not required to answer the question.
b) If the data in statement b is sufficient to answer the question, while the data in
statement a and c are not required to answer the question
c) If the data in statement a and b are sufficient to answer the question, while the data
in statement c is not required to answer the question.
d) If the data in statement a, b and c together are necessary to answer the question.
e) If the data in statement a, b and c together are not sufficient to answer the question.

2). What is the average score in an exam taken by 500 students where the minimum
score is 200?
a. Half the students scored above 700.
b. Half the students scored below 700.

www.bankingpdf.com
c. The maximum score in the exam was 850, scored by exactly 42 students.
a) If the data in statement a is sufficient to answer the question, while the data in
statement b and c are not required to answer the question
b) If the data in statement b is sufficient to answer the question, while the data in
statement a and c are not required to answer the question
c) If the data in statement a and b are sufficient to answer the question, while the data
in statement c is not required to answer the question
d) If the data in statement a, b and c together are necessary to answer the question.
e) If the data in statement a, b and c together are not sufficient to answer the question.

3). What is the wholesale cost of a dress?


a. The dress was listed at a price that would have given the store a profit of 20 percent
of the wholesale cost.
b. After as 10% discount on the list price, the dress sold for a net profit of 10 Rupees.
c. The dress sold for 50 Rupees more than the wholesale cost.
a) If the data in statement a is sufficient to answer the question, while the data in
statement b and c are not required to answer the question
b) If the data in statement b is sufficient to answer the question, while the data in
statement a and c are not required to answer the question
c) If the data in statement a and b are sufficient to answer the question, while the data
in statement c is not required to answer the question
d) If the data in statement a, b and c together are necessary to answer the question.
e) If the data in statement a, b and c together are not sufficient to answer the question.

4). Is p>q?
(A) 0<p<0.5
(B) q>0.4
a) If statement (A) alone is sufficient to answer the question but statement (B) alone is
not sufficient.

www.bankingpdf.com
b) If statement (B) alone is sufficient to answer the question but statement (A) alone is
not sufficient.
c) If the two statements taken together are sufficient to answer the question, but neither
statement alone is sufficient
d) If each statement alone is sufficient to answer the question
e) If the two statements taken together are still not sufficient to answer the question.

5). What is the remainder when positive integer ‘p’ is divided by 3?


(A) ‘p’ is an even number
(B) ‘p’ is a perfect square
a) If statement (A) alone is sufficient to answer the question but statement (B) alone is
not sufficient.
b) If statement (B) alone is sufficient to answer the question but statement (A) alone is
not sufficient.
c) If the two statements taken together are sufficient to answer the question, but neither
statement alone is sufficient
d) If each statement alone is sufficient to answer the question
e) If the two statements taken together are still not sufficient to answer the question.

Directions (Q. 6-10): In the following questions three equations numbered I, II and III are
given. Solve the equations and choose the correct option that gives the relation
between the variables.
a) x > y = z
b) x ≥ y ≥ z
c) x < y > z
d) x > y < z
e) None of these
6). I. 3x + y = 30
II. 2y + 5z = 48

www.bankingpdf.com
III. 5x – 4z = 11

7). I. 2x + 5y = 19.6
II. y = √(7.84)
III. 10x – 7z = 8.4

8). I. z – 3x = -6
II. 5x + 2y = 22.5
III. 3y + 2z = 16.5

9). I. x + 3y – 7z = -7
II. 3y + 2z = 15
III. 3z – x = 4

10). I. x2 - 11x + 28 = 0
II. y2 – 7y + 12 = 0
III. z2 – 4z + 3 = 0

Answers:
1)a 2)e 3)c 4)e 5)e 6)c 7)e 8)d 9)a 10)b

Solution:
1). From (a), we notice that the number is of the form 8n+2, which means it also leaves
a remainder 2 on being divided by 4. So, its not divisible by 12.

www.bankingpdf.com
From (b) and (c) together, the number can be any multiple of 6, from which we cant
conclusively say if number is divisible by 12.
Answer: a)

2). The fact that half scored above 700, and other half less than 700 does not tell the
actual distribution of scores to calculate the average. Statement (c) also does not help
us determine the average.
Answer: e)

3). Let C be the wholesale price, listed Price be P. from (a), P = 1.2C
From (b), 0.9P = C + 10. Using statement (a), 1.2C = C + 10, from this we can
determine C.
From (C), we have sales price = C + 50. From this we cant determine C, even if we
combine information from other statements.
So (a) and (b) together are sufficient to answer the question.
Answer: c)

4). We can have p = 0.45, and q = 0.41. here p>q


We can also have p = 0.45,and q = 10. Here p<q
So, we cant determine using both statements if p>q.
Answer: e)

5). Using (A), we can have ‘p’ as 2/4/6/8 etc. we cant determine remainder when divided
by 3.
Using (B), we can have ‘p’ as 1/4/9/16 etc. we cant determine remainder when divided
by 3.
Using both ‘p’ is the square of an even number. So ‘p’ can be 4/16/36 etc.
We see that remainder can again be 1/0 (4/16 leave remainder 1, 36 remainder 0).

www.bankingpdf.com
So, we cant determine using both statements.
Answer: e)

6). 2xI – II  6x – 5z = 12(IV)


IV – III  x – z = 1 or x = z + 1
Substituting in III, we get x = 7, z = 6
Putting x = 7 in I, we get y = 9
Hence x < y > z
Answer: c)

7). From II, y = 2.8


Putting in I, we get 2x + 14 = 19.6
= 2x = 5.6 or x = 2.8
Putting x = 2.8 in III, we get 28 – 7z = 8.4
= 7z = 19.6 or z = 2.8
Hence x = y = z
Answer: e)

8). III – 2xI  3y + 6x = 28.5 or 2y + 4x = 19(IV)


II – IV  x = 3.5
Putting in I, z – 10.5 = -6
Z = 4.5
Putting in II, 17.5 + 2y = 22.5
Y = 2.5
Hence x > y < z
Answer: d)

9). From II, 3y = -2z + 15

www.bankingpdf.com
From III, x = 3z -4
Putting in I, 3z – 4 – 2z + 15 – 7z = -7
 -6z + 11 = -7
z=3
Substituting in II and III, we get
x = 5, y =3
hence x > y = z
Answer: a)

10). From I, (x-7) (x-4) = 0


x = 7,4
From II, (y-4) (y-3) = 0
y = 4, 3
From III, (z-3) (z-1) = 0
z = 3, 1
hence x ≥ y ≥ z

Answer: b)

970-980 Questions :

Directions (Q. 1-5): study the following table carefully to answer the questions that
follow:
Production of Sugar (in tonnes) of three different States over the years

www.bankingpdf.com
1). What is the average production of sugar of State A for all the years together?
a) 6.24 tonnes
b) 6.3 tonnes
c) 7.1 tonnes
d) 6.1 tonnes
e) None of these

2). What is the approximate percentage increase in production of sugar in state B from
2006 to 2007?
a) 12
b) 18
c) 24
d) 10
e) 21

www.bankingpdf.com
3). What is the difference between production of sugar of all the three states together in
2008 and 2005 respectively?
a) 6.9 tonnes
b) 4.3 tonnes
c) 6.1 tonnes
d) 5.1 tonnes
e) None of these

4). What is the average production of sugar of all the three states in 2003 and 2004
together?
a) 4.1 tonnes
b) 4.7 tonnes
c) 5.1 tonnes
d) 4.8 tonnes
e) None of these

5). What is the ratio between total production of sugar of all the three states in year
2006 and 2007 respectively?
a) 7 : 9
b) 6 : 7
c) 8 : 7
d) 7 : 8
e) 11: 12

6). A, B, and C starts from the same place and travel in the same direction at speeds
30km/hr, 40 km/hr and 60 km/hr respectively. B starts 2 hours after A, but B and C
overtakes A at the same instant. How many hours after A did C start?
a) 1
b) 2

www.bankingpdf.com
c) 4
d) 6
e) 8

7). A bag contains Rs 510 In the form of 50 paise, 25 paise and 20 paise coins in the
ratio 2 : 3 : 4. Find the total number of coins.
a) 450
b) 1200
c) 1400
d) 1800
e) 2100

8). Rs. 1500 amounts to 1653.75 in 2 years compounded annually. At the same rate,
how much will Rs. 1800 amount to in 3 years?
a) 2000
b) 2040
c) 2080
d) 2120
e) 2160

9). Ram has a mixture of 2 Acids X and Y in a can in the ratio 2 : 3. He removes 6 litres
of Y, and replaces it with X, and the ratio between X and Y becomes 4 : 3. What was
the volume of X in the can initially?
a) 12
b) 14
c) 16
d) 18
e) Cannot be determined

www.bankingpdf.com
10). A spherical metal shall has an outer radius of 11 cm, and inner radius of 4 cm. if the
cost of metal is Rs. 12 per cm3, what is the cost of the shell?
a) Rs. 6310
b) Rs. 63712
c) Rs. 89108
d) Rs. 47234
e) Rs. 12444

Answers:
1)d 2)b 3)c 4)a 5)d 6)c 7)d 8)c 9)b 10)b

Solution:
Directions (Q. 1-5):
State C B A

Year

2003 4.3 3.1 3.9

2004 4.9 3.7 4.7

2005 5.6 4.4 5.8

2006 5.8 5.1 6.6

2007 6.7 6 7.3

2008 7.4 6.2 8.3

www.bankingpdf.com
1). Total production of sugar of state A for all the years together = 3.9 + 4.7 + 5.8 + 6.6
+ 7.3 + 8.3 = 36.6 tonnes
Average = 36.6/6 = 6.1 tonnes
Answer: d)

2). Production of sugar in state Q in 2006 = 5.1 tonnes


Production of sugar in state Q in 2007 = 6 tonnes
Percentage increase = [(6-5.1)/5.1] × 100 = 17.64%
Answer: b)

3). Total production of sugar of all the three states together in 2008 = 7.4 + 6.2 + 8.3 =
21.9 tonnes
Total production of sugar of all the three states together in 2005 = 5.6 + 4.4+ 5.8 = 15.8
tonnes
Difference = 21.9 – 15.8 = 6.1 tonnes
Answer: c)

4). Total production of sugar of all the three states in 2003 and 2004 together = (4.3 +
3.1 + 3.9) + (4.9 + 3.7 + 4.7)= 24.6 tonnes
Average = 24.6/6 = 4.1 tonnes
Answer: a)

5). Total production of sugar of all the three states in year 2006 = 5.8 + 5.1 + 6.6 = 17.5
tonnes
Total production of sugar of all the three states in year 2007 = 6.7 + 6 + 7.3 = 20 tonnes
Ratio = 17.5 : 20 = 7 : 8
Answer: d)

www.bankingpdf.com
6). Let the time taken by A when he was overtaken by B and C be T hours.
Time taken by B = T – 2 hours
Since, distance travelled is same by the time B overtakes A
30(T)= 40(T-2)
T = 8 hours
Distance travelled by A = 30×8 = 240 km
C takes 4 hours to cover a distance of 240 km
So, he starts 8-4 = 4 hours after A.
Answer: c)

7). Let the number of 50 paise, 25 paise and 20 paise coins be 2n, 3n and 4n
respectively
2n(0.5) + 3n(0.25) + 4n(0.2) = 510
n + 0.75n + 0.8n = 510
2.55n = 510
n = 200
total coins = 2n + 3n + 4n = 9n = 1800
Answer: d)

8). 1653.75 = 1500 (1 + R)2


441/400 = (1 + R)2
Solving this gives us R = 0.05 (5%)
Required Amount = 1800 × 1.05 × 1.05 × 1.05 = 2083.725 = 2080
Answer: c)

9). Let the initial volumes of X, Y be 2x, 3x.


We have (2x + 6)/(3x - 6) = 4/3

www.bankingpdf.com
6x + 18 = 12x – 24
Or x = 7
Original volume of X = 2x = 14
Answer: b)

10). Volume of Metal = (4/3) × 22/7 × (113 - 43)


= (4/3) × (22/7) × (1331 - 64)
= 4 × 22 × 181 / 3
Cost = 12× 4 × 22 × 181/3
= 4 × 4 × 22 × 181 = Rs. 63712
Answer: b)

980-990 Questions :

1). All the words of OLIVE are arranged in dictionary order. Find the position of the word
LIVEO.
a) 49
b) 55
c) 57
d) 59
e) 61

2). A man who makes a profit of 25 percent by selling sugar at Rs. 4.25/kg., lowers his
price so as to gain only 5 paise per Kg. In what ratio must his sales be increased so that
his total profit may be the same as before?
a) 1 : 5
b) 1 : 11
c) 1 : 13

www.bankingpdf.com
d) 1 : 17
e) 1 : 21

3). A contract is to be completed in 46 days and 117 men were set to work, each
working 8 hours a day. After 33 days, 4/7 of the work completed. How many additional
men may be employed so that the work may be completed in time, each man now
working 9 hours a day?
a) 79
b) 80
c) 81
d) 82
e) 83

4). The LCM of two numbers is 45 times their HCF. If one of the numbers is 125 and the
sum of HCF and LCM is 1150, then the other number is?
a) 100
b) 125
c) 250
d) 225
e) 180

5). Ashwini has a certain number of Pencils she wishes to distribute in the class. when
she tries to distribute 7 pencils to each student, 3 pencils are left out. if the class had 3
less students, each student would have got 2 pencils more, and no pencils would have
been left out. How many pencils does Ashwini have?
a) 90
b) 120
c) 140
d) 160

www.bankingpdf.com
e) None of these

6). An amount becomes 160% of itself in 5 years through simple interest. At the same
rate of interest, how long will it take for the amount to more than double itself if we use
compound interest?
a) 7
b) 8
c) 9
d) 10
e) cannot be determined

7). Pipe A usually fills a tank in 2 hours. On account of a leak at the bottom of the tank,
it takes pipe A 30 more minutes to fill the tank. How long will the leak take to empty a
full tank if pipe A is shut?
a) 4 hours
b) 3 hours
c) 6 hours
d) 8 hours
e) 10 hours

8). Four years ago, the average of a 4 member family was 40 years. 3 years ago, a boy
got married to a 23 years old girl, and they had a baby boy last year. what is the
average of the family 2 years from now assuming no one joins or leaves from the family
in the next 2 years.
a) 40
b) 42.5
c) 36.5
d) 34
e) 35.8

www.bankingpdf.com
9). In 10 years time, Ram will be 2 times older than his son. 5 years ago he was 9 times
as old as his son. What will be the sum of the ages of Ram and his son 3 years from
now?
a) 46
b) 66
c) 86
d) 76
e) None of these

10). A merchant dealer gives successive discounts of 10% and 20% in that order and
makes a profit of 20%. what will be the profit if he offers a single flat discount of 30%?
a) 20%
b) 16.67%
c) 18%
d) 14%
e) None of these

Answers:
1)d 2)d 3)c 4)d 5)e 6)a 7)e 8)e 9)b 10)b

Solution:
1). words starting with E = 4! = 24
words starting with I = 4! = 24
words starting with LE = 3! = 6
words starting with LIE = 2! = 2, LIO = 2! = 2
the first word with starting with LIV will be LIVEO
so rank = 24 + 24 + 6 + 2 + 2 + 1 = 59

www.bankingpdf.com
Answer: d)

2). 125% C.P. of sugar per kg = Rs. 4.25


C.P of the sugar per kg = [4.25×100] / 125 = Rs. 3.40.
thus, the profit on 1 kg 85 paise
by reducing price to 5 paise, to make same profit he must sell, 85/5 = 17kg
so required ratio = 1 : 17.
Answer: d)

3). work finished by 33 days = 33×117×8 = 30888 man hours


4/7th of the job is finished.
remaining work = 1-(4/7) = 3/7
if 4/7th of the work is 30888 hours, then 3/7th of the work = [30888 × (3/7)] / (4/7) =
23166 man hours
number of men required to finish 23166 man hours of work in 13 days, each now
working 9 hours a day = 23166 = (no of man) × 13 × 9
number of men = 198
since, 117 men are already working, additional men required = 198 – 117 = 81
Answer: c)

4). LCM = 45× HCF


LCM + HCF = 46× HCF = 1150
HCF = 25, LCM =1125.
product of 2 numbers = LCM×HCF
required number = (25× 1125) / 125 = 225.
Answer: d)

5). Let number of students be X.

www.bankingpdf.com
so, total Pencils = 7X + 3
7X + 3 = (7+2) (X-3)
or., 7X + 3 = 9X – 27
or X = 15
total Pencils = 7X + 3 = 108
Answer: e)

6). let P be the principal


160% of P amount means (1.6-1)P = 0.6P is the simple interest in 5 years
so, 0.6P = (P×5×R) / 100
or R = 12%
now we have to find time ‘n’ such that:
P(1+0.12)n = 2P
or 1.12n = 2
first we note that, time on compound interest will always be less than that of simple
interest.
at 12% simple interest, time to double will be 100/12 = 8.3 years.
so the correct answer must be less than 8 years.
so, we choose 7 as the answer from the options
also : 1 .127 = 2.21
Answer: a)

7). portion of the tank filled by the pipe A in 1 hour = ½ of the tank
let the portion of the tank emptied because of the leak in 1 hour be 1/x of the tank
therefore, in 1 hour , portion of the tank filled = 1/2 - 1/x
because of the leak, total time taken to fill the tank = 2.5 hours
therefore, portion of the tank filled in 1 hour when the pipe is open and when it is leaking
= 1/(2.5)
therefore,1/2 - 1/x = 1/2.5 or x = 10

www.bankingpdf.com
hence, it takes 10 hours to empty a full tank
Answer: e)

8). Four years ago, sum of all the family members age = 4× 40 = 160
three years ago, sum of all the family members age = 164 + 23 = 187
last year, sum = 197 (187 + 5 + 5)
two years from now, sum = 197 + 18 = 215
hence, average = 215/6 = 35.8
Answer: e)

9). Let current age of Ram and his son be R,S


(R + 10) = 3(S + 10) [2 times older = 3 times the age]
R = 3S + 20
(R - 5) = 9(S - 5)
3S + 20 – 5= 9S – 45
3S + 15 = 9S – 45
or S = 10., R = 50
sum of ages 3 years from now = 10 + 3 + 50 + 3 = 66.
Answer: b)

10). let marked price be 100.


SP with 10%, 20% = 80% of (90% of 100) = 72.
profit = 20%
so CP = 72/1.20 = Rs. 60
by offering a flat discount of 30%, sales price = 70% of 100 = 70
profit = (70 - 60) ×100 / 60 = 16.67%

Answer: b)

www.bankingpdf.com
990-1000 Questions :

1). Vinoth could get equal number of Rs. 55, Rs. 85 and Rs. 105 ticket for a Zoo. He
spend Rs. 3920 for all the tickets. How many of each did he buy?
a) 15
b) 18
c) 16
d) 26
e) None of these

2). The average speed of a train is 1 3/7 times the average speed of a car. The car
covers a distance of 616 km in 8 hours. How much distance will the train cover in 13
hours?
a) 1340 km
b) 1423 km
c) 1430 km
d) 1936 km
e) None of these

Directions (Q. 3-5): What should come in place of the question mark (?) in the following
number series?
3). 12 17 32 57 ? 137
a) 98
b) 95
c) 37
d) 92
e) none of these

www.bankingpdf.com
4). 187.5, 182, 171, ?, 132.5, 105, 72
a) 145.5
b) 155.5
c) 154.5
d) 144.5
e) None of these

5). 3 7 6 28 9 112 12 ?
a) 112
b) 448
c) 28
d) 38
e) None of these

6). Rosy buys Oats at Rs.10/kg and sell it in order to earn a profit of 40%. However, her
faulty balance shows 1000gm when it is actually 800gm. What is her actual gain
percentage?
a) 35%
b) 70%
c) 75%
d) 25%
e) None of the Above

7). Nithish bought two purses which together cost him Rs.440. He sold one of the purse
at a loss of 20% and the other one at a gain of 40%. The selling price of both purse are
same. Then find out the cost price of both purse.
a) Rs.150 and Rs.290
b) Rs.140 and Rs.300
c) Rs.170 and Rs.270

www.bankingpdf.com
d) Rs.160 and Rs.280
e) None of the Above

8). The population of city S increased from 650,000 in 2000 to 700,000 in 2010, and it is
projected that the population will increase by the same number from 2010 to 2020.
Approximately what is the projected percent increase in population from 2010 to 2020 ?
a) 7.1%
b) 7.7%
c) 8.3%
d) 14.3%
e) 15.3%

Directions: (9-10) What approximate value will come in place of the question-mark (?) in
the following questions. (You are not expected to calculate the exact value.)
9). 151.1% of 151.1 + 151.1 = ?
a) 380
b) 400
c) 350
d) 420
e) 440

10). 501/58 * 291/101 ÷ 31/155 = ?


a) 140
b) 125
c) 95
d) 85
e) 110

www.bankingpdf.com
Answers:
1)c 2)c 3)d 4)c 5)d 6)c 7)d 8)a 9)a 10)b

Solution:
1). Total money = 55x + 85x + 105x = 3920
245x = 3920
x = 16
Answer: c)

2). speed of the car = 616/8 = 77km/hr


speed of the train = 10/7 × 77 = 110 km/hr
Distance covered by train = 110 × 13 = 1430 km.
Answer: c)

3). The pattern of the number series is


12+5×1=17, 17+5×3=32, 32+5×5=57
57+5×7=92, 92+5×9=137
Answer: d)

4). The pattern is -5.5, -11, -16.5, -22 and so on.


Answer: c)

5). First series: 3, 6, 9 (increased by 3)


Second Series: 7, 28, 112, 448 (multiplied by 4)
Answer: d)

6). Let price of 1 kg rice = Rs.10.

www.bankingpdf.com
CP of 800 gm rice = Rs.8.
She wants to earn a profit of 40% on per Kg
SP = 10 + 40% of 10 = Rs. 14 per kg.
Faulty balance shows 800 gm = 1000 gm (1 kg)
She sells 800 gm for Rs.14.
Profit = 14 – 8 = Rs. 6.
Profit(%) = 6/8 * 100 = 75%.
Answer: c)

7). 80/100 * x = 140/100 * y


x = 7/4y
x + y = 440
7/4 y + y = 440
y = 160 ; x = 280
Answer: d)

8). Required percentage = (750000 - 700000)/700000 * 100 = 7.14%


Answer: a)

9).
Answer: a)

10).
Answer: b)

www.bankingpdf.com
www.bankingpdf.com

Das könnte Ihnen auch gefallen